You are on page 1of 231

PROBLEM 4.

1
The boom on a 4300-kg truck is used to unload a pallet of shingles of
mass 1600 kg. Determine the reaction at each of the two (a) rear
wheels B, (b) front wheels C.

SOLUTION

(
WA = mA g = (1600 kg ) 9.81 m/s 2 )
= 15696 N

or WA = 15.696 kN

(
WG = mG g = ( 4300 kg ) 9.81 m/s 2 )
= 42 183 N

or WG = 42.183 kN
(a) From f.b.d. of truck with boom

M C = 0: (15.696 kN ) ( 0.5 + 0.4 + 6 cos15 ) m 2FB ( 0.5 + 0.4 + 4.3) m


+ ( 42.183 kN )( 0.5 m ) = 0

126.185
2 FB = = 24.266 kN
5.2
or FB = 12.13 kN
(b) From f.b.d. of truck with boom

M B = 0: (15.696 kN ) ( 6 cos15 4.3) m ( 42.183 kN ) ( 4.3 + 0.4 ) m


+ 2 FC ( 4.3 + 0.9 ) m = 0

174.786
2 FC = = 33.613 kN
5.2
or FC = 16.81 kN

Check: Fy = 0: ( 33.613 42.183 + 24.266 15.696 ) kN = 0?


( 57.879 57.879 ) kN = 0 ok
PROBLEM 4.2
Two children are standing on a diving board of mass 65 kg. Knowing that
the masses of the children at C and D are 28 kg and 40 kg, respectively,
determine (a) the reaction at A, (b) the reaction at B.

SOLUTION

( )
WG = mG g = ( 65 kg ) 9.81 m/s 2 = 637.65 N

( )
WC = mC g = ( 28 kg ) 9.81 m/s 2 = 274.68 N

( )
WD = mD g = ( 40 kg ) 9.81 m/s 2 = 392.4 N

(a) From f.b.d. of diving board


M B = 0: Ay (1.2 m ) ( 637.65 N )( 0.48 m ) ( 274.68 N )(1.08 m ) ( 392.4 N )( 2.08 m ) = 0

1418.92
Ay = = 1182.43 N
1.2
or A y = 1.182 kN

(b) From f.b.d. of diving board


M A = 0: By (1.2 m ) 637.65 N (1.68 m ) 274.68 N ( 2.28 m ) 392.4 N ( 3.28 m ) = 0

2984.6
By = = 2487.2 N
1.2
or B y = 2.49 kN

Check: Fy = 0: ( 1182.43 + 2487.2 637.65 274.68 392.4 ) N = 0?


( 2487.2 2487.2 ) N = 0 ok
PROBLEM 4.3
Two crates, each weighing 250 lb, are placed as shown in the bed of a
3000-lb pickup truck. Determine the reactions at each of the two
(a) rear wheels A, (b) front wheels B.

SOLUTION

(a) From f.b.d. of truck


M B = 0: ( 250 lb )(12.1 ft ) + ( 250 lb )( 6.5 ft ) + ( 3000 lb )( 3.9 ft ) ( 2FA )( 9.8 ft ) = 0
16350
2 FA = = 1668.37 lb
9.8
FA = 834 lb
(b) From f.b.d. of truck
M A = 0: ( 2FB )( 9.8 ft ) ( 3000 lb )( 5.9 ft ) ( 250 lb )( 3.3 ft ) + ( 250 lb )( 2.3 ft ) = 0
17950
2 FB = = 1831.63 lb
9.8
FB = 916 lb

Check: Fy = 0: ( 250 + 1668.37 250 3000 + 1831.63) lb = 0?


( 3500 3500 ) lb = 0 ok
PROBLEM 4.4
Solve Problem 4.3 assuming that crate D is removed and that the position
of crate C is unchanged.
P4.3 The boom on a 4300-kg truck is used to unload a pallet of
shingles of mass 1600 kg. Determine the reaction at each of the two
(a) rear wheels B, (b) front wheels C

SOLUTION

(a) From f.b.d. of truck


M B = 0: ( 3000 lb )( 3.9 ft ) ( 2FA )( 9.8 ft ) + ( 250 lb )(12.1 ft ) = 0
14725
2 FA = = 1502.55 lb
9.8
or FA = 751 lb
(b) From f.b.d. of truck
M A = 0: ( 2FB )( 9.8 ft ) ( 3000 lb )( 5.9 ft ) + ( 250 lb )( 2.3 ft ) = 0
17125
2 FB = = 1747.45 lb
9.8
or FB = 874 lb

Check: Fy = 0: 2 ( 751 + 874 ) 3000 250 lb = 0?

( 3250 3250 ) lb = 0 ok
PROBLEM 4.5
A T-shaped bracket supports the four loads shown. Determine the reactions
at A and B if (a) a = 100 mm, (b) a = 70 mm.

SOLUTION

(a)

From f.b.d. of bracket


M B = 0: (10 N )( 0.18 m ) ( 30 N )( 0.1 m ) + ( 40 N )( 0.06 m ) + A ( 0.12 m ) = 0

2.400
A= = 20 N or A = 20.0 N
0.12
M A = 0: B ( 0.12 m ) ( 40 N )( 0.06 m ) ( 50 N )( 0.12 m ) ( 30 N )( 0.22 m ) (10 N )( 0.3 m ) = 0

18.000
B= = 150 N or B = 150.0 N
0.12
(b)

From f.b.d. of bracket


M B = 0: (10 N )( 0.15 m ) ( 30 N )( 0.07 m ) + ( 40 N )( 0.06 m ) + A ( 0.12 m ) = 0

1.200
A= = 10 N or A = 10.00 N
0.12
M A = 0: B ( 0.12 m ) ( 40 N )( 0.06 m ) ( 50 N )( 0.12 m ) ( 30 N )( 0.19 m )

(10 N )( 0.27 m ) = 0

16.800
B= = 140 N or B = 140.0 N
0.12
PROBLEM 4.6
For the bracket and loading of Problem 4.5, determine the smallest
distance a if the bracket is not to move.
P4.5 A T-shaped bracket supports the four loads shown. Determine the
reactions at A and B if (a) a = 100 mm, (b) a = 70 mm.

SOLUTION

The amin value will be based on A = 0

From f.b.d. of bracket

M B = 0: ( 40 N )( 60 mm ) ( 30 N )( a ) (10 N )( a + 80 mm ) = 0
1600
a= = 40 mm
40

or amin = 40.0 mm
PROBLEM 4.7
A hand truck is used to move two barrels, each weighing 80 lb.
Neglecting the weight of the hand truck, determine (a) the vertical force
P which should be applied to the handle to maintain equilibrium when
= 35o , (b) the corresponding reaction at each of the two wheels.

SOLUTION

a1 = ( 20 in.) sin ( 8 in.) cos

a2 = ( 32 in.) cos ( 20 in.) sin

b = ( 64 in.) cos

From f.b.d. of hand truck


M B = 0: P ( b ) W ( a2 ) + W ( a1 ) = 0 (1)

Fy = 0: P 2w + 2 B = 0 (2)

For = 35
a1 = 20sin 35 8cos 35 = 4.9183 in.

a2 = 32cos 35 20sin 35 = 14.7413 in.

b = 64cos 35 = 52.426 in.


(a) From Equation (1)
P ( 52.426 in.) 80 lb (14.7413 in.) + 80 lb ( 4.9183 in.) = 0

P = 14.9896 lb or P = 14.99 lb
(b) From Equation (2)
14.9896 lb 2 ( 80 lb ) + 2 B = 0

B = 72.505 lb or B = 72.5 lb
PROBLEM 4.8
Solve Problem 4.7 when = 40o.
P4.7 A hand truck is used to move two barrels, each weighing 80 lb.
Neglecting the weight of the hand truck, determine (a) the vertical force P
which should be applied to the handle to maintain equilibrium when
= 35o , (b) the corresponding reaction at each of the two wheels.

SOLUTION

a1 = ( 20 in.) sin ( 8 in.) cos

a2 = ( 32 in.) cos ( 20 in.) sin

b = ( 64 in.) cos

From f.b.d. of hand truck


M B = 0: P ( b ) W ( a2 ) + W ( a1 ) = 0 (1)

Fy = 0: P 2w + 2B = 0 (2)

For = 40
a1 = 20sin 40 8cos 40 = 6.7274 in.

a2 = 32cos 40 20sin 40 = 11.6577 in.

b = 64cos 40 = 49.027 in.


(a) From Equation (1)
P ( 49.027 in.) 80 lb (11.6577 in.) + 80 lb ( 6.7274 in.) = 0

P = 8.0450 lb

or P = 8.05 lb
(b) From Equation (2)
8.0450 lb 2 ( 80 lb ) + 2B = 0

B = 75.9775 lb

or B = 76.0 lb
PROBLEM 4.9
Four boxes are placed on a uniform 14-kg wooden plank which rests
on two sawhorses. Knowing that the masses of boxes B and D are
4.5 kg and 45 kg, respectively, determine the range of values of the
mass of box A so that the plank remains in equilibrium when box C is
removed.

SOLUTION

WA = m A g WD = mD g = 45 g

WB = mB g = 4.5 g WG = mG g = 14 g

For ( m A )min, E = 0

M F = 0: ( mA g )( 2.5 m ) + ( 4.5g )(1.6 m )


+ (14 g )(1 m ) ( 45g )( 0.6 m ) = 0

m A = 2.32 kg

For ( m A )max, F = 0:

M E = 0: mA g ( 0.5 m ) ( 4.5g )( 0.4 m ) (14 g )(1 m )

( 45g )( 2.6 m ) = 0

mA = 265.6 kg

or 2.32 kg mA 266 kg
PROBLEM 4.10
A control rod is attached to a crank at A and cords are attached at B and
C. For the given force in the rod, determine the range of values of the
tension in the cord at C knowing that the cords must remain taut and that
the maximum allowed tension in a cord is 180 N.

SOLUTION

For (TC )max, TB = 0

M O = 0: (TC )max ( 0.120 m ) ( 400 N )( 0.060 m ) = 0

(TC )max = 200 N > Tmax = 180 N

(TC )max = 180.0 N

For (TC )min , TB = Tmax = 180 N

M O = 0: (TC )min ( 0.120 m ) + (180 N )( 0.040 m )


( 400 N )( 0.060 m ) = 0

(TC )min = 140.0 N

Therefore, 140.0 N TC 180.0 N


PROBLEM 4.11
The maximum allowable value of each of the reactions is 360 N.
Neglecting the weight of the beam, determine the range of values of the
distance d for which the beam is safe.

SOLUTION

From f.b.d. of beam

Fx = 0: Bx = 0 so that B = By

Fy = 0: A + B (100 + 200 + 300 ) N = 0

or A + B = 600 N
Therefore, if either A or B has a magnitude of the maximum of 360 N,
the other support reaction will be < 360 N ( 600 N 360 N = 240 N ) .

M A = 0: (100 N )( d ) ( 200 N )( 0.9 d ) ( 300 N )(1.8 d )


+ B (1.8 d ) = 0

720 1.8B
or d =
600 B
Since B 360 N,

720 1.8 ( 360 )


d = = 0.300 m or d 300 mm
600 360
M B = 0: (100 N )(1.8) A (1.8 d ) + ( 200 N )( 0.9 ) = 0
1.8 A 360
or d =
A
Since A 360 N,

1.8 ( 360 ) 360


d = = 0.800 m or d 800 mm
360
or 300 mm d 800 mm
PROBLEM 4.12
Solve Problem 4.11 assuming that the 100-N load is replaced by a 160-N
load.
P4.11 The maximum allowable value of each of the reactions is 360 N.
Neglecting the weight of the beam, determine the range of values of the
distance d for which the beam is safe.

SOLUTION

From f.b.d of beam

Fx = 0: Bx = 0 so that B = By

Fy = 0: A + B (160 + 200 + 300 ) N = 0

or A + B = 660 N
Therefore, if either A or B has a magnitude of the maximum of 360 N,
the other support reaction will be < 360 N ( 660 360 = 300 N ) .

M A = 0: 160 N ( d ) 200 N ( 0.9 d ) 300 N (1.8 d )

+ B (1.8 d ) = 0

720 1.8B
or d =
660 B
Since B 360 N,

720 1.8 ( 360 )


d = = 0.240 m or d 240 mm
660 360
M B = 0: 160 N (1.8 ) A (1.8 d ) + 200 N ( 0.9 ) = 0

1.8 A 468
or d =
A
Since A 360 N,

1.8 ( 360 ) 468


d = = 0.500 m or d 500 mm
360
or 240 mm d 500 mm
PROBLEM 4.13
For the beam of Sample Problem 4.2, determine the range of values of
P for which the beam will be safe knowing that the maximum
allowable value of each of the reactions is 45 kips and that the
reaction at A must be directed upward.

SOLUTION

For the force of P to be a minimum, A = 0.


With A = 0,

M B = 0: Pmin ( 6 ft ) ( 6 kips )( 2 ft ) ( 6 kips )( 4 ft ) = 0

Pmin = 6.00 kips

For the force P to be a maximum, A = A max = 45 kips


With A = 45 kips,

M B = 0: ( 45 kips )( 9 ft ) + Pmax ( 6 ft ) ( 6 kips )( 2 ft ) ( 6 kips )( 4 ft ) = 0

Pmax = 73.5 kips


A check must be made to verify the assumption that the maximum value of P is based on the reaction force at
A. This is done by making sure the corresponding value of B is < 45 kips.

Fy = 0: 45 kips 73.5 kips + B 6 kips 6 kips = 0

B = 40.5 kips < 45 kips ok or Pmax = 73.5 kips

and 6.00 kips P 73.5 kips


PROBLEM 4.14
For the beam and loading shown, determine the range of values of the
distance a for which the reaction at B does not exceed 50 lb
downward or 100 lb upward.

SOLUTION

To determine amax the two 150-lb forces need to be as close to B without


having the vertical upward force at B exceed 100 lb.
From f.b.d. of beam with B = 100 lb

M D = 0: (150 lb ) ( amax 4 in.) (150 lb ) ( amax 1 in.)

( 25 lb )( 2 in.) + (100 lb )( 8 in.) = 0

or amax = 5.00 in.

To determine amin the two 150-lb forces need to be as close to A without


having the vertical downward force at B exceed 50 lb.
From f.b.d. of beam with B = 50 lb

M D = 0: (150 lb )( 4 in. amin ) (150 lb )( amin 1 in.)

( 25 lb )( 2 in.) ( 50 lb )( 8 in.) = 0

or amin = 1.00 in.


Therefore, or 1.00 in. a 5.00 in.
PROBLEM 4.15
A follower ABCD is held against a circular cam by a stretched spring,
which exerts a force of 21 N for the position shown. Knowing that the
tension in rod BE is 14 N, determine (a) the force exerted on the roller
at A, (b) the reaction at bearing C.

SOLUTION

Note: From f.b.d. of ABCD


A
Ax = A cos 60 =
2

3
Ay = A sin 60 = A
2
(a) From f.b.d. of ABCD
A
M C = 0: ( 40 mm ) 21 N ( 40 mm )
2

+ 14 N ( 20 mm ) = 0

A = 28 N
or A = 28.0 N 60
(b) From f.b.d. of ABCD

Fx = 0: C x + 14 N + ( 28 N ) cos 60 = 0

C x = 28 N or C x = 28.0 N

Fy = 0: C y 21 N + ( 28 N ) sin 60 = 0

C y = 3.2487 N or C y = 3.25 N

Then C = C x2 + C y2 = ( 28)2 + ( 3.2487 )2 = 28.188 N

Cy 1 3.2487
and = tan 1 = tan = 6.6182
Cx 28

or C = 28.2 N 6.62
PROBLEM 4.16
A 6-m-long pole AB is placed in a hole and is guyed by three cables.
Knowing that the tensions in cables BD and BE are 442 N and 322 N,
respectively, determine (a) the tension in cable CD, (b) the reaction
at A.

SOLUTION

Note:

DB = ( 2.8)2 + ( 5.25)2 = 5.95 m

DC = ( 2.8)2 + ( 2.10 )2 = 3.50 m

(a) From f.b.d. of pole


2.8 m
M A = 0: ( 322 N )( 6 m ) + ( 442 N ) ( 6 m )
5.95 m

2.8 m
+ TCD ( 2.85 m ) = 0
3.50 m
TCD = 300 N

or TCD = 300 N
(b) From f.b.d. of pole
2.8 m
Fx = 0: 322 N ( 442 N )
5.95 m

2.8 m
( 300 N ) + Ax = 0
3.50 m
Ax = 126 N or A x = 126 N

5.25 m 2.10 m
Fy = 0: Ay ( 442 N ) ( 300 N ) = 0
5.95 m 3.50 m
Ay = 570 N or A y = 570 N

Then A= Ax2 + Ay2 = (126 )2 + ( 570 )2 = 583.76 N

570 N
and = tan 1 = 77.535
126 N
or A = 584 N 77.5
PROBLEM 4.17
Determine the reactions at A and C when (a) = 0, (b) = 30o.

SOLUTION

(a) (a) = 0
From f.b.d. of member ABC
M C = 0: (80 lb )(10 in.) + (80 lb )( 20 in.) A ( 40 in.) = 0
A = 60 lb

or A = 60.0 lb
Fy = 0: C y + 60 lb = 0

C y = 60 lb or C y = 60 lb

Fx = 0: 80 lb + 80 lb + Cx = 0

C x = 160 lb or C x = 160 lb

Then C = C x2 + C y2 = (160 )2 + ( 60 )2 = 170.880 lb

Cy 1 60
and = tan 1 = tan = 20.556
Cx 160

or C = 170.9 lb 20.6
(b) (b) = 30
From f.b.d. of member ABC
M C = 0: (80 lb )(10 in.) + ( 80 lb )( 20 in.) ( A cos 30 )( 40 in.)
+ ( A sin 30 )( 20 in.) = 0

A = 97.399 lb
or A = 97.4 lb 60
PROBLEM 4.17 CONTINUED
Fx = 0: 80 lb + 80 lb + ( 97.399 lb ) sin 30 + Cx = 0

C x = 208.70 lb or C x = 209 lb

Fy = 0: C y + ( 97.399 lb ) cos 30 = 0

C y = 84.350 lb or C y = 84.4 lb

Then C = C x2 + C y2 = ( 208.70 )2 + (84.350 )2 = 225.10 lb

Cy 1 84.350
and = tan 1 = tan = 22.007
x
C 208.70

or C = 225 lb 22.0
PROBLEM 4.18
Determine the reactions at A and B when (a) h = 0, (b) h = 8 in.

SOLUTION

(a) (a) h = 0
From f.b.d. of plate
M A = 0: ( B sin 30 )( 20 in.) ( 40 lb )(10 in.) = 0
B = 40 lb
or B = 40.0 lb 30

Fx = 0: Ax ( 40 lb ) cos 30 = 0

Ax = 34.641 lb or A x = 34.6 lb

Fy = 0: Ay 40 lb + ( 40 lb ) sin 30 = 0

Ay = 20 lb or A y = 20.0 lb

Then A= Ax2 + Ay2 = ( 34.641)2 + ( 20 )2 = 39.999 lb

Ay 1 20
and = tan 1 = tan = 30.001
Ax 34.641

or A = 40.0 lb 30
(b) (b) h = 8 in.
From f.b.d. of plate
M A = 0: ( B sin 30 )( 20 in.) ( B cos 30 )(8 in.)
( 40 lb )(10 in.) = 0

B = 130.217 lb
or B = 130.2 lb 30.0
PROBLEM 4.18 CONTINUED
Fx = 0: Ax (130.217 lb ) cos30 = 0

Ax = 112.771 lb or A x = 112.8 lb

Fy = 0: Ay 40 lb + (130.217 lb ) sin 30 = 0

Ay = 25.108 lb or A y = 25.1 lb

Then A= Ax2 + Ay2 = (112.771)2 + ( 25.108)2 = 115.532 lb

Ay 1 25.108
and = tan 1 = tan = 12.5519
x
A 112.771

or A = 115.5 lb 12.55
PROBLEM 4.19
The lever BCD is hinged at C and is attached to a control rod at B. If
P = 200 N, determine (a) the tension in rod AB, (b) the reaction at C.

SOLUTION

(a) From f.b.d. of lever BCD


M C = 0: TAB ( 50 mm ) 200 N ( 75 mm ) = 0

TAB = 300 N
(b) From f.b.d. of lever BCD

Fx = 0: 200 N + Cx + 0.6 ( 300 N ) = 0

C x = 380 N or C x = 380 N

Fy = 0: C y + 0.8 ( 300 N ) = 0

C y = 240 N or C y = 240 N

Then C = C x2 + C y2 = ( 380 )2 + ( 240 )2 = 449.44 N

Cy 1 240
and = tan 1 = tan = 32.276
Cx 380

or C = 449 N 32.3
PROBLEM 4.20
The lever BCD is hinged at C and is attached to a control rod at B.
Determine the maximum force P which can be safely applied at D if the
maximum allowable value of the reaction at C is 500 N.

SOLUTION

From f.b.d. of lever BCD


M C = 0: TAB ( 50 mm ) P ( 75 mm ) = 0

TAB = 1.5P (1)

Fx = 0: 0.6TAB + P C x = 0

C x = P + 0.6TAB (2)

From Equation (1) Cx = P + 0.6 (1.5P ) = 1.9 P

Fy = 0: 0.8TAB C y = 0

C y = 0.8TAB (3)

From Equation (1) C y = 0.8 (1.5P ) = 1.2P

From Equations (2) and (3)

C = C x2 + C y2 = (1.9P )2 + (1.2P )2 = 2.2472 P

Since Cmax = 500 N,

500 N = 2.2472Pmax

or Pmax = 222.49 lb
or P = 222 lb
PROBLEM 4.21
The required tension in cable AB is 800 N. Determine (a) the vertical
force P which must be applied to the pedal, (b) the corresponding
reaction at C.

SOLUTION

(a) From f.b.d. of pedal

M C = 0: P ( 0.4 m ) ( 800 N ) ( 0.18 m ) sin 60 = 0

P = 311.77 N

or P = 312 N

(b) From f.b.d. of pedal

Fx = 0: Cx 800 N = 0

Cx = 800 N

or C x = 800 N

Fy = 0: C y 311.77 N = 0

C y = 311.77 N

or C y = 311.77 N

Then C = C x2 + C y2 = (800 )2 + ( 311.77 )2 = 858.60 N

Cy 1 311.77
and = tan 1 = tan = 21.291
x
C 800

or C = 859 N 21.3
PROBLEM 4.22
Determine the maximum tension which can be developed in cable AB
if the maximum allowable value of the reaction at C is 1000 N.

SOLUTION

Have Cmax = 1000 N

Now C 2 = C x2 + C y2

Cy = (1000 )2 Cx2 (1)

From f.b.d. of pedal

Fx = 0: C x Tmax = 0

C x = Tmax (2)

M D = 0: C y ( 0.4 m ) Tmax ( 0.18 m ) sin 60 = 0

C y = 0.38971Tmax (3)

Equating the expressions for C y in Equations (1) and (3), with Cx = Tmax
from Equation (2)

(1000 )2 Tmax
2
= 0.389711Tmax

Tmax
2
= 868,150

and Tmax = 931.75 N

or Tmax = 932 N
PROBLEM 4.23
A steel rod is bent to form a mounting bracket. For each of the mounting
brackets and loadings shown, determine the reactions at A and B.

SOLUTION
(a) From f.b.d. of mounting bracket
(a) M E = 0: A ( 8 in.) 80 lb in. (10 lb )( 6 in.)

( 20 lb )(12 in.) = 0

A = 47.5 lb
or A = 47.5 lb

Fx = 0: Bx 10 lb + 47.5 lb = 0

Bx = 37.5 lb

or B x = 37.5 lb

Fy = 0: By 20 lb = 0

By = 20 lb

or B y = 20.0 lb

Then B= Bx2 + By2 = ( 37.5)2 + ( 20.0 )2 = 42.5 lb

By 1 20
and = tan 1 = tan = 28.072
Bx 37.5

or B = 42.5 lb 28.1
(b) From f.b.d. of mounting bracket
(b)
M B = 0: ( A cos 45 )(8 in.) 80 lb in.
(10 lb )( 6 in.) ( 20 lb )(12 in.) = 0

A = 67.175 lb
or A = 67.2 lb 45

Fx = 0: Bx 10 lb + 67.175cos 45 = 0

Bx = 37.500 lb

or B x = 37.5 lb
PROBLEM 4.23 CONTINUED

Fy = 0: By 20 lb + 67.175sin 45 = 0

By = 27.500 lb

or B y = 27.5 lb

Then B= Bx2 + By2 = ( 37.5)2 + ( 27.5)2 = 46.503 lb

By 1 27.5
and = tan 1 = tan = 36.254
Bx 37.5

or B = 46.5 lb 36.3
PROBLEM 4.24
A steel rod is bent to form a mounting bracket. For each of the
mounting brackets and loadings shown, determine the reactions at A
and B.

SOLUTION

(a) (a) From f.b.d. of mounting bracket


M A = 0: B ( 8 in.) ( 20 lb )(12 in.)

+ (10 lb )( 2 in.) 80 lb in. = 0

B = 37.5 lb
or B = 37.5 lb

Fx = 0: 37.5 lb 10 lb + Ax = 0

Ax = 47.5 lb

or A x = 47.5 lb

Fy = 0: 20 lb + Ay = 0

Ay = 20 lb

or A y = 20.0 lb

Then A= Ax2 + Ay2 = ( 47.5)2 + ( 20 )2 = 51.539 lb

Ay 1 20
and = tan 1 = tan = 22.834
Ax 47.5

or A = 51.5 lb 22.8
(b) (b) From f.b.d. of mounting bracket
M A = 0: ( B cos 45 )( 8 in.) ( 20 lb )( 2 in.)

80 lb in. + (10 lb )( 2 in.) = 0

B = 53.033 lb
or B = 53.0 lb 45

Fx = 0: Ax + ( 53.033 lb ) cos 45 10 = 0

Ax = 47.500 lb

or A x = 47.5 lb
PROBLEM 4.24 CONTINUED

Fy = 0: Ay ( 53.033 lb ) sin 45 20 = 0

Ay = 17.500 lb

or A y = 17.50 lb

Then A= Ax2 + Ay2 = ( 47.5)2 + (17.5)2 = 50.621 lb

Ay 1 17.5
and = tan 1 = tan = 20.225
Ax 47.5

or A = 50.6 lb 20.2
PROBLEM 4.25
A sign is hung by two chains from mast AB. The mast is hinged at A and
is supported by cable BC. Knowing that the tensions in chains DE and
FH are 50 lb and 30 lb, respectively, and that d = 1.3 ft, determine
(a) the tension in cable BC, (b) the reaction at A.

SOLUTION

First note BC = (8.4 )2 + (1.3)2 = 8.5 ft

(a) From f.b.d. of mast AB

8.4
M A = 0: TBC ( 2.5 ft ) ( 30 lb )( 7.2 ft )
8.5

50 lb ( 2.2 ft ) = 0

TBC = 131.952 lb

or TBC = 132.0 lb

(b) From f.b.d. of mast AB

8.4
Fx = 0: Ax (131.952 lb ) = 0
8.5

Ax = 130.400 lb

or A x = 130.4 lb

1.3
Fy = 0: Ay + (131.952 lb ) 30 lb 50 lb = 0
8.5

Ay = 59.819 lb

or A y = 59.819 lb

Then A= Ax2 + Ay2 = (130.4 )2 + ( 59.819 )2 = 143.466 lb

Ay 1 59.819
and = tan 1 = tan = 24.643
x
A 130.4

or A = 143.5 lb 24.6
PROBLEM 4.26
A sign is hung by two chains from mast AB. The mast is hinged at A
and is supported by cable BC. Knowing that the tensions in chains DE
and FH are 30 lb and 20 lb, respectively, and that d = 1.54 ft,
determine (a) the tension in cable BC, (b) the reaction at A.

SOLUTION

First note BC = (8.4 )2 + (1.54 )2 = 8.54 ft

(a) From f.b.d. of mast AB


8.4
M A = 0: TBC ( 2.5 ft ) 20 lb ( 7.2 ft )
8.54

30 lb ( 2.2 ft ) = 0

TBC = 85.401 lb

or TBC = 85.4 lb
(b) From f.b.d. of mast AB
8.4
Fx = 0: Ax ( 85.401 lb ) = 0
8.54
Ax = 84.001 lb

or A x = 84.001 lb

1.54
Fy = 0: Ay + ( 85.401 lb ) 20 lb 30 lb = 0
8.54
Ay = 34.600 lb

or A y = 34.600 lb

Then A= Ax2 + Ay2 = (84.001)2 + ( 34.600 )2 = 90.848 lb

Ay 1 34.6
and = tan 1 = tan = 22.387
Ax 84.001

or A = 90.8 lb 22.4
PROBLEM 4.27
For the frame and loading shown, determine the reactions at A and E
when (a) = 30o , (b) = 45o.

SOLUTION

(a) Given = 30
(a)
From f.b.d. of frame

M A = 0: ( 90 N )( 0.2 m ) ( 90 N )( 0.06 m )

+ ( E cos 60 )( 0.160 m ) + ( E sin 60 )( 0.100 m ) = 0

E = 140.454 N

or E = 140.5 N 60

Fx = 0: Ax 90 N + (140.454 N ) cos 60 = 0

Ax = 19.7730 N

or A x = 19.7730 N

Fy = 0: Ay 90 N + (140.454 N ) sin 60 = 0

Ay = 31.637 N

or A y = 31.6 N

Then A= Ax2 + Ay2 = (19.7730 )2 + ( 31.637 )2


= 37.308 lb

Ay 1 31.637
and = tan 1 = tan
Ax 19.7730

= 57.995

or A = 37.3 N 58.0
PROBLEM 4.27 CONTINUED

(b) (b) Given = 45


From f.b.d. of frame
M A = 0: ( 90 N )( 0.2 m ) ( 90 N )( 0.06 m )

+ ( E cos 45 )( 0.160 m ) + ( E sin 45 )( 0.100 m ) = 0

E = 127.279 N
or E = 127.3 N 45

Fx = 0: Ax 90 + (127.279 N ) cos 45 = 0

Ax = 0

Fy = 0: Ay 90 + (127.279 N ) sin 45 = 0

Ay = 0

or A = 0
PROBLEM 4.28
A lever AB is hinged at C and is attached to a control cable at A. If the
lever is subjected to a 300-N vertical force at B, determine
(a) the tension in the cable, (b) the reaction at C.

SOLUTION
First
x AC = ( 0.200 m ) cos 20 = 0.187 939 m

y AC = ( 0.200 m ) sin 20 = 0.068 404 m

Then
yDA = 0.240 m y AC
= 0.240 m 0.068404 m
= 0.171596 m
yDA 0.171 596
and tan = =
x AC 0.187 939

= 42.397
and = 90 20 42.397 = 27.603
(a) From f.b.d. of lever AB
M C = 0: T cos 27.603 ( 0.2 m )

300 N ( 0.3 m ) cos 20 = 0

T = 477.17 N or T = 477 N
(b) From f.b.d. of lever AB

Fx = 0: C x + ( 477.17 N ) cos 42.397 = 0

C x = 352.39 N

or C x = 352.39 N

Fy = 0: C y 300 N ( 477.17 N ) sin 42.397 = 0

C y = 621.74 N

or C y = 621.74 N
PROBLEM 4.28 CONTINUED

Then C = C x2 + C y2 = ( 352.39 )2 + ( 621.74 )2 = 714.66 N

Cy 1 621.74
and = tan 1 = tan = 60.456
x
C 352.39

or C = 715 N 60.5
PROBLEM 4.29
Neglecting friction and the radius of the pulley, determine the tension
in cable BCD and the reaction at support A when d = 80 mm.

SOLUTION

First
60
= tan 1 = 12.0948
280

60
= tan 1 = 36.870
80
From f.b.d. of object BAD
M A = 0: ( 40 N )( 0.18 m ) + (T cos )( 0.08 m )
+ (T sin )( 0.18 m ) (T cos )( 0.08 m )

(T sin )( 0.18 m ) = 0

7.2 N m
T = = 128.433 N
0.056061
or T = 128.4 N

Fx = 0: (128.433 N )( cos cos ) + Ax = 0

Ax = 22.836 N

or A x = 22.836 N

Fy = 0: Ay + (128.433 N )( sin + sin ) + 40 N = 0

Ay = 143.970 N

or A y = 143.970 N

Then A= Ax2 + Ay2 = ( 22.836 )2 + (143.970 )2 = 145.770 N

Ay 1 143.970
and = tan 1 = tan = 80.987
Ax 22.836

or A = 145.8 N 81.0
PROBLEM 4.30
Neglecting friction and the radius of the pulley, determine the tension in
cable BCD and the reaction at support A when d = 144 mm.

SOLUTION

First note
60
= tan 1 = 15.5241
216

60
= tan 1 = 22.620
144
From f.b.d. of member BAD
M A = 0: ( 40 N )( 0.18 m ) + (T cos )( 0.08 m )
+ (T sin )( 0.18 m ) (T cos )( 0.08 m )

(T sin )( 0.18 m ) = 0

7.2 N m
T = = 404.04 N
0.0178199 m
or T = 404 N

Fx = 0: Ax + ( 404.04 N )( cos cos ) = 0

Ax = 16.3402 N

or A x = 16.3402 N

Fy = 0: Ay + ( 404.04 N )( sin + sin ) + 40 N = 0

Ay = 303.54 N

or A y = 303.54 N

Then A= Ax2 + Ay2 = (16.3402 )2 + ( 303.54 )2 = 303.98 N

Ay 1 303.54
and = tan 1 = tan = 86.919
x
A 16.3402

or A = 304 N 86.9
PROBLEM 4.31
Neglecting friction, determine the tension in cable ABD and the reaction
at support C.

SOLUTION

From f.b.d. of inverted T-member

M C = 0: T ( 25 in.) T (10 in.) ( 30 lb )(10 in.) = 0

T = 20 lb

or T = 20.0 lb W

Fx = 0: Cx 20 lb = 0

C x = 20 lb

or C x = 20.0 lb

Fy = 0: C y + 20 lb 30 lb = 0

C y = 10 lb

or C y = 10.00 lb

Then C = C x2 + C y2 = ( 20 )2 + (10 )2 = 22.361 lb

Cy 1 10
and = tan 1 = tan = 26.565
Cx 20

or C = 22.4 lb 26.6 W
PROBLEM 4.32
Rod ABC is bent in the shape of a circular arc of radius R. Knowing
that = 35o , determine the reaction (a) at B, (b) at C.

SOLUTION

For = 35

(a) From the f.b.d. of rod ABC

M D = 0: Cx( R ) P( R ) = 0

Cx = P

or Cx = P

Fx = 0: P B sin 35 = 0

P
B= = 1.74345P
sin 35

or B = 1.743P 55.0 W

(b) From the f.b.d. of rod ABC

Fy = 0: C y + (1.74345P ) cos 35 P = 0

C y = 0.42815P

or C y = 0.42815P

Then C = C x2 + C y2 = ( P )2 + ( 0.42815P )2 = 1.08780 P

Cy 1 0.42815P
and = tan 1 = tan = 23.178
Cx P

or C = 1.088P 23.2 W
PROBLEM 4.33
Rod ABC is bent in the shape of a circular arc of radius R. Knowing
that = 50o , determine the reaction (a) at B, (b) at C.

SOLUTION

For = 50

(a) From the f.b.d. of rod ABC

M D = 0: Cx ( R ) P ( R ) = 0

Cx = P

or Cx = P

Fx = 0: P B sin 50 = 0

P
B= = 1.30541P
sin 50

or B = 1.305P 40.0 W

(b) From the f.b.d. of rod ABC

Fy = 0: C y P + (1.30541P ) cos 50 = 0

C y = 0.160900P

or C y = 0.1609 P

Then C = C x2 + C y2 = ( P )2 + ( 0.1609P )2 = 1.01286 P

Cy 1 0.1609 P
and = tan 1 = tan = 9.1405
x
C P

or C = 1.013P 9.14 W
PROBLEM 4.34
Neglecting friction and the radius of the pulley, determine (a) the
tension in cable ABD, (b) the reaction at C.

SOLUTION

First note
15
= tan 1 = 22.620
36

15
= tan 1 = 36.870
20
(a) From f.b.d. of member ABC
M C = 0: ( 30 lb )( 28 in.) (T sin 22.620 )( 36 in.)
(T sin 36.870 )( 20 in.) = 0

T = 32.500 lb
or T = 32.5 lb W
(b) From f.b.d. of member ABC

Fx = 0: Cx + ( 32.500 lb )( cos 22.620 + cos 36.870 ) = 0

C x = 56.000 lb

or C x = 56.000 lb

Fy = 0: C y 30 lb + ( 32.500 lb )( sin 22.620 + sin 36.870 ) = 0

C y = 2.0001 lb

or C y = 2.0001 lb

Then C = C x2 + C y2 = ( 56.0 )2 + ( 2.001)2 = 56.036 lb

Cy 1 2.0
and = tan 1 = tan = 2.0454
Cx 56.0

or C = 56.0 lb 2.05 W
PROBLEM 4.35
Neglecting friction, determine the tension in cable ABD and the
reaction at C when = 60o.

SOLUTION

From f.b.d. of bent ACD

M C = 0: (T cos30 )( 2a sin 60 ) + (T sin 30 )( a + 2a cos 60 )


T (a) P (a) = 0

P
T =
1.5

2P
or T = W
3

2P
Fx = 0: C x cos 30 = 0
3

3
Cx = P = 0.57735P
3

or C x = 0.577 P

2 2P
Fy = 0: C y + PP+ cos 60 = 0
3 3

Cy = 0

or C = 0.577P W
PROBLEM 4.36
Neglecting friction, determine the tension in cable ABD and the
reaction at C when = 30o.

SOLUTION

From f.b.d. of bent ACD

M C = 0: (T cos 60 )( 2a sin 30 ) + T sin 60 ( a + 2a cos 30 )


P (a) T (a) = 0

P
T = = 0.53590P
1.86603

or T = 0.536 P W

Fx = 0: C x ( 0.53590P ) cos 60 = 0

Cx = 0.26795P

or C x = 0.268P

Fy = 0: C y + 0.53590 P P + ( 0.53590 P ) sin 60 = 0

Cy = 0

or C = 0.268P W
PROBLEM 4.37
Determine the tension in each cable and the reaction at D.

SOLUTION

First note

BE = ( 20 )2 + (8)2 in. = 21.541 in.

CF = (10 )2 + (8)2 in. = 12.8062 in.

From f.b.d. of member ABCD



(120 lb )( 20 in.)
8
M C = 0: TBE (10 in.) = 0
21.541
TBE = 646.24 lb

or TBE = 646 lb W

8 8
Fy = 0: 120 lb + ( 646.24 lb ) TCF = 0
21.541 12.8062
TCF = 192.099 lb

or TCF = 192.1 lb W

20 10
Fx = 0: ( 646.24 lb ) + (192.099 lb ) D = 0
21.541 12.8062
D = 750.01 lb
or D = 750 lb W
PROBLEM 4.38
Rod ABCD is bent in the shape of a circular arc of radius 80 mm and
rests against frictionless surfaces at A and D. Knowing that the collar
at B can move freely on the rod and that = 45o. determine (a) the
tension in cord OB, (b) the reactions at A and D.

SOLUTION

(a) From f.b.d. of rod ABCD

M E = 0: ( 25 N ) cos 60 ( dOE ) (T cos 45 ) ( dOE ) = 0


T = 17.6777 N

or T = 17.68 N W

(b) From f.b.d. of rod ABCD

Fx = 0: (17.6777 N ) cos 45 + ( 25 N ) cos 60

+ N D cos 45 N A cos 45 = 0

N A ND = 0

or ND = N A (1)

Fy = 0: N A sin 45 + N D sin 45 (17.6777 N ) sin 45

( 25 N ) sin 60 = 0

N A + N D = 48.296 N (2)

Substituting Equation (1) into Equation (2),

2 N A = 48.296 N

N A = 24.148 N

or N A = 24.1 N 45.0 W

and N D = 24.1 N 45.0 W


PROBLEM 4.39
Rod ABCD is bent in the shape of a circular arc of radius 80 mm and
rests against frictionless surfaces at A and D. Knowing that the collar
at B can move freely on the rod, determine (a) the value of for
which the tension in cord OB is as small as possible, (b) the
corresponding value of the tension, (c) the reactions at A and D.

SOLUTION

(a) From f.b.d. of rod ABCD


M E = 0: ( 25 N ) cos 60 ( dOE ) (T cos ) ( dOE ) = 0
12.5 N
or T = (1)
cos
T is minimum when cos is maximum,
or = 0 W
(b) From Equation (1)
12.5 N
T = = 12.5 N
cos 0
or Tmin = 12.50 N W

(c) Fx = 0: N A cos 45 + N D cos 45 + 12.5 N

( 25 N ) cos 60 = 0

ND N A = 0

or ND = N A (2)

Fy = 0: N A sin 45 + N D sin 45 ( 25 N ) sin 60 = 0

N D + N A = 30.619 N (3)
Substituting Equation (2) into Equation (3),
2 N A = 30.619

N A = 15.3095 N

or N A = 15.31 N 45.0 W

and N D = 15.31 N 45.0 W


PROBLEM 4.40
Bar AC supports two 100-lb loads as shown. Rollers A and C rest against
frictionless surfaces and a cable BD is attached at B. Determine (a) the
tension in cable BD, (b) the reaction at A, (c) the reaction at C.

SOLUTION

First note that from similar triangles

yDB 10
= yDB = 3 in.
6 20

and BD = ( 3)2 + (14 )2 in. = 14.3178 in.

14
Tx = T = 0.97780T
14.3178
3
Ty = T = 0.20953T
14.3178
(a) From f.b.d. of bar AC
M E = 0: ( 0.97780T )( 7 in.) ( 0.20953T )( 6 in.)
(100 lb )(16 in.) (100 lb )( 4 in.) = 0

T = 357.95 lb
or T = 358 lb W
(b) From f.b.d. of bar AC
Fy = 0: A 100 0.20953 ( 357.95 ) 100 = 0

A = 275.00 lb
or A = 275 lb W
(c) From f.b.d of bar AC

Fx = 0: 0.97780 ( 357.95 ) C = 0

C = 350.00 lb
or C = 350 lb W
PROBLEM 4.41
A parabolic slot has been cut in plate AD, and the plate has been
placed so that the slot fits two fixed, frictionless pins B and C. The
equation of the slot is y = x 2 /100, where x and y are expressed in mm.
Knowing that the input force P = 4 N, determine (a) the force each
pin exerts on the plate, (b) the output force Q.

SOLUTION

x2
The equation of the slot is y =
100
dy
Now = slope of the slot at C
dx C

2x
= = 1.200
100 x = 60 mm

= tan 1 (1.200 ) = 50.194

and = 90 = 90 50.194 = 39.806


Coordinates of C are

xC = 60 mm, yC =
( 60 ) 2 = 36 mm
100
Also, the coordinates of D are xD = 60 mm

yD = 46 mm + ( 40 mm ) sin

120 66
where = tan 1 = 12.6804
240

yD = 46 mm + ( 40 mm ) tan12.6804

= 55.000 mm
PROBLEM 4.41 CONTINUED

60 mm 60 mm
Also, yED = =
tan tan12.6804
= 266.67 mm
From f.b.d. of plate AD

M E = 0: ( NC cos ) yED ( yD yC ) + ( NC sin )( xC ) ( 4 N )( yED yD ) = 0

( NC cos 39.806 ) 266.67 ( 55.0 36.0 ) mm + NC sin ( 39.806 )( 60 mm ) ( 4 N )( 266.67 55.0 ) mm = 0


NC = 3.7025 N

or NC = 3.70 N 39.8

Fx = 0: 4 N + NC cos + Q sin = 0

4 N + ( 3.7025 N ) cos 39.806 + Q sin12.6804 = 0

Q = 5.2649 N
or Q = 5.26 N 77.3
Fy = 0: N B + NC sin Q cos = 0

N B + ( 3.7025 N ) sin 39.806 ( 5.2649 N ) cos12.6804 = 0

N B = 2.7662 N

or N B = 2.77 N

(a) N B = 2.77 N , NC = 3.70 N 39.8

(b) Q = 5.26 N 77.3 ( output )


PROBLEM 4.42
A parabolic slot has been cut in plate AD, and the plate has been placed
so that the slot fits two fixed, frictionless pins B and C. The equation of
the slot is y = x 2 /100, where x and y are expressed in mm. Knowing that
the maximum allowable force exerted on the roller at D is 8.5 N,
determine (a) the corresponding magnitude of the input force P, (b) the
force each pin exerts on the plate.

SOLUTION
x2
The equation of the slot is, y =
100
dy
Now = slope of slot at C
dx C

2x
= = 1.200
100 x = 60 mm

= tan 1 (1.200 ) = 50.194

and = 90 = 90 50.194 = 39.806


Coordinates of C are

xC = 60 mm, yC =
( 60 )2 = 36 mm
100
Also, the coordinates of D are
xD = 60 mm

yD = 46 mm + ( 40 mm ) sin

120 66
where = tan 1 = 12.6804
240

yD = 46 mm + ( 40 mm ) tan12.6804 = 55.000 mm

Note: xE = 0

yE = yC + ( 60 mm ) tan

= 36 mm + ( 60 mm ) tan 39.806

= 86.001 mm
(a) From f.b.d. of plate AD

M E = 0: P ( yE ) ( 8.5 N ) sin ( yE yD )

( 8.5 N ) cos ( 60 mm ) = 0
PROBLEM 4.42 CONITNIUED

P ( 86.001 mm ) ( 8.5 N ) sin12.6804 ( 31.001 mm )

( 8.5 N ) cos12.6804 ( 60 mm ) = 0

P = 6.4581 N

or P = 6.46 N

(b) Fx = 0: P ( 8.5 N ) sin NC cos = 0

6.458 N ( 8.5 N )( sin12.6804 ) NC ( cos 39.806 ) = 0

NC = 5.9778 N

or NC = 5.98 N 39.8

Fy = 0: N B + NC sin ( 8.5 N ) cos = 0

N B + ( 5.9778 N ) sin 39.806 ( 8.5 N ) cos12.6804 = 0

N B = 4.4657 N

or N B = 4.47 N
PROBLEM 4.43
A movable bracket is held at rest by a cable attached at E and by frictionless
rollers. Knowing that the width of post FG is slightly less than the distance
between the rollers, determine the force exerted on the post by each roller
when = 20o.

SOLUTION

From f.b.d. of bracket


Fy = 0: T sin 20 60 lb = 0

T = 175.428 lb
Tx = (175.428 lb ) cos 20 = 164.849 lb

Ty = (175.428 lb ) sin 20 = 60 lb

Note: Ty and 60 lb force form a couple of

60 lb (10 in.) = 600 lb in.

M B = 0: 164.849 lb ( 5 in.) 600 lb in. + FCD ( 8 in.) = 0

FCD = 28.030 lb

or FCD = 28.0 lb

Fx = 0: FCD + FAB Tx = 0

28.030 lb + FAB 164.849 lb = 0

FAB = 192.879 lb

or FAB = 192.9 lb
Rollers A and C can only apply a horizontal force to the right onto the
vertical post corresponding to the equal and opposite force to the left on
the bracket. Since FAB is directed to the right onto the bracket, roller B
will react FAB. Also, since FCD is acting to the left on the bracket, it will
act to the right on the post at roller C.
PROBLEM 4.43 CONTINUED

A=D=0
B = 192.9 lb
C = 28.0 lb
Forces exerted on the post are
A=D=0
B = 192.9 lb
C = 28.0 lb
PROBLEM 4.44
Solve Problem 4.43 when = 30o.
P4.43 A movable bracket is held at rest by a cable attached at E and by
frictionless rollers. Knowing that the width of post FG is slightly less
than the distance between the rollers, determine the force exerted on the
post by each roller when = 20o.

SOLUTION
From f.b.d. of bracket
Fy = 0: T sin 30 60 lb = 0

T = 120 lb

Tx = (120 lb ) cos 30 = 103.923 lb

Ty = (120 lb ) sin 30 = 60 lb

Note: Ty and 60 lb force form a couple of

( 60 lb )(10 in.) = 600 lb in.


M B = 0: (103.923 lb )( 5 in.) 600 lb in. + FCD (8 in.) = 0
FCD = 10.0481 lb

or FCD = 10.05 lb

Fx = 0: FCD + FAB Tx = 0

10.0481 lb + FAB 103.923 lb = 0

FAB = 93.875 lb

or FAB = 93.9 lb
Rollers A and C can only apply a horizontal force to the right on the
vertical post corresponding to the equal and opposite force to the left on
the bracket. The opposite direction apply to roller B and D. Since both
FAB and FCD act to the right on the bracket, rollers B and D will react
these forces.
A=C=0
B = 93.9 lb
D = 10.05 lb
Forces exerted on the post are
A=C=0
B = 93.9 lb
D = 10.05 lb
PROBLEM 4.45
A 20-lb weight can be supported in the three different ways shown.
Knowing that the pulleys have a 4-in. radius, determine the reaction at A
in each case.

SOLUTION

(a) From f.b.d. of AB

Fx = 0: Ax = 0

Fy = 0: Ay 20 lb = 0

or Ay = 20.0 lb

and A = 20.0 lb

M A = 0: M A ( 20 lb )(1.5 ft ) = 0

M A = 30.0 lb ft

or M A = 30.0 lb ft

1 ft
(b) Note: 4 in. = 0.33333 ft
12 in.
From f.b.d. of AB

Fx = 0: Ax 20 lb = 0

or Ax = 20.0 lb

Fy = 0: Ay 20 lb = 0

or Ay = 20.0 lb

Then A= Ax2 + Ay2 = ( 20.0 )2 + ( 20.0 )2 = 28.284 lb

A = 28.3 lb 45

M A = 0: M A + ( 20 lb )( 0.33333 ft )

( 20 lb )(1.5 ft + 0.33333 ft ) = 0

M A = 30.0 lb ft

or M A = 30.0 lb ft
PROBLEM 4.45 CONTINUED

(c) From f.b.d. of AB

Fx = 0: Ax = 0

Fy = 0: Ay 20 lb 20 lb = 0

or Ay = 40.0 lb

and A = 40.0 lb

M A = 0: M A ( 20 lb )(1.5 ft 0.33333 ft )

( 20 lb )(1.5 ft + 0.33333 ft ) = 0

M A = 60.0 lb ft

or M A = 60.0 lb ft
PROBLEM 4.46
A belt passes over two 50-mm-diameter pulleys which are mounted on a
bracket as shown. Knowing that M = 0 and Ti = TO = 24 N, determine
the reaction at C.

SOLUTION

From f.b.d. of bracket

Fx = 0: Cx 24 N = 0

Cx = 24 N

Fy = 0: C y 24 N = 0

C y = 24 N

Then C = C x2 + C y2 = ( 24 )2 + ( 24 )2 = 33.941 N

C = 33.9 N 45.0

M C = 0: M C ( 24 N ) ( 45 25 ) mm

+ ( 24 N ) ( 25 + 50 60 ) mm = 0

M C = 120 N mm

or M C = 0.120 N m
PROBLEM 4.47
A belt passes over two 50-mm-diameter pulleys which are mounted on a
bracket as shown. Knowing that M = 0.40 N m m and that Ti and TO
are equal to 32 N and 16 N, respectively, determine the reaction at C.

SOLUTION

From f.b.d. of bracket

Fx = 0: C x 32 N = 0

C x = 32 N

Fy = 0: C y 16 N = 0

C y = 16 N

Then C = C x2 + C y2 = ( 32 )2 + (16 )2 = 35.777 N

Cy 1 16
and = tan 1 = tan = 26.565
Cx 32

or C = 35.8 N 26.6

M C = 0: M C ( 32 N )( 45 mm 25 mm )

+ (16 N )( 25 mm + 50 mm 60 mm ) 400 N mm = 0

M C = 800 N mm

or M C = 0.800 N m
PROBLEM 4.48
A 350-lb utility pole is used to support at C the end of an electric wire.
The tension in the wire is 120 lb, and the wire forms an angle of 15
with the horizontal at C. Determine the largest and smallest allowable
tensions in the guy cable BD if the magnitude of the couple at A may not
exceed 200 lb ft.

SOLUTION

First note

LBD = ( 4.5)2 + (10 )2 = 10.9659 ft

Tmax : From f.b.d. of utility pole with M A = 200 lb ft

M A = 0: 200 lb ft (120 lb ) cos15 (14 ft )

4.5
+ Tmax (10 ft ) = 0
10.9659
Tmax = 444.19 lb

or Tmax = 444 lb

Tmin : From f.b.d. of utility pole with M A = 200 lb ft

M A = 0: 200 lb ft (120 lb ) cos15 (14 ft )

4.5
+ Tmin (10 ft ) = 0
10.9659
Tmin = 346.71 lb

or Tmin = 347 lb
PROBLEM 4.49
In a laboratory experiment, students hang the masses shown from a beam
of negligible mass. (a) Determine the reaction at the fixed support A
knowing that end D of the beam does not touch support E. (b) Determine
the reaction at the fixed support A knowing that the adjustable support E
exerts an upward force of 6 N on the beam.

SOLUTION

( )
WB = mB g = (1 kg ) 9.81 m/s 2 = 9.81 N

( )
WC = mC g = ( 0.5 kg ) 9.81 m/s 2 = 4.905 N

(a) From f.b.d. of beam ABCD

Fx = 0: Ax = 0

Fy = 0: Ay WB WC = 0

Ay 9.81 N 4.905 N = 0

Ay = 14.715 N

or A = 14.72 N

M A = 0: M A WB ( 0.2 m ) WC ( 0.3 m ) = 0

M A ( 9.81 N )( 0.2 m ) ( 4.905 N )( 0.3 m ) = 0

M A = 3.4335 N m

or M A = 3.43 N m
(b) From f.b.d. of beam ABCD

Fx = 0: Ax = 0

Fy = 0: Ay WB WC + 6 N = 0

Ay 9.81 N 4.905 N + 6 N = 0

Ay = 8.715 N or A = 8.72 N

M A = 0: M A WB ( 0.2 m ) WC ( 0.3 m ) + ( 6 N )( 0.4 m ) = 0

M A ( 9.81 N )( 0.2 m ) ( 4.905 N )( 0.3 m ) + ( 6 N )( 0.4 m ) = 0

M A = 1.03350 N m

or M A = 1.034 N m
PROBLEM 4.50
In a laboratory experiment, students hang the masses shown from a beam
of negligible mass. Determine the range of values of the force exerted on
the beam by the adjustable support E for which the magnitude of the
couple at A does not exceed 2.5 N m.

SOLUTION

( )
WB = mB g = 1 kg 9.81 m/s 2 = 9.81 N

( )
WC = mC g = 0.5 kg 9.81 m/s 2 = 4.905 N

Maximum M A value is 2.5 N m

Fmin : From f.b.d. of beam ABCD with M A = 2.5 N m

M A = 0: 2.5 N m WB ( 0.2 m ) WC ( 0.3 m )

+ Fmin ( 0.4 m ) = 0

2.5 N m ( 9.81 N )( 0.2 m ) ( 4.905 N )( 0.3 m ) + Fmin ( 0.4 m ) = 0

Fmin = 2.3338 N

or Fmin = 2.33 N

Fmax : From f.b.d. of beam ABCD with M A = 2.5 N m

M A = 0: 2.5 N m WB ( 0.2 m ) WC ( 0.3 m )

+ Fmax ( 0.4 m ) = 0

2.5 N m ( 9.81 N )( 0.2 m ) ( 4.905 N )( 0.3 m ) + Fmax ( 0.4 m ) = 0

Fmax = 14.8338 N

or Fmax = 14.83 N

or 2.33 N FE 14.83 N
PROBLEM 4.51
Knowing that the tension in wire BD is 300 lb, determine the reaction at
fixed support C for the frame shown.

SOLUTION

From f.b.d. of frame with T = 300 lb

5
Fx = 0: C x 100 lb + 300 lb = 0
13
C x = 15.3846 lb or C x = 15.3846 lb

12
Fy = 0: C y 180 lb 300 lb = 0
13
C y = 456.92 lb or C y = 456.92 lb

Then C = C x2 + C y2 = (15.3846 )2 + ( 456.92 )2 = 457.18 lb

Cy 1 456.92
and = tan 1 = tan = 88.072
x
C 15.3846

or C = 457 lb 88.1

12
M C = 0: M C + (180 lb )( 20 in.) + (100 lb )(16 in.) 300 lb (16 in.) = 0
13
M C = 769.23 lb in.

or M C = 769 lb in.
PROBLEM 4.52
Determine the range of allowable values of the tension in wire BD if the
magnitude of the couple at the fixed support C is not to exceed 75 lb ft.

SOLUTION

Tmax From f.b.d. of frame with M C = 75 lb ft = 900 lb in.

12
M C = 0: 900 lb in. + (180 lb )( 20 in.) + (100 lb )(16 in.) Tmax (16 in.) = 0
13
Tmax = 413.02 lb

Tmin From f.b.d. of frame with M C = 75 lb ft = 900 lb in.

12
M C = 0: 900 lb in. + (180 lb )( 20 in.) + (100 lb )(16 in.) Tmin (16 in.) = 0
13
Tmin = 291.15 lb
291 lb T 413 lb
PROBLEM 4.53
Uniform rod AB of length l and weight W lies in a vertical plane and is
acted upon by a couple M. The ends of the rod are connected to small
rollers which rest against frictionless surfaces. (a) Express the angle
corresponding to equilibrium in terms of M, W, and l. (b) Determine the
value of corresponding to equilibrium when M = 1.5 lb ft,
W = 4 lb, and l = 2 ft.

SOLUTION

(a) From f.b.d. of uniform rod AB

Fx = 0: A cos 45 + B cos 45 = 0
A + B = 0 or B= A (1)

Fy = 0: A sin 45 + B sin 45 W = 0

A+B = 2W (2)
From Equations (1) and (2)

2A = 2W
1
A= W
2
From f.b.d. of uniform rod AB
l
M B = 0: W cos + M
2

1
W l cos ( 45 ) = 0 (3)
2
From trigonometric identity
cos ( ) = cos cos + sin sin

Equation (3) becomes


Wl Wl
cos + M ( cos + sin ) = 0
2 2
PROBLEM 4.53 CONTINUED

Wl Wl Wl
or cos + M cos sin = 0
2 2 2
2M
sin =
Wl
2M
or = sin 1
Wl

2 (1.5 lb ft )
(b) = sin 1 = 22.024
( 4 lb )( 2 ft )
or = 22.0
PROBLEM 4.54
A slender rod AB, of weight W, is attached to blocks A and B, which
move freely in the guides shown. The blocks are connected by an elastic
cord which passes over a pulley at C. (a) Express the tension in the cord
in terms of W and . (b) Determine the value of for which the tension
in the cord is equal to 3W.

SOLUTION

(a) From f.b.d. of rod AB


l
M C = 0: T ( l sin ) + W cos T ( l cos ) = 0
2
W cos
T =
2 ( cos sin )

Dividing both numerator and denominator by cos ,

W 1
T =
2 1 tan

W

or T = 2
(1 tan )
(b) For T = 3W ,

W

3W = 2
(1 tan )
1
1 tan =
6
5
or = tan 1 = 39.806
6
or = 39.8
PROBLEM 4.55
A thin, uniform ring of mass m and radius R is attached by a frictionless
pin to a collar at A and rests against a small roller at B. The ring lies in a
vertical plane, and the collar can move freely on a horizontal rod and is
acted upon by a horizontal force P. (a) Express the angle
corresponding to equilibrium in terms of m and P. (b) Determine the
value of corresponding to equilibrium when m = 500 g and P = 5 N.

SOLUTION

(a) From f.b.d. of ring


M C = 0: P ( R cos + R cos ) W ( R sin ) = 0

2P = W tan where W = mg

2P
tan =
mg

2P
or = tan 1
mg
(b) Have m = 500 g = 0.500 kg and P = 5 N

2 (5 N )
= tan 1
(
( 0.500 kg ) 9.81 m/s 2 )

= 63.872
or = 63.9
PROBLEM 4.56
Rod AB is acted upon by a couple M and two forces, each of magnitude
P. (a) Derive an equation in , P, M, and l which must be satisfied when
the rod is in equilibrium. (b) Determine the value of corresponding to
equilibrium when M = 150 lb in., P = 20 lb, and l = 6 in.

SOLUTION

(a) From f.b.d. of rod AB


M C = 0: P ( l cos ) + P ( l sin ) M = 0

M
or sin + cos =
Pl
(b) For M = 150 lb in., P = 20 lb, and l = 6 in.

150 lb in. 5
sin + cos = = = 1.25
( 20 lb )( 6 in.) 4
Using identity sin 2 + cos 2 = 1

( )
1
sin + 1 sin 2 2
= 1.25

( )
1
1 sin 2 2
= 1.25 sin

1 sin 2 = 1.5625 2.5sin + sin 2

2sin 2 2.5sin + 0.5625 = 0


Using quadratic formula

( 2.5 ) ( 6.25) 4 ( 2 )( 0.5625)


sin =
2 ( 2)

2.5 1.75
=
4
or sin = 0.95572 and sin = 0.29428
= 72.886 and = 17.1144
or = 17.11 and = 72.9
PROBLEM 4.57
A vertical load P is applied at end B of rod BC. The constant of the spring
is k, and the spring is unstretched when = 90o. (a) Neglecting the
weight of the rod, express the angle corresponding to equilibrium in
terms of P, k, and l. (b) Determine the value of corresponding to
1
equilibrium when P = kl.
4

SOLUTION

First note
T = tension in spring = ks
where s = elongation of spring

( ) ( AB )
= AB
= 90

90
= 2l sin 2l sin
2 2

1
= 2l sin
2 2

1
T = 2kl sin (1)
2 2
(a) From f.b.d. of rod BC

M C = 0: T l cos P ( l sin ) = 0
2
Substituting T From Equation (1)
1
2kl sin l cos 2 P ( l sin ) = 0
2
2

1
2kl 2 sin cos 2 Pl 2sin 2 cos 2 = 0
2
2


Factoring out 2l cos , leaves
2
PROBLEM 4.57 CONTINUED

1
kl sin P sin = 0
2 2 2
or
1 kl
sin =
2 2 kl P

kl
= 2sin 1
2 ( kl P )
kl
(b) P =
4

kl 4
kl 1 4
= 2sin 1 = 2sin 1 = 2sin
(
2 kl kl
4 ) 2 3 kl 3 2

= 2sin 1 ( 0.94281)

= 141.058
or = 141.1
PROBLEM 4.58
Solve Sample Problem 4.5 assuming that the spring is unstretched when
= 90o.

SOLUTION

First note
T = tension in spring = ks
where s = deformation of spring
= r
F = kr
From f.b.d. of assembly
M 0 = 0: W ( l cos ) F ( r ) = 0

or Wl cos kr 2 = 0

kr 2
cos =
Wl
For k = 250 lb/in., r = 3 in., l = 8 in., W = 400 lb

cos =
( 250 lb/in.)( 3 in.)2
( 400 lb )(8 in.)
or cos = 0.703125
Solving numerically,
= 0.89245 rad
or = 51.134
Then = 90 + 51.134 = 141.134
or = 141.1
PROBLEM 4.59
A collar B of weight W can move freely along the vertical rod shown. The
constant of the spring is k, and the spring is unstretched when = 0.
(a) Derive an equation in , W, k, and l which must be satisfied when the
collar is in equilibrium. (b) Knowing that W = 3 lb, l = 6 in., and
k = 8 lb/ft, determine the value of corresponding to equilibrium.

SOLUTION

First note T = ks
where k = spring constant
s = elongation of spring

l l
= l = (1 cos )
cos cos
kl
T = (1 cos )
cos
(a) From f.b.d. of collar B
Fy = 0: T sin W = 0

kl
or (1 cos ) sin W = 0
cos
W
or tan sin =
kl
(b) For W = 3 lb, l = 6 in., k = 8 lb/ft

6 in.
l = = 0.5 ft
12 in./ft

3 lb
tan sin = = 0.75
(8 lb/ft )( 0.5 ft )
Solving Numerically,
= 57.957
or = 58.0
PROBLEM 4.60
A slender rod AB, of mass m, is attached to blocks A and B which move
freely in the guides shown. The constant of the spring is k, and the spring
is unstretched when = 0 . (a) Neglecting the mass of the blocks, derive
an equation in m, g, k, l, and which must be satisfied when the rod is in
equilibrium. (b) Determine the value of when m = 2 kg, l = 750
mm, and k = 30 N/m.

SOLUTION

First note
Fs = spring force = ks
where k = spring constant
s = spring deformation
= l l cos

= l (1 cos )

Fs = kl (1 cos )

(a) From f.b.d. of assembly


l
M D = 0: Fs ( l sin ) W cos = 0
2

l
kl (1 cos )( l sin ) W cos = 0
2

W
kl ( sin cos sin ) cos = 0
2
Dividing by cos
W
kl ( tan sin ) =
2
W
tan sin =
2kl
mg
or tan sin =
2kl
(b) For m = 2 kg, l = 750 mm, k = 30 N/m
l = 750 mm = 0.750 m
PROBLEM 4.60 CONTINUED

( 2 kg ) ( 9.81 m/s2 )
Then tan sin = = 0.436
2 ( 30 N/m )( 0.750 m )

Solving Numerically,
= 50.328
or = 50.3
PROBLEM 4.61
The bracket ABC can be supported in the eight different ways shown. All
connections consist of smooth pins, rollers, or short links. In each case,
determine whether (a) the plate is completely, partially, or improperly
constrained, (b) the reactions are statically determinate or indeterminate,
(c) the equilibrium of the plate is maintained in the position shown. Also,
wherever possible, compute the reactions assuming that the magnitude of
the force P is 100 N.

SOLUTION
1. Three non-concurrent, non-parallel reactions
(a) Completely constrained
(b) Determinate
(c) Equilibrium
From f.b.d. of bracket:
M A = 0: B (1 m ) (100 N )( 0.6 m ) = 0

B = 60.0 N

Fx = 0: Ax 60 N = 0

A x = 60.0 N

Fy = 0: Ay 100 N = 0

A y = 100 N

Then A= ( 60.0 )2 + (100 )2 = 116.619 N

100
and = tan 1 = 59.036
60.0
A = 116.6 N 59.0
2. Four concurrent reactions through A
(a) Improperly constrained
(b) Indeterminate
(c) No equilibrium
3. Two reactions
(a) Partially constrained
(b) Determinate
(c) Equilibrium
PROBLEM 4.61 CONTINUED
From f.b.d. of bracket
M A = 0: C (1.2 m ) (100 N )( 0.6 m ) = 0

C = 50.0 N
Fy = 0: A 100 N + 50 N = 0

A = 50.0 N
4. Three non-concurrent, non-parallel reactions
(a) Completely constrained
(b) Determinate
(c) Equilibrium
From f.b.d. of bracket
1.0
= tan 1 = 39.8
1.2

BC = (1.2 )2 + (1.0 )2 = 1.56205 m

1.2
M A = 0: B (1 m ) (100 N )( 0.6 m ) = 0
1.56205
B = 78.1 N 39.8

Fx = 0: C ( 78.102 N ) cos 39.806 = 0

C = 60.0 N
Fy = 0: A + ( 78.102 N ) sin 39.806 100 N = 0

A = 50.0 N
5. Four non-concurrent, non-parallel reactions
(a) Completely constrained
(b) Indeterminate
(c) Equilibrium
From f.b.d. of bracket
M C = 0: (100 N )( 0.6 m ) Ay (1.2 m ) = 0
Ay = 50 N or A y = 50.0 N

6. Four non-concurrent non-parallel reactions


(a) Completely constrained
(b) Indeterminate
(c) Equilibrium
PROBLEM 4.61 CONTINUED
From f.b.d. of bracket
M A = 0: Bx (1 m ) (100 N )( 0.6 m ) = 0

Bx = 60.0 N

or B x = 60.0 N

Fx = 0: 60 + Ax = 0

Ax = 60.0 N

or A x = 60.0 N
7. Three non-concurrent, non-parallel reactions
(a) Completely constrained
(b) Determinate
(c) Equilibrium
From f.b.d. of bracket

Fx = 0: Ax = 0

M A = 0: C (1.2 m ) (100 N )( 0.6 m ) = 0

C = 50.0 N
or C = 50.0 N

Fy = 0: Ay 100 N + 50.0 N = 0

Ay = 50.0 N

A = 50.0 N
8. Three concurrent, non-parallel reactions
(a) Improperly constrained
(b) Indeterminate
(c) No equilibrium
PROBLEM 4.62
Eight identical 20 30-in. rectangular plates, each weighing 50 lb, are
held in a vertical plane as shown. All connections consist of frictionless
pins, rollers, or short links. For each case, answer the questions listed in
Problem 4.61, and, wherever possible, compute the reactions.
P6.1 The bracket ABC can be supported in the eight different ways
shown. All connections consist of smooth pins, rollers, or short links. In
each case, determine whether (a) the plate is completely, partially, or
improperly constrained, (b) the reactions are statically determinate or
indeterminate, (c) the equilibrium of the plate is maintained in the
position shown. Also, wherever possible, compute the reactions assuming
that the magnitude of the force P is 100 N.

SOLUTION
1. Three non-concurrent, non-parallel reactions
(a) Completely constrained
(b) Determinate
(c) Equilibrium
From f.b.d. of plate
M A = 0: C ( 30 in.) 50 lb (15 in.) = 0

C = 25.0 lb

Fx = 0: Ax = 0

Fy = 0: Ay 50 lb + 25 lb = 0

Ay = 25 lb A = 25.0 lb

2. Three non-current, non-parallel reactions


(a) Completely constrained
(b) Determinate
(c) Equilibrium
From f.b.d. of plate

Fx = 0: B=0

M B = 0: ( 50 lb )(15 in.) D ( 30 in.) = 0


D = 25.0 lb
Fy = 0: 25.0 lb 50 lb + C = 0

C = 25.0 lb
PROBLEM 4.62 CONTINUED
3. Four non-concurrent, non-parallel reactions
(a) Completely constrained
(b) Indeterminate
(c) Equilibrium
From f.b.d. of plate
M D = 0: Ax ( 20 in.) ( 50 lb )(15 in.)

A x = 37.5 lb

Fx = 0: Dx + 37.5 lb = 0

D x = 37.5 lb
4. Three concurrent reactions
(a) Improperly constrained
(b) Indeterminate
(c) No equilibrium
5. Two parallel reactions
(a) Partial constraint
(b) Determinate
(c) Equilibrium
From f.b.d. of plate
M D = 0: C ( 30 in.) ( 50 lb )(15 in.) = 0

C = 25.0 lb
Fy = 0: D 50 lb + 25 lb = 0

D = 25.0 lb
6. Three non-concurrent, non-parallel reactions
(a) Completely constrained
(b) Determinate
(c) Equilibrium
From f.b.d. of plate
M D = 0: B ( 20 in.) ( 50 lb )(15 in.) = 0

B = 37.5 lb

Fx = 0: Dx + 37.5 lb = 0 D x = 37.5 lb

Fy = 0: Dy 50 lb = 0 D y = 50.0 lb

or D = 62.5 lb 53.1
PROBLEM 4.62 CONTINUED
7. Two parallel reactions
(a) Improperly constrained
(b) Reactions determined by dynamics
(c) No equilibrium
8. Four non-concurrent, non-parallel reactions
(a) Completely constrained
(b) Indeterminate
(c) Equilibrium
From f.b.d. of plate
M D = 0: B ( 30 in.) ( 50 lb )(15 in.) = 0

B = 25.0 lb
Fy = 0: Dy 50 lb + 25.0 lb = 0

D y = 25.0 lb

Fx = 0: Dx + C = 0
PROBLEM 4.63
Horizontal and vertical links are hinged to a wheel, and forces are applied
to the links as shown. Knowing that a = 3.0 in., determine the value of P
and the reaction at A.

SOLUTION

As shown on the f.b.d., the wheel is a three-force body. Let point D be


the intersection of the three forces.

From force triangle

A P 21 lb
= =
5 4 3

4
P= ( 21 lb ) = 28 lb
3

or P = 28.0 lb

5
and A= ( 21 lb ) = 35 lb
3

3
= tan 1 = 36.870

4

A = 35.0 lb 36.9
PROBLEM 4.64
Horizontal and vertical links are hinged to a wheel, and forces are applied
to the links as shown. Determine the range of values of the distance a for
which the magnitude of the reaction at A does not exceed 42 lb.

SOLUTION

Let D be the intersection of the three forces acting on the wheel.

From the force triangle

21 lb A
=
a 16 + a 2

16
or A = 21 +1
a2

For A = 42 lb

21 lb 42 lb
=
a 16 + a 2

16 + a 2
or a2 =
4

16
or a= = 2.3094 in.
3

or a 2.31 in.

16
Since A = 21 +1
a2

as a increases, A decreases
PROBLEM 4.65
Using the method of Section 4.7, solve Problem 4.21.
P4.21 The required tension in cable AB is 800 N. Determine (a) the
vertical force P which must be applied to the pedal, (b) the corresponding
reaction at C.

SOLUTION

Let E be the intersection of the three forces acting on the pedal device.

First note

(180 mm ) sin 60
= tan 1 = 21.291
400 mm

From force triangle

(a) P = ( 800 N ) tan 21.291

= 311.76 N

or P = 312 N

800 N
(b) C =
cos 21.291

= 858.60 N

or C = 859 N 21.3
PROBLEM 4.66
Using the method of Section 4.7, solve Problem 4.22.
P4.22 Determine the maximum tension which can be developed in cable
AB if the maximum allowable value of the reaction at C is 1000 N.

SOLUTION

Let E be the intersection of the three forces acting on the pedal device.

First note

(180 mm ) sin 60
= tan 1 = 21.291
400 mm

From force triangle

Tmax = (1000 N ) cos 21.291

= 931.75 N

or Tmax = 932 N
PROBLEM 4.67
To remove a nail, a small block of wood is placed under a crowbar, and a
horizontal force P is applied as shown. Knowing that l = 3.5 in. and
P = 30 lb, determine the vertical force exerted on the nail and the
reaction at B.

SOLUTION

Let D be the intersection of the three forces acting on the crowbar.

First note

( 36 in.) sin 50
= tan 1 = 82.767
3.5 in.

From force triangle

FN = P tan = ( 30 lb ) tan 82.767

= 236.381 lb

on nail FN = 236 lb

P 30 lb
RB = = = 238.28 lb
cos cos82.767

or R B = 238 lb 82.8
PROBLEM 4.68
To remove a nail, a small block of wood is placed under a crowbar, and a
horizontal force P is applied as shown. Knowing that the maximum
vertical force needed to extract the nail is 600 lb and that the horizontal
force P is not to exceed 65 lb, determine the largest acceptable value of
distance l.

SOLUTION

Let D be the intersection of the three forces acting on the crowbar.

From force diagram

FN 600 lb
tan = = = 9.2308
P 65 lb

= 83.817

From f.b.d.

tan =
( 36 in.) sin 50
l

l =
( 36 in.) sin 50 = 2.9876 in.
tan 83.817

or l = 2.99 in.
PROBLEM 4.69
For the frame and loading shown, determine the reactions at C and D.

SOLUTION

Since member BD is acted upon by two forces, B and D, they must be colinear, have the same magnitude, and
be opposite in direction for BD to be in equilibrium. The force B acting at B of member ABC will be equal in
magnitude but opposite in direction to force B acting on member BD. Member ABC is a three-force body with
member forces intersecting at E. The f.b.d.s of members ABC and BD illustrate the above conditions. The
force triangle for member ABC is also shown. The angles and are found from the member dimensions:

0.5 m
= tan 1 = 26.565
1.0 m

1.5 m
= tan 1 = 56.310
1.0 m
Applying the law of sines to the force triangle for member ABC,
150 N C B
= =
sin ( ) sin ( 90 + ) sin ( 90 )

150 N C B
or = =
sin 29.745 sin116.565 sin 33.690

C =
(150 N ) sin116.565 = 270.42 N
sin 29.745
or C = 270 N 56.3

and D= B=
(150 N ) sin 33.690 = 167.704 N
sin 29.745
or D = 167.7 N 26.6
PROBLEM 4.70
For the frame and loading shown, determine the reactions at A and C.

SOLUTION

Since member AB is acted upon by two forces, A and B, they must be colinear, have the same magnitude, and
be opposite in direction for AB to be in equilibrium. The force B acting at B of member BCD will be equal in
magnitude but opposite in direction to force B acting on member AB. Member BCD is a three-force body with
member forces intersecting at E. The f.b.d.s of members AB and BCD illustrate the above conditions. The
force triangle for member BCD is also shown. The angle is found from the member dimensions:

60 m
= tan 1 = 30.964
100 m
Applying of the law of sines to the force triangle for member BCD,
130 N B C
= =
sin ( 45 ) sin sin135

130 N B C
or = =
sin14.036 sin 30.964 sin135

A= B=
(130 N ) sin 30.964 = 275.78 N
sin14.036
or A = 276 N 45.0

and C =
(130 N ) sin135 = 379.02 N
sin14.036
or C = 379 N 59.0
PROBLEM 4.71
To remove the lid from a 5-gallon pail, the tool shown is used to apply an
upward and radially outward force to the bottom inside rim of the lid.
Assuming that the rim rests against the tool at A and that a 100-N force is
applied as indicated to the handle, determine the force acting on the rim.

SOLUTION

The three-force member ABC has forces that intersect at D, where

90 mm
= tan 1
yDC yBC 45 mm

and

yDC =
xBC
=
( 360 mm ) cos 35
tan 20 tan 20

= 810.22 mm

yBC = ( 360 mm ) sin 35

= 206.49 mm

90
= tan 1 = 9.1506
558.73

Based on the force triangle, the law of sines gives

100 N A
=
sin sin 20

A=
(100 N ) sin 20 = 215.07 N
sin 9.1506

or A = 215 N 80.8 on tool

and A = 215 N 80.8 on rim of can


PROBLEM 4.72
To remove the lid from a 5-gallon pail, the tool shown is used to apply
an upward and radially outward force to the bottom inside rim of the
lid. Assuming that the top and the rim of the lid rest against the tool at
A and B, respectively, and that a 60-N force is applied as indicated to
the handle, determine the force acting on the rim.

SOLUTION

The three-force member ABC has forces that intersect at point D, where,
from the law of sines ( CDE )

L 150 mm + (19 mm ) tan 35


=
sin 95 sin 30

L = 325.37 mm

Then

45 mm
= tan 1
yBD

where
yBD = L y AE 22 mm

19 mm
= 325.37 mm 22 mm
cos 35
= 280.18 mm

45 mm
= tan 1 = 9.1246
280.18 mm

Applying the law of sines to the force triangle,

B 60 N
=
sin150 sin 9.1246

B = 189.177 N

Or, on member B = 189.2 N 80.9

and, on lid B = 189.2 N 80.9


PROBLEM 4.73
A 200-lb crate is attached to the trolley-beam system shown. Knowing
that a = 1.5 ft, determine (a) the tension in cable CD, (b) the reaction
at B.

SOLUTION

From geometry of forces


y
= tan 1 BE
1.5 ft
where
yBE = 2.0 yDE
= 2.0 1.5 tan 35
= 0.94969 ft
0.94969
= tan 1 = 32.339
1.5

and = 90 = 90 32.339 = 57.661

= + 35 = 32.339 + 35 = 67.339

Applying the law of sines to the force triangle,

200 lb T B
= =
sin sin sin 55

or
( 200 lb ) =
T
=
B
sin 67.339 sin 57.661 sin 55

(a) T =
( 200 lb )( sin 57.661 ) = 183.116 lb
sin 67.339
or T = 183.1 lb

(b) B=
( 200 lb )( sin 55 ) = 177.536 lb
sin 67.339

or B = 177.5 lb 32.3
PROBLEM 4.74
Solve Problem 4.73 assuming that a = 3 ft.
P4.73 A 200-lb crate is attached to the trolley-beam system shown.
Knowing that a = 1.5 ft, determine (a) the tension in cable CD, (b) the
reaction
at B.

SOLUTION

From geometry of forces


y
= tan 1 BE
3 ft
where
yBE = yDE 2.0 ft
= 3tan 35 2.0
= 0.100623 ft
0.100623
= tan 1 = 1.92103
3
and = 90 + = 90 + 1.92103 = 91.921
= 35 = 35 1.92103 = 33.079
Applying the law of sines to the force triangle,
200 lb T B
= =
sin sin sin 55
200 lb T B
or = =
sin 33.079 sin 91.921 sin 55

(a) T =
( 200 lb )( sin 91.921 ) = 366.23 lb
sin 33.079
or T = 366 lb

(b) B=
( 200 lb )( sin 55 ) = 300.17 lb
sin 33.079

or B = 300 lb 1.921
PROBLEM 4.75
A 20-kg roller, of diameter 200 mm, which is to be used on a tile floor, is
resting directly on the subflooring as shown. Knowing that the thickness
of each tile is 8 mm, determine the force P required to move the roller
onto the tiles if the roller is pushed to the left.

SOLUTION

Based on the roller having impending motion to the left, the only contact
between the roller and floor will be at the edge of the tile.

First note ( )
W = mg = ( 20 kg ) 9.81 m/s 2 = 196.2 N

From the geometry of the three forces acting on the roller

92 mm
= cos 1 = 23.074
100 mm

and = 90 30

= 60 23.074

= 36.926

Applying the law of sines to the force triangle,

W P
=
sin sin

196.2 N P
or =
sin 36.926 sin 23.074

P = 127.991 N
or P = 128.0 N 30
PROBLEM 4.76
A 20-kg roller, of diameter 200 mm, which is to be used on a tile floor, is
resting directly on the subflooring as shown. Knowing that the thickness
of each tile is 8 mm, determine the force P required to move the roller
onto the tiles if the roller is pulled to the right.

SOLUTION

Based on the roller having impending motion to the right, the only
contact between the roller and floor will be at the edge of the tile.

First note (
W = mg = ( 20 kg ) 9.81 m/s 2 )
= 196.2 N

From the geometry of the three forces acting on the roller

92 mm
= cos 1 = 23.074
100 mm

and = 90 + 30

= 120 23.074

= 96.926

Applying the law of sines to the force triangle,

W P
=
sin sin

196.2 N P
or =
sin 96.926 sin 23.074

P = 77.460 N
or P = 77.5 N 30
PROBLEM 4.77
A small hoist is mounted on the back of a pickup truck and is used to lift
a 120-kg crate. Determine (a) the force exerted on the hoist by the
hydraulic cylinder BC, (b) the reaction at A.

SOLUTION

First note ( )
W = mg = (120 kg ) 9.81 m/s 2 = 1177.2 N

From the geometry of the three forces acting on the small hoist

x AD = (1.2 m ) cos 30 = 1.03923 m

y AD = (1.2 m ) sin 30 = 0.6 m

and yBE = x AD tan 75 = (1.03923 m ) tan 75 = 3.8785 m

yBE 0.4 m 1 3.4785


Then = tan 1 = tan = 73.366
x AD 1.03923

= 75 = 75 73.366 = 1.63412

= 180 15 = 165 1.63412 = 163.366

Applying the law of sines to the force triangle,

W B A
= =
sin sin sin15

1177.2 N B A
or = =
sin1.63412 sin163.366 sin15

(a) B = 11 816.9 N
or B = 11.82 kN 75.0
(b) A = 10 684.2 N
or A = 10.68 kN 73.4
PROBLEM 4.78
The clamp shown is used to hold the rough workpiece C. Knowing that
the maximum allowable compressive force on the workpiece is 200 N
and neglecting the effect of friction at A, determine the corresponding
(a) reaction at B, (b) reaction at A, (c) tension in the bolt.

SOLUTION

From the geometry of the three forces acting on the clamp

y AD = (105 mm ) tan 78 = 493.99 mm

yBD = y AD 70 mm = ( 493.99 70 ) mm = 423.99 mm

y 423.99
Then = tan 1 BD 1
= tan = 65.301
195 mm 195

= 90 12 = 78 65.301 = 12.6987

(a) Based on the maximum allowable compressive force on the


workpiece of 200 N,
( RB ) y = 200 N

or RB sin = 200 N

200 N
RB = = 220.14 N
sin 65.301
or R B = 220 N 65.3
Applying the law of sines to the force triangle,

RB NA T
= =
sin12 sin sin ( 90 + )

220.14 N NA T
or = =
sin12 sin12.6987 sin155.301

(b) N A = 232.75 N

or N A = 233 N
(c) T = 442.43 N
or T = 442 N
PROBLEM 4.79
A modified peavey is used to lift a 0.2-m-diameter log of mass 36 kg.
Knowing that = 45 and that the force exerted at C by the worker is
perpendicular to the handle of the peavey, determine (a) the force exerted
at C, (b) the reaction at A.

SOLUTION

First note ( )
W = mg = ( 36 kg ) 9.81 m/s 2 = 353.16 N

From the geometry of the three forces acting on the modified peavey

1.1 m
= tan 1 = 40.236
1.1 m + 0.2 m

= 45 = 45 40.236 = 4.7636

Applying the law of sines to the force triangle,

W C A
= =
sin sin sin135

353.16 N C A
or = =
sin 40.236 sin 4.7636 sin135

(a) C = 45.404 N

or C = 45.4 N 45.0
(b) A = 386.60 N
or A = 387 N 85.2
PROBLEM 4.80
A modified peavey is used to lift a 0.2-m-diameter log of mass 36 kg.
Knowing that = 60 and that the force exerted at C by the worker is
perpendicular to the handle of the peavey, determine (a) the force exerted
at C, (b) the reaction at A.

SOLUTION

First note ( )
W = mg = ( 36 kg ) 9.81 m/s 2 = 353.16 N

From the geometry of the three forces acting on the modified peavey
1.1 m
= tan 1
DC + 0.2 m

where DC = (1.1 m + a ) tan 30

R
a= R
tan 30

0.1 m
= 0.1 m
tan 30
= 0.073205 m

DC = (1.173205 ) tan 30

= 0.67735 m
1.1
and = tan 1 = 51.424
0.87735
= 60 = 60 51.424 = 8.5756
Applying the law of sines to the force triangle,
W C A
= =
sin sin sin120

353.16 N C A
or = =
sin 51.424 sin 8.5756 sin120
(a) C = 67.360 N
or C = 67.4 N 30
(b) A = 391.22 N
or A = 391 N 81.4
PROBLEM 4.81
Member ABC is supported by a pin and bracket at B and by an
inextensible cord at A and C and passing over a frictionless pulley at D.
The tension may be assumed to be the same in portion AD and CD of the
cord. For the loading shown and neglecting the size of the pulley,
determine the tension in the cord and the reaction at B.

SOLUTION
From the f.b.d. of member ABC, it is seen that the member can be treated
as a three-force body.

From the force triangle

T 300 3
=
T 4

3T = 4T 1200

T = 1200 lb

B 5
Also, =
T 4

5 5
B= T = (1200 lb ) = 1500 lb
4 4

3
= tan 1 = 36.870
4

and B = 1500 lb 36.9


PROBLEM 4.82
Member ABCD is supported by a pin and bracket at C and by an
inextensible cord attached at A and D and passing over frictionless
pulleys at B and E. Neglecting the size of the pulleys, determine the
tension in the cord and the reaction at C.

SOLUTION
From the geometry of the forces acting on member ABCD

200
= tan 1 = 33.690
300

375
= tan 1 = 61.928
200

= 61.928 33.690 = 28.237

180 = 180 61.928 = 118.072

Applying the law of sines to the force triangle,

T 80 N T C
= =
sin ( ) sin sin (180 )

T 80 N T C
or = =
sin 28.237 sin 33.690 sin118.072

Then (T 80 N ) sin 33.690 = T sin 28.237

T = 543.96 N

or T = 544 N

and ( 543.96 N ) sin118.072 = C sin 33.690


C = 865.27 N

or C = 865 N 33.7
PROBLEM 4.83
Using the method of Section 4.7, solve Problem 4.18.
P4.18 Determine the reactions at A and B when (a) h = 0 , (b) h = 8 in.

SOLUTION
(a) Based on symmetry
= 30
From force triangle
A = B = 40 lb
or A = 40.0 lb 30
and B = 40.0 lb 30

(b) From geometry of forces


8 in. (10 in.) tan 30
= tan 1 = 12.5521
10 in.
Also,
30 = 30 12.5521 = 17.4479
90 + = 90 + 12.5521 = 102.5521
Applying law of sines to the force triangle,
40 lb A B
= =
sin ( 30 ) sin 60 sin ( 90 + )

40 lb A B
or = =
sin17.4479 sin 60 sin102.5521
A = 115.533 lb
or A = 115.5 lb 12.55
B = 130.217 lb
or B = 130.2 lb 30.0
PROBLEM 4.84
Using the method of Section 4.7, solve Problem 4.28.
P4.28 A lever is hinged at C and is attached to a control cable at A. If the
lever is subjected to a 300-N vertical force at B, determine
(a) the tension in the cable, (b) the reaction at C.

SOLUTION
From geometry of forces acting on lever
yDA
= tan 1
xDA
where
yDA = 0.24 m y AC = 0.24 m ( 0.2 m ) sin 20

= 0.171596 m

xDA = ( 0.2 m ) cos 20

= 0.187939 m

0.171596
= tan 1 = 42.397
0.187939

y AC + yEA
= 90 tan 1
xCE

where xCE = ( 0.3 m ) cos 20 = 0.28191 m

y AC = ( 0.2 m ) sin 20 = 0.068404 m

yEA = ( xDA + xCE ) tan

= ( 0.187939 + 0.28191) tan 42.397

= 0.42898 m

0.49739
= 90 tan 1 = 29.544
0.28191

Also, 90 ( + ) = 90 71.941 = 18.0593

90 + = 90 + 42.397 = 132.397
PROBLEM 4.84 CONTINUED

Applying the law of sines to the force triangle,

300 N T C
= =
sin 90 ( + ) sin sin ( 90 + )

300 N T C
or = =
sin18.0593 sin 29.544 sin132.397

(a) T = 477.18 N or T = 477 N

(b) C = 714.67 N or C = 715 N 60.5


PROBLEM 4.85
Knowing that = 35o , determine the reaction (a) at B, (b) at C.

SOLUTION
From the geometry of the three forces applied to the member ABC

y
= tan 1 CD
R

where

yCD = R tan 55 R = 0.42815R

= tan 1 ( 0.42815 ) = 23.178

Then 55 = 55 23.178 = 31.822

90 + = 90 + 23.178 = 113.178

Applying the law of sines to the force triangle,

P B C
= =
sin ( 55 ) sin ( 90 + ) sin 35

P B C
or = =
sin 31.822 sin113.178 sin 35

(a) B = 1.74344P

or B = 1.743P 55.0

(b) C = 1.08780P

or C = 1.088P 23.2
PROBLEM 4.86
Knowing that = 50o , determine the reaction (a) at B, (b) at C.

SOLUTION
From the geometry of the three forces acting on member ABC

y
= tan 1 DC
R

where

yDC = R y AD = R 1 tan ( 90 50 )

= 0.160900 R

= tan 1 ( 0.160900 ) = 9.1406

Then 90 = 90 9.1406 = 80.859

40 + = 40 + 9.1406 = 49.141

Applying the law of sines to the force triangle,

P B C
= =
sin ( 40 + ) sin ( 90 ) sin 50

P B C
or = =
sin 49.141 sin ( 80.859 ) sin 50

(a) B = 1.30540P

or B = 1.305P 40.0

(b) C = 1.01286P

or C = 1.013P 9.14
PROBLEM 4.87
A slender rod of length L and weight W is held in equilibrium as shown,
with one end against a frictionless wall and the other end attached to a
cord of length S. Derive an expression for the distance h in terms of L and
S. Show that this position of equilibrium does not exist if S > 2L.

SOLUTION
From the f.b.d of the three-force member AB, forces must intersect at D.
Since the force T intersects point D, directly above G,
yBE = h
For triangle ACE:

S 2 = ( AE ) + ( 2h )
2 2
(1)

For triangle ABE:

L2 = ( AE ) + ( h )
2 2
(2)

Subtracting Equation (2) from Equation (1)


S 2 L2 = 3h 2 (3)

S 2 L2
or h =
3
As length S increases relative to length L, angle increases until rod AB
is vertical. At this vertical position:
h+L=S or h=SL
Therefore, for all positions of AB hSL (4)

S 2 L2
or SL
3

or ( )
S 2 L2 3 ( S L ) = 3 S 2 2SL + L2 = 3S 2 6SL + 3L2
2

or 0 2S 2 6SL + 4 L2

and 0 S 2 3SL + 2 L2 = ( S L )( S 2L )

For SL=0 S = L
Minimum value of S is L
For S 2L = 0 S = 2L
Maximum value of S is 2L
Therefore, equilibrium does not exist if S > 2L
PROBLEM 4.88
A slender rod of length L = 200 mm is held in equilibrium as shown,
with one end against a frictionless wall and the other end attached to a
cord of length S = 300 mm. Knowing that the mass of the rod is 1.5 kg,
determine (a) the distance h, (b) the tension in the cord, (c) the reaction
at B.

SOLUTION
From the f.b.d of the three-force member AB, forces must intersect at D.
Since the force T intersects point D, directly above G,
yBE = h
For triangle ACE:

S 2 = ( AE ) + ( 2h )
2 2
(1)

For triangle ABE:

L2 = ( AE ) + ( h )
2 2
(2)

Subtracting Equation (2) from Equation (1)


S 2 L2 = 3h 2

S 2 L2
or h =
3
(a) For L = 200 mm and S = 300 mm

h=
( 300 )2 ( 200 )2 = 129.099 mm
3
or h = 129.1 mm

(b) Have (
W = mg = (1.5 kg ) 9.81 m/s 2 = 14.715 N)
2h 2 (129.099 )
and = sin 1 = sin 1
s 300
= 59.391
From the force triangle
W 14.715 N
T = = = 17.0973 N
sin sin 59.391
or T = 17.10 N
W 14.715 N
(c) B= = = 8.7055 N
tan tan 59.391
or B = 8.71 N
PROBLEM 4.89
A slender rod of length L and weight W is attached to collars which can
slide freely along the guides shown. Knowing that the rod is in
equilibrium, derive an expression for the angle in terms of the
angle .

SOLUTION
As shown in the f.b.d of the slender rod AB, the three forces intersect at
C. From the force geometry

xGB
tan =
y AB

where

y AB = L cos

1
and xGB = L sin
2
1 L sin 1
tan = 2
= tan
L cos 2

or tan = 2 tan
PROBLEM 4.90
A 10-kg slender rod of length L is attached to collars which can slide
freely along the guides shown. Knowing that the rod is in equilibrium and
that = 25, determine (a) the angle that the rod forms with the
vertical, (b) the reactions at A and B.

SOLUTION
(a) As shown in the f.b.d. of the slender rod AB, the three forces
intersect at C. From the geometry of the forces
xCB
tan =
yBC

where
1
xCB = L sin
2
and yBC = L cos

1
tan = tan
2
or tan = 2 tan
For = 25
tan = 2 tan 25 = 0.93262
= 43.003
or = 43.0

(b) (
W = mg = (10 kg ) 9.81 m/s 2 = 98.1 N )
From force triangle
A = W tan

= ( 98.1 N ) tan 25

= 45.745 N
or A = 45.7 N

W 98.1 N
and B= = = 108.241 N
cos cos 25
or B = 108.2 N 65.0
PROBLEM 4.91
A uniform slender rod of mass 5 g and length 250 mm is balanced on a
glass of inner diameter 70 mm. Neglecting friction, determine the angle
corresponding to equilibrium.

SOLUTION
From the geometry of the forces acting on the three-force member AB
Triangle ACF
yCF = d tan

Triangle CEF xFE = yCF tan = d tan 2


Triangle AGE
d + xFE d + d tan 2
cos = =
L L

2 2

=
2d
L
(
1 + tan 2 )
Now (1 + tan ) = sec
2 2
and sec =
1
cos
2d 2d 1
Then cos = sec 2 =
L L cos 2

2d
cos3 =
L
For d = 70 mm and L = 250 mm

2 ( 70 )
cos3 = = 0.56
250
cos = 0.82426
and = 34.487
or = 34.5
PROBLEM 4.92
Rod AB is bent into the shape of a circular arc and is lodged between two
pegs D and E. It supports a load P at end B. Neglecting friction and the
weight of the rod, determine the distance c corresponding to equilibrium
when a = 1 in. and R = 5 in.

SOLUTION
Since yED = xED = a,

Slope of ED is 45

slope of HC is 45

Also DE = 2a

1 a
and DH = HE = DE =
2 2

For triangles DHC and EHC

a/ 2 a
sin = =
R 2R

Now c = R sin ( 45 )

For a = 1 in. and R = 5 in.

1 in.
sin = = 0.141421
2 ( 5 in.)

= 8.1301 or = 8.13

and c = ( 5 in.) sin ( 45 8.1301 ) = 3.00 in.

or c = 3.00 in.
PROBLEM 4.93
A uniform rod AB of weight W and length 2R rests inside a hemispherical
bowl of radius R as shown. Neglecting friction determine the angle
corresponding to equilibrium.

SOLUTION
Based on the f.b.d., the uniform rod AB is a three-force body. Point E is
the point of intersection of the three forces. Since force A passes through
O, the center of the circle, and since force C is perpendicular to the rod,
triangle ACE is a right triangle inscribed in the circle. Thus, E is a point
on the circle.

Note that the angle of triangle DOA is the central angle corresponding
to the inscribed angle of triangle DCA.

= 2

The horizontal projections of AE , ( x AE ) , and AG, ( x AG ) , are equal.

x AE = x AG = x A

or ( AE ) cos 2 = ( AG ) cos

and ( 2R ) cos 2 = R cos

Now cos 2 = 2cos 2 1

then 4cos 2 2 = cos

or 4cos 2 cos 2 = 0

Applying the quadratic equation

cos = 0.84307 and cos = 0.59307

= 32.534 and = 126.375(Discard)

or = 32.5
PROBLEM 4.94
A uniform slender rod of mass m and length 4r rests on the surface shown
and is held in the given equilibrium position by the force P. Neglecting
the effect of friction at A and C, (a) determine the angle , (b) derive an
expression for P in terms of m.

SOLUTION
The forces acting on the three-force member intersect at D.
(a) From triangle ACO
r 1 1
= tan 1 = tan = 18.4349 or = 18.43
3r 3
r
(b) From triangle DCG tan =
DC
r r
DC = = = 3r
tan tan18.4349
and DO = DC + r = 3r + r = 4r
yDO
= tan 1
x AG

where yDO = ( DO ) cos = ( 4r ) cos18.4349

= 3.4947r
and x AG = ( 2r ) cos = ( 2r ) cos18.4349

= 1.89737r
3.4947r
= tan 1 = 63.435
1.89737r
where 90 + ( ) = 90 + 45 = 135.00

Applying the law of sines to the force triangle,


mg R
= A
sin 90 + ( ) sin

RA = ( 0.44721) mg

Finally, P = RA cos

= ( 0.44721mg ) cos 63.435

mg
= 0.20000mg or P =
5
PROBLEM 4.95
A uniform slender rod of length 2L and mass m rests against a roller at D
and is held in the equilibrium position shown by a cord of length a.
Knowing that L = 200 mm, determine (a) the angle , (b) the length a.

SOLUTION
(a) The forces acting on the three-force member AB intersect at E. Since
triangle DBC is isosceles, DB = a.
From triangle BDE
ED = DB tan 2 = a tan 2
From triangle GED

ED =
( L a)
tan
La
a tan 2 = or a ( tan tan 2 + 1) = L (1)
tan
1 (1.25L )
L
From triangle BCD a= 2
or = 1.6cos (2)
cos a
Substituting Equation (2) into Equation (1) yields
1.6cos = 1 + tan tan 2
sin sin 2
Now tan tan 2 =
cos cos 2
sin 2sin cos
=
cos 2 cos 2 1

2 (1 cos 2 )
=
2cos 2 1

2 (1 cos 2 )
Then 1.6cos = 1 +
2cos 2 1

or 3.2cos3 1.6cos 1 = 0
Solving numerically = 23.515 or = 23.5
(b) From Equation (2) for L = 200 mm and = 23.5

5 ( 200 mm )
a= = 136.321 mm
8 cos 23.515
or a = 136.3 mm
PROBLEM 4.96
Gears A and B are attached to a shaft supported by bearings at C and D.
The diameters of gears A and B are 150 mm and 75 mm, respectively, and
the tangential and radial forces acting on the gears are as shown.
Knowing that the system rotates at a constant rate, determine the
reactions at C and D. Assume that the bearing at C does not exert any
axial force, and neglect the weights of the gears and the shaft.

SOLUTION
Assume moment reactions at the bearing supports are zero. From f.b.d. of
shaft
Fx = 0: Dx = 0

M D( z -axis ) = 0: C y (175 mm ) + ( 482 N ) ( 75 mm )

+ ( 2650 N ) ( 50 mm ) = 0

C y = 963.71 N

or C y = ( 964 N ) j

M D( y -axis ) = 0: Cz (175 mm ) + (1325 N ) ( 75 mm )

+ ( 964 N ) ( 50 mm ) = 0

C z = 843.29 N

or C z = ( 843 N ) k

and C = ( 964 N ) j ( 843 N ) k

M C ( z -axis ) = 0: ( 482 N ) (100 mm ) + Dy (175 mm )

+ ( 2650 N ) ( 225 mm ) = 0

D y = 3131.7 N

or D y = ( 3130 N ) j

M C ( y -axis ) = 0: (1325 N ) (100 mm ) Dz (175 mm )

+ ( 964 N ) ( 225 mm ) = 0

Dz = 482.29 N

or D z = ( 482 N ) k

and D = ( 3130 N ) j + ( 482 N ) k


PROBLEM 4.97
Solve Problem 4.96 assuming that for gear A the tangential and radial
forces are acting at E, so that FA = (1325 N)j + (482 N)k.
P4.96 Gears A and B are attached to a shaft supported by bearings at C and
D. The diameters of gears A and B are 150 mm and 75 mm, respectively,
and the tangential and radial forces acting on the gears are as shown.
Knowing that the system rotates at a constant rate, determine the reactions at
C and D. Assume that the bearing at C does not exert any axial force, and
neglect the weights of the gears and the shaft.

SOLUTION
Assume moment reactions at the bearing supports are zero. From f.b.d. of
shaft
Fx = 0: Dx = 0

M D( z -axis ) = 0: C y (175 mm ) (1325 N ) ( 75 mm )

+ ( 2650 N ) ( 50 mm ) = 0

C y = 189.286 N

or C y = (189.3 N ) j

M D( y -axis ) = 0: C z (175 mm ) + ( 482 N ) ( 75 mm )

+ ( 964 N ) ( 50 mm ) = 0

C z = 482.00 N

or C z = ( 482 N ) k

and C = (189.3 N ) j ( 482 N ) k

M C ( z -axis ) = 0: (1325 N ) (100 mm ) + Dy (175 mm )


+ ( 2650 N ) ( 225 mm ) = 0

Dy = 4164.3 N

or D y = ( 4160 N ) j

M C ( y -axis ) = 0: ( 482 N )(100 mm ) Dz (175 mm )

+ ( 964 N )( 225 mm ) = 0
Dz = 964.00 N

or D z = ( 964 N ) k

and D = ( 4160 N ) j + ( 964 N ) k


PROBLEM 4.98
Two transmission belts pass over sheaves welded to an axle supported by
bearings at B and D. The sheave at A has a radius of 50 mm, and the
sheave at C has a radius of 40 mm. Knowing that the system rotates with
a constant rate, determine (a) the tension T, (b) the reactions at B and D.
Assume that the bearing at D does not exert any axial thrust and neglect
the weights of the sheaves and the axle.

SOLUTION

Assume moment reactions at the bearing supports are zero. From f.b.d. of shaft
(a) M x-axis = 0: ( 240 N 180 N )( 50 mm ) + ( 300 N T )( 40 mm ) = 0
T = 375 N

(b) Fx = 0: Bx = 0

M D( z -axis ) = 0: ( 300 N + 375 N )(120 mm ) By ( 240 mm ) = 0


By = 337.5 N

M D( y -axis ) = 0: ( 240 N + 180 N )( 400 mm ) + Bz ( 240 mm ) = 0


Bz = 700 N

or B = ( 338 N ) j ( 700 N ) k

M B( z -axis ) = 0: ( 300 N + 375 N )(120 mm ) + D y ( 240 mm ) = 0

D y = 337.5 N

M B( y -axis ) = 0: ( 240 N + 180 N )(160 mm ) + Dz ( 240 mm ) = 0


Dz = 280 N

or D = ( 338 N ) j ( 280 N ) k
PROBLEM 4.99
For the portion of a machine shown, the 4-in.-diameter pulley A and
wheel B are fixed to a shaft supported by bearings at C and D. The spring
of constant 2 lb/in. is unstretched when = 0, and the bearing at C does
not exert any axial force. Knowing that = 180 and that the machine is
at rest and in equilibrium, determine (a) the tension T, (b) the reactions at
C and D. Neglect the weights of the shaft, pulley, and wheel.

SOLUTION

First, determine the spring force, FE , at = 180.

FE = ks x

where ks = 2 lb/in.

x = ( yE )final ( yE )initial = (12 in. + 3.5 in.) (12 in. 3.5 in.) = 7.0 in.

FE = ( 2 lb/in.)( 7.0 in.) = 14.0 lb

(a) From f.b.d. of machine part


M x = 0: ( 34 lb )( 2 in.) T ( 2 in.) = 0
T = 34 lb or T = 34.0 lb

(b) M D ( z -axis ) = 0: C y (10 in.) FE ( 2 in. + 1 in.) = 0

C y (10 in.) 14.0 lb ( 3 in.) = 0

C y = 4.2 lb or C y = ( 4.20 lb ) j

M D( y -axis ) = 0: C z (10 in.) + 34 lb ( 4 in.) + 34 lb ( 4 in.) = 0

C z = 27.2 lb or C z = ( 27.2 lb ) k

and C = ( 4.20 lb ) j ( 27.2 lb ) k


PROBLEM 4.99 CONTINUED

Fx = 0: Dx = 0

M C ( z -axis ) = 0: Dy (10 in.) FE (12 in. + 1 in.) = 0

or Dy (10 in.) 14.0 (13 in.) = 0

Dy = 18.2 lb or D y = (18.20 lb ) j

M C ( y -axis ) = 0: 2 ( 34 lb ) ( 6 in.) Dz (10 in.) = 0

Dz = 40.8 lb or D z = ( 40.8 lb ) k

and D = (18.20 lb ) j ( 40.8 lb ) k


PROBLEM 4.100
Solve Problem 4.99 for = 90.
P4.99 For the portion of a machine shown, the 4-in.-diameter pulley A and
wheel B are fixed to a shaft supported by bearings at C and D. The spring of
constant 2 lb/in. is unstretched when = 0, and the bearing at C does not
exert any axial force. Knowing that = 180 and that the machine is at rest
and in equilibrium, determine (a) the tension T, (b) the reactions at C and D.
Neglect the weights of the shaft, pulley, and wheel.

SOLUTION

First, determine the spring force, FE , at = 90.

FE = ks x

where ks = 2 lb/in.


and x = Lfinal Linitial = ( 3.5)2 + (12 )2 (12 3.5) = 12.5 8.5 = 4.0 in.

FE = ( 2 lb/in.)( 4.0 in.) = 8.0 lb

12.0 3.5
Then FE = (8.0 lb ) j + (8.0 lb ) k = ( 7.68 lb ) j + ( 2.24 lb ) k
12.5 12.5
(a) From f.b.d. of machine part
M x = 0: ( 34 lb ) ( 2 in.) T ( 2 in.) ( 7.68 lb )( 3.5 in.) = 0
T = 20.56 lb or T = 20.6 lb

(b) M D ( z -axis ) = 0: C y (10 in.) ( 7.68 lb ) ( 3.0 in.) = 0

C y = 2.304 lb or C y = ( 2.30 lb ) j

M D ( y -axis ) = 0: Cz (10 in.) + ( 34 lb ) ( 4.0 in.) + ( 20.56 lb ) ( 4.0 in.) ( 2.24 lb ) ( 3 in.) = 0

C z = 21.152 lb or C z = ( 21.2 lb ) k

and C = ( 2.30 lb ) j ( 21.2 lb ) k


PROBLEM 4.100 CONTINUED

Fx = 0: Dx = 0

M C ( z -axis ) = 0: Dy (10 in.) ( 7.68 lb )(13 in.) = 0

D y = 9.984 lb or D y = ( 9.98 lb ) j

M C ( y -axis ) = 0: ( 34 lb )( 6 in.) ( 20.56 lb )( 6 in.) Dz (10 in.) ( 2.24 lb )(13 in.) = 0

Dz = 35.648 lb or D z = ( 35.6 lb ) k

and D = ( 9.98 lb ) j ( 35.6 lb ) k


PROBLEM 4.101
A 1.2 2.4-m sheet of plywood having a mass of 17 kg has been
temporarily placed among three pipe supports. The lower edge of the
sheet rests on small collars A and B and its upper edge leans against pipe
C. Neglecting friction at all surfaces, determine the reactions at A, B,
and C.

SOLUTION

First note ( )
W = mg = (17 kg ) 9.81 m/s 2 = 166.77 N

h= (1.2 )2 (1.125)2 = 0.41758 m

From f.b.d. of plywood sheet

(1.125 m )
M z = 0: C ( h ) W =0
2

C ( 0.41758 m ) (166.77 N ) ( 0.5625 m ) = 0

C = 224.65 N or C = ( 225 N ) i

M B( y -axis ) = 0: ( 224.65 N ) ( 0.6 m ) + Ax (1.2 m ) = 0

Ax = 112.324 N or A x = (112.3 N ) i

M B( x-axis ) = 0: (166.77 N ) ( 0.3 m ) Ay (1.2 m ) = 0

Ay = 41.693 N or A y = ( 41.7 N ) j

M A( y -axis ) = 0: ( 224.65 N ) ( 0.6 m ) Bx (1.2 m ) = 0

Bx = 112.325 N or B x = (112.3 N ) i
PROBLEM 4.101 CONTINUED

M A( x-axis ) = 0: By (1.2 m ) (166.77 N ) ( 0.9 m ) = 0

By = 125.078 N or B y = (125.1 N ) j

A = (112.3 N ) i + ( 41.7 N ) j

B = (112.3 N ) i + (125.1 N ) j

C = ( 225 N ) i
PROBLEM 4.102
The 200 200-mm square plate shown has a mass of 25 kg and is
supported by three vertical wires. Determine the tension in each wire.

SOLUTION

First note ( )
W = mg = ( 25 kg ) 9.81 m/s 2 = 245.25 N

From f.b.d. of plate


M x = 0: ( 245.25 N ) (100 mm ) TA (100 mm ) TC ( 200 mm ) = 0

TA + 2TC = 245.25 N (1)

M z = 0: TB (160 mm ) + TC (160 mm ) ( 245.25 N ) (100 mm ) = 0

TB + TC = 153.281 N (2)
Fy = 0: TA + TB + TC 245.25 N = 0

TB + TC = 245.25 TA (3)
Equating Equations (2) and (3) yields
TA = 245.25 N 153.281 N = 91.969 N (4)
or TA = 92.0 N
Substituting the value of TA into Equation (1)

TC =
( 245.25 N 91.969 N ) = 76.641 N (5)
2
or TC = 76.6 N
Substituting the value of TC into Equation (2)
TB = 153.281 N 76.641 N = 76.639 N or TB = 76.6 N
TA = 92.0 N
TB = 76.6 N
TC = 76.6 N
PROBLEM 4.103
The 200 200-mm square plate shown has a mass of 25 kg and is
supported by three vertical wires. Determine the mass and location of the
lightest block which should be placed on the plate if the tensions in the
three cables are to be equal.

SOLUTION

First note ( )
WG = m p1g = ( 25 kg ) 9.81 m/s 2 = 245.25 N

( )
W1 = mg = m 9.81 m/s 2 = ( 9.81m ) N

From f.b.d. of plate


Fy = 0: 3T WG W1 = 0 (1)

M x = 0: WG (100 mm ) + W1 ( z ) T (100 mm ) T ( 200 mm ) = 0

or 300T + 100WG + W1z = 0 (2)

M z = 0: 2T (160 mm ) WG (100 mm ) W1 ( x ) = 0

or 320T 100WG W1x = 0 (3)

Eliminate T by forming 100 Eq. (1) + Eq. ( 2 )

100W1 + W1z = 0

z = 100 mm 0 z 200 mm, okay

Now, 3 Eq. ( 3) 320 Eq. (1) yields

3 ( 320T ) 3 (100 )WG 3W1x 320 ( 3T ) + 320WG + 320W1 = 0


PROBLEM 4.103 CONTINUED

or 20WG + ( 320 3x )W1 = 0

W1 20
or =
WG ( 3 x 320 )

W1
The smallest value of will result in the smallest value of W1 since WG is given.
WG

Use x = xmax = 200 mm

W1 20 1
and then = =
WG 3 ( 200 ) 320 14

WG 245.25 N
W1 = = = 17.5179 N ( minimum )
14 14
W1 17.5179 N
and m= = = 1.78571 kg
g 9.81 m/s 2
or m = 1.786 kg
at x = 200 mm, z = 100 mm
PROBLEM 4.104
A camera of mass 240 g is mounted on a small tripod of mass 200 g.
Assuming that the mass of the camera is uniformly distributed and that
the line of action of the weight of the tripod passes through D, determine
(a) the vertical components of the reactions at A, B, and C when = 0,
(b) the maximum value of if the tripod is not to tip over.

SOLUTION

First note ( )
WC = mC g = ( 0.24 kg ) 9.81 m/s 2 = 2.3544 N

Wtp = mtp g = ( 0.20 kg ) ( 9.81 m/s ) = 1.9620 N


2

For = 0 xC = ( 60 mm 24 mm ) = 36 mm

zC = 0
(a) From f.b.d. of camera and tripod as projected onto plane ABCD
Fy = 0: Ay + By + C y WC Wtp = 0

Ay + By + C y = 2.3544 N + 1.9620 N = 4.3164 N (1)

M x = 0: C y ( 38 mm ) By ( 38 mm ) = 0 C y = By (2)

M z = 0: By ( 35 mm ) + C y ( 35 mm ) + ( 2.3544 N ) ( 36 mm ) Ay ( 45 mm ) = 0

9 Ay 7By 7C y = 16.9517 (3)

Substitute C y with By from Equation (2) into Equations (1) and (3), and solve by elimination

(
7 Ay + 2 By = 4.3164 )
9 Ay 14By = 16.9517

16 Ay = 47.166
PROBLEM 4.104 CONTINUED
Ay = 2.9479 N

or A y = 2.95 N

Substituting Ay = 2.9479 N into Equation (1)

2.9479 N + 2 By = 4.3164

By = 0.68425 N

C y = 0.68425 N

or B y = C y = 0.684 N

(b) By = 0 for impending tipping

From f.b.d. of camera and tripod as projected onto plane ABCD


Fy = 0: Ay + C y WC Wtp = 0

Ay + C y = 4.3164 N (1)

M x = 0: C y ( 38 mm ) ( 2.3544 N ) ( 36 mm ) sin = 0

C y = 2.2305sin (2)

M z = 0: C y ( 35 mm ) Ay ( 45 mm ) + ( 2.3544 N ) ( 36 mm ) cos = 0

9 Ay 7C y = (16.9517 N ) cos (3)

Forming 7 Eq. (1) + Eq. ( 3) yields

16 Ay = 30.215 N + (16.9517 N ) cos (4)


PROBLEM 4.104 CONTINUED
Substituting Equation (2) into Equation (3)
9 Ay (15.6134 N ) sin = (16.9517 N ) cos (5)

Forming 9 Eq. ( 4 ) 16 Eq. ( 5 ) yields

( 249.81 N ) sin = 271.93 N (118.662 N ) cos

cos 2 = 2.2916 N ( 2.1053 N ) sin


2
or

Now cos 2 = 1 sin 2

5.4323sin 2 9.6490sin + 4.2514 = 0


Using quadratic formula to solve,
sin = 0.80981 and sin = 0.96641
= 54.078 and = 75.108
or max = 54.1 before tipping
PROBLEM 4.105
Two steel pipes AB and BC, each having a weight per unit length of
5 lb/ft, are welded together at B and are supported by three wires.
Knowing that a = 1.25 ft, determine the tension in each wire.

SOLUTION

First note WAB = ( 5 lb/ft )( 2 ft ) = 10 lb

WBC = ( 5 lb/ft )( 4 ft ) = 20 lb

W = WAB + WBC = 30 lb
To locate the equivalent force of the pipe assembly weight
rG/B W = ( ri Wi ) = rG ( AB ) WAB + rG ( BC ) WBC

or ( xGi + zGk ) ( 30 lb ) j = (1 ft ) k ( 10 lb ) j + ( 2 ft ) i ( 20 lb ) j
( 30 lb ) xGk + ( 30 lb ) zG i = (10 lb ft ) i ( 40 lb ft ) k

10 lb ft 1
From i-coefficient zG = = ft
30 lb 3
40 lb ft 1
k-coefficient xG = = 1 ft
30 lb 3
From f.b.d. of piping
M x = 0: W ( zG ) TA ( 2 ft ) = 0

1 1
TA = ft 30 lb ft = 5 lb or TA = 5.00 lb
2 3
Fy = 0: 5 lb + TD + TC 30 lb = 0

TD + TC = 25 lb (1)
PROBLEM 4.105 CONTINUED
4
M z = 0: TD (1.25 ft ) + TC ( 4 ft ) 30 lb ft = 0
3
1.25TD + 4TC = 40 lb ft (2)

4 Equation (1) 4TD 4TC = 100 (3)

Equation (2) + Equation (3) 2.75TD = 60

TD = 21.818 lb or TD = 21.8 lb

From Equation (1) TC = 25 21.818 = 3.1818 lb or TC = 3.18 lb

Results: TA = 5.00 lb

TC = 3.18 lb

TD = 21.8 lb
PROBLEM 4.106
For the pile assembly of Problem 4.105, determine (a) the largest
permissible value of a if the assembly is not to tip, (b) the corresponding
tension in each wire.
P4.105 Two steel pipes AB and BC, each having a weight per unit length
of 5 lb/ft, are welded together at B and are supported by three wires.
Knowing that a = 1.25 ft, determine the tension in each wire.

SOLUTION

First note WAB = ( 5 lb/ft )( 2 ft ) = 10 lb

WBC = ( 5 lb/ft )( 4 ft ) = 20 lb

From f.b.d. of pipe assembly

Fy = 0: TA + TC + TD 10 lb 20 lb = 0

TA + TC + TD = 30 lb (1)

M x = 0: (10 lb )(1 ft ) TA ( 2 ft ) = 0

or TA = 5.00 lb (2)

From Equations (1) and (2) TC + TD = 25 lb (3)

M z = 0: TC ( 4 ft ) + TD ( amax ) 20 lb ( 2 ft ) = 0

or ( 4 ft ) TC + TD amax = 40 lb ft (4)
PROBLEM 4.106 CONTINUED

Using Equation (3) to eliminate TC

4 ( 25 TD ) + TD amax = 40

60
or amax = 4
TD

By observation, a is maximum when TD is maximum. From Equation (3), (TD )max occurs when TC = 0.
Therefore, (TD )max = 25 lb and

60
amax = 4
25

= 1.600 ft

Results: (a) amax = 1.600 ft

(b) TA = 5.00 lb

TC = 0

TD = 25.0 lb
PROBLEM 4.107
A uniform aluminum rod of weight W is bent into a circular ring of radius
R and is supported by three wires as shown. Determine the tension in
each wire.

SOLUTION
From f.b.d. of ring

Fy = 0: TA + TB + TC W = 0

TA + TB + TC = W (1)

M x = 0: TA ( R ) TC ( R sin 30 ) = 0

TA = 0.5TC (2)

M z = 0: TC ( R cos 30 ) TB ( R ) = 0

TB = 0.86603TC (3)

Substituting TA and TB from Equations (2) and (3) into Equation (1)

0.5TC + 0.86603TC + TC = W

TC = 0.42265W

From Equation (2)

TA = 0.5 ( 0.42265W ) = 0.21132W

From Equation (3)

TB = 0.86603 ( 0.42265W ) = 0.36603W

or TA = 0.211W

TB = 0.366W

TC = 0.423W
PROBLEM 4.108
A uniform aluminum rod of weight W is bent into a circular ring of radius
R and is supported by three wires as shown. A small collar of weight W
is then placed on the ring and positioned so that the tensions in the three
wires are equal. Determine (a) the position of the collar, (b) the value of
W , (c) the tension in the wires.

SOLUTION
Let = angle from x-axis to small collar of weight W
From f.b.d. of ring
Fy = 0: 3T W W = 0 (1)

M x = 0: T ( R ) T ( R sin 30 ) + W ( R sin ) = 0

1
or W sin = T (2)
2
M z = 0: T ( R cos 30 ) W ( R cos ) T ( R ) = 0

3
or W cos = 1 T (3)
2

Dividing Equation (2) by Equation (3)
1
1 3
tan = 1 = 3.7321
2 2

= 75.000 and = 255.00


Based on Equations (2) and (3), = 75.000 will give a negative value
for W , which is not acceptable.
(a) W is located at = 255 from the x-axis or 15 from A
towards B.
(b) From Equation (1) and Equation (2)
W = 3 ( 2W )( sin 255 ) W

W = 0.20853W
or W = 0.209W
(c) From Equation (1)
T = 2 ( 0.20853W ) sin 255

= 0.40285W
or T = 0.403W
PROBLEM 4.109
An opening in a floor is covered by a 3 4-ft sheet of plywood weighing
12 lb. The sheet is hinged at A and B and is maintained in a position
slightly above the floor by a small block C. Determine the vertical
component of the reaction (a) at A, (b) at B, (c) at C.

SOLUTION

From f.b.d. of plywood sheet

M x = 0: (12 lb )( 2 ft ) C y ( 3.5 ft ) = 0
C y = 6.8571 lb or C y = 6.86 lb

M B( z -axis ) = 0: (12 lb )(1 ft ) + ( 6.8571 lb )( 0.5 ft ) Ay ( 2 ft ) = 0

Ay = 7.7143 lb or Ay = 7.71 lb

M A( z -axis ) = 0: (12 lb )(1 ft ) + By ( 2 ft ) + ( 6.8571 lb )( 2.5 ft ) = 0

By = 2.5714 lb or By = 2.57 lb

(a) Ay = 7.71 lb

(b) By = 2.57 lb

(c) C y = 6.86 lb
PROBLEM 4.110
Solve Problem 4.109 assuming that the small block C is moved and
placed under edge DE at a point 0.5 ft from corner E.

SOLUTION

First, rB/ A = ( 2 ft ) i

rC/ A = ( 2 ft ) i + ( 4 ft ) k

rG/ A = (1 ft ) i + ( 2 ft ) k

From f.b.d. of plywood sheet

M A = 0: rB/ A ( B y j + Bzk ) + rC/ A C y j + rG/ A ( Wj) = 0

( 2 ft ) i By j + ( 2 ft ) i Bzk + [( 2 ft ) i + ( 4 ft ) k ] C y j

+ [(1 ft ) i + ( 2 ft ) k ] ( 12 lb ) j = 0

2Byk 2 Bz j + 2C yk 4C y i 12k + 24i = 0

i-coeff. 4C y + 24 = 0 C y = 6.00 lb

j-coeff. 2 Bz = 0 Bz = 0

k-coeff. 2By + 2C y 12 = 0

or 2By + 2 ( 6 ) 12 = 0 By = 0
PROBLEM 4.110 CONTINUED

F = 0: Ay j + Azk + By j + Bzk + C y j Wj = 0

Ay j + Az k + 0 j + 0k + 6 j 12 j = 0

j-coeff. Ay + 6 12 = 0 Ay = 6.00 lb

k-coeff. Az = 0 Az = 0

a) Ay = 6.00 lb

b) By = 0

c) C y = 6.00 lb
PROBLEM 4.111
The 10-kg square plate shown is supported by three vertical wires.
Determine (a) the tension in each wire when a = 100 mm, (b) the value
of a for which tensions in the three wires are equal.

SOLUTION
First note ( )
W = mg = (10 kg ) 9.81 m/s 2 = 98.1 N

(a) (a) From f.b.d. of plate

Fy = 0: TA + TB + TC W = 0

TA + TB + TC = 98.1 N (1)

M x = 0: W (150 mm ) TB( 300 mm ) TC (100 mm ) = 0

6TB + 2TC = 294.3 (2)

M z = 0: TB(100 mm ) + TC ( 300 mm ) ( 98.1 N )(150 mm ) = 0

6TB 18TC = 882.9 (3)

Equation (2) + Equation (3)

16TC = 588.6

TC = 36.788 N

or TC = 36.8 N W

Substitution into Equation (2)

6TB + 2 ( 36.788 N ) = 294.3 N

TB = 36.788 N or TB = 36.8 N W

From Equation (1)

TA + 36.788 + 36.788 = 98.1 N

TA = 24.525 N or TA = 24.5 N W
PROBLEM 4.111 CONTINUED

(b) (b) From f.b.d. of plate

Fy = 0: 3T W = 0

1
T = W (1)
3

M x = 0: W (150 mm ) T ( a ) T ( 300 mm ) = 0

150W
T = (2)
a + 300

Equating Equation (1) to Equation (2)

1 150W
W =
3 a + 300

or a + 300 = 3 (150 )

or a = 150.0 mm W
PROBLEM 4.112
The 3-m flagpole AC forms an angle of 30o with the z axis. It is held by
a ball-and-socket joint at C and by two thin braces BD and BE. Knowing
that the distance BC is 0.9 m, determine the tension in each brace and the
reaction at C.

SOLUTION

TBE can be found from M about line CE


From f.b.d. of flagpole

( )
M CE = 0: CE rB/C TBD + CE ( rA/C FA ) = 0

( 0.9 m ) i + ( 0.9 m ) j 1
where CE = = ( i + j)
( 0.9 )2 + ( 0.9 )2 m 2

rB/C = ( 0.9 m ) sin 30 j + ( 0.9 m ) cos 30 k

= ( 0.45 m ) j + ( 0.77942 m ) k

( 0.9 m ) i + 0.9 m ( 0.9 m ) sin 30 j ( 0.9 m ) cos 30 k


TBD = BDTBD = T
BD
( 0.9 ) + ( 0.45 ) + ( 0.77942 ) m
2 2 2

T
= ( 0.9 m ) i + ( 0.45 m ) j ( 0.77942 m ) k BD
1.62
= ( 0.70711i + 0.35355 j 0.61237k ) TBD

rA/C = ( 3 m ) sin 30 j + ( 3 m ) cos30k = (1.5 m ) j + ( 2.5981 m ) k

FA = ( 300 N ) j

1 1 0 1 1 0
TBD 1
0 0.45 0.77942 + 0 1.5 2.5981 =0
2 2
0.70711 0.35355 0.61237 0 300 0
PROBLEM 4.112 CONTINUED

or 1.10227TBD + 779.43 = 0

TBD = 707.12 N or TBD = 707 N W

Based on symmetry with yz-plane, TBE = TBD = 707.12 N or TBE = 707 N W


The reaction forces at C are found from F = 0
Fx = 0: (TBD ) x + (TBE ) x + C x = 0 or Cx = 0

Fy = 0: (TBD ) y + (TBE ) y + C y 300 N = 0


C y = 300 N 2 ( 0.35355)( 707.12 N )

C y = 200.00 N

Fz = 0: Cz (TBD ) z (TBE ) z = 0

Cz = 2 ( 0.61237 )( 707.12 N )

C z = 866.04 N

or C = ( 200 N ) j + ( 866 N ) k W
PROBLEM 4.113
A 3-m boom is acted upon by the 4-kN force shown. Determine the
tension in each cable and the reaction at the ball-and-socket joint at A.

SOLUTION

From f.b.d. of boom

( ) (
M AE = 0: AE rB/ A TBD + AE rC/ A FC = 0 )
where AE =
( 2.1 m ) j (1.8 m ) k
( 2.1)2 + (1.8)2 m
= 0.27451j 0.23529k

rB/ A = (1.8 m ) i

TBD = BDTBD =
( 1.8 m ) i + ( 2.1 m ) j + (1.8 m ) k T
BD
(1.8)2 + ( 2.1)2 + (1.8)2 m
= ( 0.54545i + 0.63636 j + 0.54545k ) TBD

rC/ A = ( 3.0 m ) i

FC = ( 4 kN ) j
PROBLEM 4.113 CONTINUED

0 0.27451 0.23529 0 0.27451 0.23529


1.8 0 0 TBD + 3 0 0 =0
0.54545 0.63636 0.54545 0 4 0

( 0.149731 0.149729 )1.8TBD + 2.82348 = 0


TBD = 5.2381 kN or TBD = 5.24 kN W

Based on symmetry, TBE = TBD = 5.2381 kN or TBE = 5.24 kN W

Fz = 0: Az + (TBD ) z (TBE ) z = 0 Az = 0

Fy = 0: Ay + (TBD ) y + (TBD ) y 4 kN = 0

Ay + 2 ( 0.63636 )( 5.2381 kN ) 4 kN = 0

Ay = 2.6666 kN

Fx = 0: Ax (TBD ) x (TBE ) x = 0

Ax 2 ( 0.54545 )( 5.2381 kN ) = 0

Ax = 5.7142 kN

and A = ( 5.71 N ) i ( 2.67 N ) j W


PROBLEM 4.114
An 8-ft-long boom is held by a ball-and-socket joint at C and by two
cables AD and BE. Determine the tension in each cable and the reaction
at C.

SOLUTION

From f.b.d. of boom

( )
M CE = 0: CE rA/C TAD + CE rA/C FA = 0 ( )
( 2 ft ) j ( 3 ft ) k 1
where CE = = ( 2 j 3k )
( 2 )2 + ( 3)2 ft 13

rA/C = ( 8 ft ) i

( 8 ft ) i + (1 ft ) j + ( 4 ft ) k
TAD = ADTAD = TAD
(8) + (1) + ( 4 ) ft
2 2 2

1
= TAD ( 8i + j + 4k )
9

FA = (198 lb ) j

0 2 3 0 2 3
TAD 198
8 0 0 + 8 0 0 =0
9 13 13
8 1 4 0 1 0
PROBLEM 4.114 CONTINUED

TAD 198
( 64 24 ) + ( 24 ) =0
9 13 13
TAD = 486.00 lb

or TAD = 486 lb W

( )
M CD = 0: CD rB/C TBE + CD rA/C FA ( )
(1 ft ) j + ( 4 ft ) k 1
where CD = = (1j + 4k )
17 ft 17

rB/C = ( 6 ft ) i

( 6 ft ) i + ( 2 ft ) j ( 3 ft ) k 1
TBE = BETBE = TBE = TBE ( 6i + 2 j 3k )
(6) 2
+ ( 2 ) + ( 3) ft
2 2 7

0 1 4 0 1 4
TBE 198
6 0 0 + 8 0 0 =0
7 17 17
6 2 3 0 1 0

TBE
(18 + 48) + ( 32 )198 = 0
7
TBE = 672.00 lb

or TBE = 672 lb W

Fx = 0: C x (TAD ) x (TBE ) x = 0

8 6
Cx 486 672 = 0

9 7
C x = 1008 lb

Fy = 0: C y + (TAD ) y + (TBE ) y 198 lb = 0

1 2
C y + 486 + 672 198 lb = 0
9 7
C y = 48.0 lb

Fz = 0: Cz + (TAD ) z (TBE ) z = 0

4 3
Cz + 486 ( 672 ) = 0

9 7
Cz = 72.0 lb

or C = (1008 lb ) i ( 48.0 lb ) j + ( 72.0 lb ) k W


PROBLEM 4.115
Solve Problem 4.114 assuming that the given 198-lb load is replaced
with two 99-lb loads applied at A and B.
P4.114 An 8-ft-long boom is held by a ball-and-socket joint at C and by
two cables AD and BE. Determine the tension in each cable and the
reaction at C.

SOLUTION

From f.b.d. of boom

( ) ( )
M CE = 0: CE rA/C TAD + CE rA/C FA + CE rB/C FB = 0 ( )
( 2 ft ) j ( 3 ft ) k 1
where CE = = ( 2 j 3k )
( 2 )2 + ( 3)2 ft 13

rA/C = ( 8 ft ) i

rB/C = ( 6 ft ) i

( 8 ft ) i + (1 ft ) j + ( 4 ft ) k
TAD = ADTAD = TAD
(8) + (1) + ( 4 ) ft
2 2 2

1
= TAD ( 8i + j + 4k )
9

FA = ( 99 lb ) j

FB = ( 99 lb ) j

0 2 3 0 2 3 0 2 3
TAD 99 99
8 0 0 + 8 0 0 + 6 0 0 =0
9 13 13 13
8 1 4 0 1 0 0 1 0
PROBLEM 4.115 CONTINUED

TAD 99
( 64 24 ) + ( 24 + 18 ) =0
9 13 13
or TAD = 425.25 lb

or TAD = 425 lb W

( ) ( )
M CD = 0: CD rB/C TBE + CD rA/C FA + CD rB/C FB = 0 ( )
(1 ft ) j + ( 4 ft ) k 1
where CD = = ( j + 4k )
17 17

rB/C = ( 6 ft ) i

rA/C = ( 8 ft ) j

( 6 ft ) i + ( 2 ft ) j ( 3 ft ) k TBE
TBE = BETBE = TBE = ( 6i + 2 j 3k )
(6) 2
+ ( 2 ) + ( 3) ft
2 2 7

0 1 4 0 1 4 0 1 4
T 99 99
6 0 0 BE + 8 0 0 + 6 0 0 =0
7 17 17 17
6 2 3 0 1 0 0 1 0

(18 + 48)
TBE 99
+ ( 32 24 ) =0
7 17 17
or TBE = 588.00 lb

or TBE = 588 lb W

Fx = 0: C x (TAD ) x (TBE ) x = 0

8 6
Cx 425.25 588.00 = 0
9 7
C x = 882 lb

Fy = 0: C y + (TAD ) y + (TBE ) y 99 99 = 0

1 2
C y + 425.25 + 588.00 198 = 0

9 7
C y = 17.25 lb

Fz = 0: Cz + (TAD ) z (TBE ) z = 0

4 3
Cz + 425.25 588.00 = 0
9 7
Cz = 63.0 lb

or C = ( 882 lb ) i (17.25 lb ) j + ( 63.0 lb ) k W


PROBLEM 4.116
The 18-ft pole ABC is acted upon by a 210-lb force as shown. The pole is
held by a ball-and-socket joint at A and by two cables BD and BE. For
a = 9 ft, determine the tension in each cable and the reaction at A.

SOLUTION

From f.b.d. of pole ABC

( )
M AE = 0: AE rB/ A TBD + AE rC/ A FC = 0 ( )
( 4.5 ft ) i + ( 9 ft ) k 1
where AE = = ( 4.5i + 9k )
( 4.5 )2 + ( 9 )2 ft 101.25

rB/ A = ( 9 ft ) j

rC/ A = (18 ft ) j

TBD = BDTBD =
( 4.5 ft ) i ( 9 ft ) j ( 9 ft ) k T
BD
( 4.5)2 + ( 9 )2 + ( 9 )2 ft
T
= BD ( 4.5i 9 j 9k )
13.5
9i 18 j + 6k
FC = CF ( 210 lb ) = ( 210 lb ) = 10 lb ( 9i 18j + 6k )
( 9 )2 + (18)2 + ( 6 )2
4.5 0 9 4.5 0 9
TBD 10 lb
0 9 0 + 0 18 0 =0
13.5 101.25 101.25
4.5 9 9 9 18 6
PROBLEM 4.116 CONTINUED

( 364.5 364.5) T +
( 486 + 1458) (10 lb ) = 0
BD
13.5 101.25 101.25
and TBD = 360.00 lb

or TBD = 360 lb

( )
M AD = 0: AD rB/ A TBE + AD rC/ A FC = 0 ( )
( 4.5 ft ) i ( 9 ft ) k 1
where AD = = ( 4.5i 9k )
( 4.5 )2 + ( 9 )2 ft 101.25

rB/ A = ( 9 ft ) j

rC/ A = (18 ft ) j

( 4.5 ft ) i ( 9 ft ) j + ( 9 ft ) k T TBE
TBE = BETBE = BE = ( 4.5i 9 j + 9k )
( 4.5 ) + ( 9 ) + ( 9 ) ft
2 2 2 13.5

4.5 0 9 4.5 0 9
TBE 10 lb
0 9 0 + 0 18 0 =0
13.5 101.25 101.25
4.5 9 9 9 18 6

( 364.5 + 364.5 ) T +
( 486 1458 )10 lb =0
BE
13.5 101.25 101.25
or TBE = 180.0 lb

or TBE = 180.0 lb

Fx = 0: Ax + (TBD ) x + (TBE ) x ( FC ) x = 0

4.5 4.5 9
Ax + 360 + 180 210 = 0
13.5 13.5 21
Ax = 90.0 lb

Fy = 0: Ay (TBD ) y (TBE ) y ( FC ) y = 0

9 9 18
Ay 360 180 210 = 0
13.5 13.5 21
Ay = 540 lb

Fz = 0: Az (TBD ) z + (TBE ) z + ( FC ) z = 0

9 9 6
Az 360 + 180 + 210 = 0
13.5 13.5 21
Az = 60.0 lb

or A = ( 90.0 lb ) i + ( 540 lb ) j + ( 60.0 lb ) k


PROBLEM 4.117
Solve Problem 4.116 for a = 4.5 ft.
P4.116 The 18-ft pole ABC is acted upon by a 210-lb force as shown. The
pole is held by a ball-and-socket joint at A and by two cables BD and BE.
For a = 9 ft, determine the tension in each cable and the reaction at A.

SOLUTION

From f.b.d. of pole ABC

( ) (
M AE = 0 : AE rB/ A TBD + AE rC/ A FC = 0 )
( 4.5 ft ) i + ( 9 ft ) k 1
where AE = = ( 4.5i + 9k )
( 4.5 )2 + ( 9 )2 ft 101.25

rB/ A = ( 9 ft ) j

rC/ A = (18 ft ) j

TBD = BDTBD =
( 4.5 ft ) i ( 9 ft ) j ( 9 ft ) k T
BD
( 4.5)2 + ( 9 )2 + ( 9 )2 ft
T
= BD ( 4.5i 9 j 9k )
13.5
4.5i 18 j + 6k
FC = CF ( 210 lb ) = ( 210 lb )
( 4.5)2 + (18)2 + ( 6 )2
210 lb
= ( 4.5i 18 j + 6k )
19.5
4.5 0 9 4.5 0 9
TBD 210 lb
0 9 0 + 0 18 0 =0
13.5 101.25 19.5 101.25
4.5 9 9 4.5 18 6
PROBLEM 4.117 CONTINUED

( 364.5 364.5) T ( 486 + 729 )


BD + ( 210 lb ) = 0
13.5 101.25 19.5 101.25
or TBD = 242.31 lb or TBD = 242 lb

( )
M AD = 0: AD rB/ A TBE + AD rC/ A FC = 0 ( )
( 4.5 ft ) i ( 9 ft ) k 1
where AD = = ( 4.5i 9k ) ,
( 4.5 )2 + ( 9 )2 ft 101.25

rB/ A = ( 9 ft ) j

rC/ A = (18 ft ) j

( 4.5 ft ) i ( 9 ft ) j + ( 9 ft ) k T TBE
TBE = BETBE = BE = ( 4.5i 9 j + 9k )
( 4.5 ) + ( 9 ) + ( 9 ) ft
2 2 2 13.5

4.5 0 9 4.5 0 9
TBE 210 lb
0 9 0 + 0 18 0 =0
13.5 101.25 19.5 101.25
4.5 9 9 4.5 18 6

( 364.5 + 364.5 ) T +
( 486 729 )( 210 lb ) =0
BE
13.5 101.25 19.5 101.25
or TBE = 48.462 lb

or TBE = 48.5 lb

Fx = 0: Ax + (TBD ) x + (TBE ) x ( FC ) x = 0

4.5 4.5 4.5


Ax + 242.31 + 48.462 210 = 0
13.5 13.5 19.5
Ax = 48.459 lb

Fy = 0: Ay (TBD ) y (TBE ) y ( FC ) y = 0

9 9 18
Ay 242.31 48.462 210 =
13.5 13.5 19.5
Ay = 387.69 lb

Fz = 0: Az (TBD ) z + (TBE ) z + ( FC ) z = 0

9 9 6
Az 242.31 + 48.462 + 2
13.5 13.5 19.5
Az = 64.591 lb

or A = ( 48.5 lb ) i + ( 388 lb ) j + ( 64.6 lb ) k


PROBLEM 4.118
Two steel pipes ABCD and EBF are welded together at B to form the
boom shown. The boom is held by a ball-and-socket joint at D and by
two cables EG and ICFH; cable ICFH passes around frictionless pulleys
at C and F. For the loading shown, determine the tension in each cable
and the reaction at D.

SOLUTION

From f.b.d. of boom

( ) (
M z = 0: k rC/D TCI + k rA/D FA = 0 )
where rC/D = (1.8 m ) i

(1.8 m ) i + (1.12 m ) j
TCI = CI TCI = TCI
(1.8) + (1.12 ) m
2 2

T
= CI ( 1.8i + 1.12 j)
2.12

rA/D = ( 3.5 m ) i

FA = ( 560 N ) j

0 0 1 0 0 1
TCI
M z = 1.8 0 0 + 3.5 0 0 ( 560 N ) = 0
2.12
1.8 1.12 0 0 1 0

TCI
( 2.016 ) + ( 3.5 ) 560 = 0
2.12
or TCI = TFH = 2061.1 N

TICFH = 2.06 kN
PROBLEM 4.118 CONTINUED

( )
M y = 0: j rG/D TEG + j rH /D TFH = 0 ( )
where rG/D = ( 3.4 m ) k

rH /D = ( 2.5 m ) k

( 3.0 m ) i + ( 3.15 m ) k T
TEG = TEG = EG ( 3i + 3.15k )
( 3)2 + ( 3.15)2 m 4.35

( 3.0 m ) i ( 2.25 m ) k 2061.1 N


TFH = FH TFH = ( 2061.1 N ) = ( 3i 2.25k )
( 3) 2
+ ( 2.25 ) m
2 3.75

0 1 0 0 1 0
TEG 2061.1 N
0 0 3.4 + 0 0 2.5 =0
4.35 3.75
3 0 3.15 3 0 2.25

TEG 2061.1 N
(10.2 ) + ( 7.5 ) =0
4.35 3.75
or TEG = 1758.00 N

TEG = 1.758 kN

Fx = 0: Dx (TCI ) x (TFH ) x (TEG ) x = 0

1.8 ( 3.0 ( 3 (
Dx 2061.1 N ) 2061.1 N ) 1758 N ) = 0
2.12 3.75 4.35
Dx = 4611.3 N

Fy = 0: Dy + (TCI ) y 560 N = 0

1.12 (
Dy + 2061.1 N ) 560 N = 0
2.12
D y = 528.88 N

Fz = 0: Dz + (TEG ) z (TFH ) z = 0

3.15 ( 2.25 (
Dz + 1758 N ) 2061.1 N ) = 0
4.35 3.75
Dz = 36.374 N

and D = ( 4610 N ) i ( 529 N ) j ( 36.4 N ) k


PROBLEM 4.119
Solve Problem 4.118 assuming that the 560-N load is applied at B.
P4.118 Two steel pipes ABCD and EBF are welded together at B to form
the boom shown. The boom is held by a ball-and-socket joint at D and by
two cables EG and ICFH; cable ICFH passes around frictionless pulleys
at C and F. For the loading shown, determine the tension in each cable
and the reaction at D.

SOLUTION

From f.b.d. of boom

( ) (
M z = 0: k rC/D TCI + k rB/D FB = 0 )
where rC/D = (1.8 m ) i

(1.8 m ) i + (1.12 m ) j
TCI = CI TCI = TCI
(1.8)2 + (1.12 )2 m

T
= CI ( 1.8i + 1.12 j)
2.12

rB/D = ( 3.0 m ) i

FB = ( 560 N ) j

0 0 1 0 0 1
TCI
1.8 0 0 + 3 0 0 ( 560 N ) = 0
2.12
1.8 1.12 0 0 1 0

TCI
( 2.016 ) + ( 3) 560 = 0
2.12
or TCI = TFH = 1766.67 N

TICFH = 1.767 kN
PROBLEM 4.119 CONTINUED

( )
M y = 0: j rG/D TEG + j rH /D TFH = 0( )
where rG/D = ( 3.4 m ) k

rH /D = ( 2.5 m ) k

( 3.0 m ) i + ( 3.15 m ) k TEG


TEG = EGTEG = TEG = ( 3i + 3.15k )
( 3)2 + ( 3.15)2 m 4.35

( 3.0 m ) i ( 2.25 m ) k 1766.67 N


TFH = FH TFH = TFH = ( 3i 2.25k )
( 3) 2
+ ( 2.25 ) m
2 3.75

0 1 0 0 1 0
TEG 1766.67
0 0 3.4 + 0 0 2.5 =0
4.35 3.75
3 0 3.15 3 0 2.25

TEG 1766.67
(10.2 ) + ( 7.5 ) =0
4.35 3.75
or TEG = 1506.86 N

TEG = 1.507 kN

Fx = 0: Dx (TCI ) x (TFH ) x (TEG ) x = 0

1.8 ( 3 ( 3 (
Dx 1766.67 N ) 1766.67 N ) 1506.86 N ) = 0
2.12 3.75 4.35
Dx = 3952.5 N

Fy = 0: Dy + (TCI ) y 560 N = 0

1.12 (
Dy + 1766.67 N ) 560 N = 0
2.12
D y = 373.34 N

Fz = 0: Dz + (TEG ) z (TFH ) z = 0

3.15 ( 2.25 (
Dz + 1506.86 N ) 1766.67 N ) = 0
4.35 3.75
Dz = 31.172 N

D = ( 3950 N ) i ( 373 N ) j ( 31.2 N ) k


PROBLEM 4.120
The lever AB is welded to the bent rod BCD which is supported by
bearings at E and F and by cable DG. Knowing that the bearing at E does
not exert any axial thrust, determine (a) the tension in cable DG, (b) the
reactions at E and F.

SOLUTION

(a) From f.b.d. of assembly


( 0.12 m ) j ( 0.225 m ) k TDG
TDG = DGTDG = = 0.255 ( 0.12 ) j ( 0.225 ) k
( 0.12 ) + ( 0.225 ) m
2 2

0.225
M y = 0: ( 220 N )( 0.24 m ) + TDG ( 0.16 m ) = 0
0.255

TDG = 374.00 N

or TDG = 374 N
(b) From f.b.d. of assembly
0.120
M F ( z -axis ) = 0: ( 220 N )( 0.19 m ) Ex ( 0.13 m ) 374 N ( 0.16 m ) = 0
0.255

Ex = 104.923 N

Fx = 0: Fx + 104.923 N 220 N = 0

Fx = 115.077 N

0.225
M F ( x-axis ) = 0: Ez ( 0.13 m ) + 374 N ( 0.06 m ) = 0
0.255

Ez = 152.308 N
PROBLEM 4.120 CONTINUED

0.225
Fz = 0: Fz 152.308 N ( 374 N ) =0
0.255
Fz = 482.31 N

0.12
Fy = 0: Fy ( 374 N ) =0
0.255
Fy = 176.0 N

E = (104.9 N ) i (152.3 N ) k

F = (115.1 N ) i + (176.0 N ) j + ( 482 N ) k


PROBLEM 4.121
A 30-kg cover for a roof opening is hinged at corners A and B. The roof
forms an angle of 30o with the horizontal, and the cover is maintained in
a horizontal position by the brace CE. Determine (a) the magnitude of the
force exerted by the brace, (b) the reactions at the hinges. Assume that the
hinge at A does not exert any axial thrust.

SOLUTION

First note ( )
W = mg = ( 30 kg ) 9.81 m/s2 = 294.3 N

FEC = EC FEC = ( sin15 ) i + ( cos15 ) j FEC

From f.b.d. of cover


(a) M z = 0: ( FEC cos15 ) (1.0 m ) W ( 0.5 m ) = 0
or FEC cos15 (1.0 m ) ( 294.3 N )( 0.5 m ) = 0

FEC = 152.341 N or FEC = 152.3 N

(b) M x = 0: W ( 0.4 m ) Ay ( 0.8 m ) ( FEC cos15 ) ( 0.8 m ) = 0

or ( 294.3 N )( 0.4 m ) Ay ( 0.8 m ) (152.341 N ) cos15 ( 0.8 m ) = 0


Ay = 0

M y = 0: Ax ( 0.8 m ) + ( FEC sin15 ) ( 0.8 m ) = 0

or Ax ( 0.8 m ) + (152.341 N ) sin15 ( 0.8 m ) = 0

Ax = 39.429 N

Fx = 0: Ax + Bx + FEC sin15 = 0

39.429 N + Bx + (152.341 N ) sin15 = 0

Bx = 0
PROBLEM 4.121 CONTINUED

Fy = 0: FEC cos15 W + By = 0

or (152.341 N ) cos15 294.3 N + By =0

By = 147.180 N

or A = ( 39.4 N ) i

B = (147.2 N ) j
PROBLEM 4.122
The rectangular plate shown has a mass of 15 kg and is held in the
position shown by hinges A and B and cable EF. Assuming that the hinge
at B does not exert any axial thrust, determine (a) the tension in the cable,
(b) the reactions at A and B.

SOLUTION

First note ( )
W = mg = (15 kg ) 9.81 m/s 2 = 147.15 N

( 0.08 m ) i + ( 0.25 m ) j ( 0.2 m ) k TEF


TEF = EFTEF = TEF = 0.33 ( 0.08i + 0.25j 0.2k )
( 0.08 ) + ( 0.25) + ( 0.2 ) m
2 2 2

From f.b.d. of rectangular plate


M x = 0: (147.15 N )( 0.1 m ) (TEF ) y ( 0.2 m ) = 0
0.25
or 14.715 N m TEF ( 0.2 m ) = 0
0.33
or TEF = 97.119 N

or TEF = 97.1 N

Fx = 0: Ax + (TEF ) x = 0

0.08
Ax + ( 97.119 N ) = 0
0.33
Ax = 23.544 N
PROBLEM 4.122 CONTINUED

M B( z -axis ) = 0: Ay ( 0.3 m ) (TEF ) y ( 0.04 m ) + W ( 0.15 m ) = 0

0.25
or Ay ( 0.3 m ) 97.119 N ( 0.04 m ) + 147.15 N ( 0.15 m ) = 0
0.33
Ay = 63.765 N

M B( y -axis ) = 0: Az ( 0.3 m ) + (TEF ) x ( 0.2 m ) + (TEF ) z ( 0.04 m ) = 0

0.08 0.2
Az ( 0.3 m ) + TEF ( 0.2 m ) TEF ( 0.04 m ) = 0
0.33 0.33
Az = 7.848 N

and A = ( 23.5 N ) i + ( 63.8 N ) j ( 7.85 N ) k

Fy = 0: Ay W + (TEF ) y + By = 0

0.25
63.765 N 147.15 N + ( 97.119 N ) + By = 0
0.33
By = 9.81 N

Fz = 0: Az (TEF ) z + Bz = 0

0.2
7.848 N ( 97.119 N ) + Bz = 0
0.33
Bz = 66.708 N

and B = ( 9.81 N ) j + ( 66.7 N ) k


PROBLEM 4.123
Solve Problem 4.122 assuming that cable EF is replaced by a cable
attached at points E and H.
P4.122 The rectangular plate shown has a mass of 15 kg and is held in
the position shown by hinges A and B and cable EF. Assuming that the
hinge at B does not exert any axial thrust, determine (a) the tension in the
cable, (b) the reactions at A and B.

SOLUTION

First note ( )
W = mg = (15 kg ) 9.81 m/s 2 = 147.15 N


( 0.3 m ) i + ( 0.12 m ) j ( 0.2 m ) k T
TEH = EH TEH = T = EH ( 0.3) i + ( 0.12 ) j ( 0.2 ) k
EH
( 0.3) + ( 0.12 ) + ( 0.2 ) m
2 2 2 0.38

From f.b.d. of rectangular plate


M x = 0: (147.15 N )( 0.1 m ) (TEH ) y ( 0.2 m ) = 0
0.12
or (147.15 N )( 0.1 m ) TEH ( 0.2 m ) = 0
0.38
or TEH = 232.99 N

or TEH = 233 N

Fx = 0: Ax + (TEH ) x = 0

0.3
Ax ( 232.99 N ) = 0
0.38
Ax = 183.938 N
PROBLEM 4.123 CONTINUED

M B( z -axis ) = 0: Ay ( 0.3 m ) (TEH ) y ( 0.04 m ) + W ( 0.15 m ) = 0

0.12
or Ay ( 0.3 m ) ( 232.99 N ) ( 0.04 m ) + (147.15 N )( 0.15 m ) = 0
0.38
Ay = 63.765 N

M B( y -axis ) = 0: Az ( 0.3 m ) + (TEH ) x ( 0.2 m ) + (TEH ) z ( 0.04 m ) = 0

0.3 0.2
or Az ( 0.3 m ) ( 232.99 N ) ( 0.2 m ) ( 232.99 ) ( 0.04 m ) = 0
0.38 0.38
Az = 138.976 N

and A = (183.9 N ) i + ( 63.8 N ) j + (139.0 N ) k

Fy = 0: Ay + By W + (TEH ) y = 0

0.12
63.765 N + By 147.15 N + ( 232.99 N ) = 0
0.38
By = 9.8092 N

Fz = 0: Az + Bz (TEH ) z = 0

0.2
138.976 N + Bz ( 232.99 N ) = 0
0.38
Bz = 16.3497 N

and B = ( 9.81 N ) j (16.35 N ) k


PROBLEM 4.124
A small door weighing 16 lb is attached by hinges A and B to a wall and
is held in the horizontal position shown by rope EFH. The rope passes
around a small, frictionless pulley at F and is tied to a fixed cleat at H.
Assuming that the hinge at A does not exert any axial thrust, determine
(a) the tension in the rope, (b) the reactions at A and B.

SOLUTION

First note T = EFT =


(12 in.) i + ( 54 in.) j ( 28 in.) k T
(12 )2 + ( 54 )2 + ( 28)2 in.
T T
= (12i + 54 j 28k ) = ( 6i + 27 j 14k )
62 31
W = (16 lb ) j at G

From f.b.d. of door ABCD


(a) M x = 0: Ty ( 28 in.) W (14 in.) = 0

27
T 31 ( 28 in.) (16 lb )(14 in.) = 0

T = 9.1852 lb
or T = 9.19 lb

(b) M B( z -axis ) = 0: Ay ( 30 in.) + W (15 in.) Ty ( 4 in.) = 0

27
Ay ( 30 in.) + (16 lb )(15 in.) ( 9.1852 lb ) ( 4 in.) = 0
31

Ay = 6.9333 lb
PROBLEM 4.124 CONTINUED

M B( y -axis ) = 0: Az ( 30 in.) + Tx ( 28 in.) Tz ( 4 in.) = 0

6 14
Az ( 30 in.) + ( 9.1852 lb ) ( 28 in.) ( 9.1852 lb ) ( 4 in.) = 0
31 31

Az = 1.10617 lb

or A = ( 6.93 lb ) j (1.106 lb ) k

6
Fx = 0: Bx + Tx = Bx + ( 9.1852 lb ) = 0
31
Bx = 1.77778 lb

Fy = 0: By + Ty W + Ay = 0

27
By + ( 9.1852 lb ) 16 lb + 6.9333 lb = 0
31
By = 1.06666 lb

Fz = 0: Az Tz + Bz = 0

14
1.10617 lb ( 9.1852 lb ) + Bz = 0
31
Bz = 5.2543 lb

or B = (1.778 lb ) i + (1.067 lb ) j + ( 5.25 lb ) k


PROBLEM 4.125
Solve Problem 4.124 assuming that the rope is attached to the door at I.
P4.124 A small door weighing 16 lb is attached by hinges A and B to a
wall and is held in the horizontal position shown by rope EFH. The rope
passes around a small, frictionless pulley at F and is tied to a fixed cleat
at H. Assuming that the hinge at A does not exert any axial thrust,
determine (a) the tension in the rope, (b) the reactions at A and B.

SOLUTION

First note T = IF T =
( 3 in.) i + ( 54 in.) j (10 in.) k T
( 3)2 + ( 54 )2 + (10 )2 in.
T
= ( 3i + 54 j 10k )
55
W = (16 lb ) j

From f.b.d. of door ABCD


(a) M x = 0: W (14 in.) Ty (10 in.) = 0

(16 lb )(14 in.)


54
T (10 in.) = 0
55
T = 22.815 lb
or T = 22.8 lb

(b) M B( z -axis ) = 0: Ay ( 30 in.) + W (15 in.) + Ty ( 5 in.) = 0

54
Ay ( 30 in.) + (16 lb )(15 in.) + ( 22.815 lb ) ( 5 in.) = 0
55
Ay = 11.7334 lb
PROBLEM 4.125 CONTINUED

M B( y -axis ) = 0: Az ( 30 in.) + Tx (10 in.) + Tz ( 5 in.) = 0

3 10
Az ( 30 in.) + ( 22.815 lb ) (10 in.) + ( 22.815 lb ) ( 5 in.) = 0
55 55

Az = 1.10618 lb

or A = (11.73 lb ) j (1.106 lb ) k

Fx = 0: Bx + Tx = 0

3
Bx + ( 22.815 lb ) = 0
55
Bx = 1.24444 lb

Fy = 0: Ay W + Ty + By = 0

54
11.7334 lb 16 lb + ( 22.815 lb ) + By = 0
55
By = 18.1336 lb

Fz = 0: Az Tz + Bz = 0

10
1.10618 lb ( 22.815 lb ) + Bz = 0
55
Bz = 5.2544 lb

or B = (1.244 lb ) i (18.13 lb ) j + ( 5.25 lb ) k


PROBLEM 4.126
A 285-lb uniform rectangular plate is supported in the position shown by
hinges A and B and by cable DCE, which passes over a frictionless hook
at C. Assuming that the tension is the same in both parts of the cable,
determine (a) the tension in the cable, (b) the reactions at A and B.
Assume that the hinge at B does not exert any axial thrust.

SOLUTION

( 23 in.) i + ( 22.5 in.) j (15 in.) k


First note CD =
35.5 in.
1
= ( 23i + 22.5 j 15k )
35.5

CE =
( 9 in.) i + ( 22.5 in.) j (15 in.) k
28.5 in.
1
= ( 9i + 22.5 j 15k )
28.5
W = ( 285 lb ) j

From f.b.d. of plate


22.5 22.5
(a) M x = 0: ( 285 lb )( 7.5 in.) T (15 in.) T (15 in.) = 0
35.5 28.5
T = 100.121 lb
or T = 100.1 lb
PROBLEM 4.126 CONTINUED

23 9
(b) Fx = 0: Ax T +T =0
35.5 28.5

23 ( 9
Ax (100.121 lb ) + 100.121 lb ) =0
35.5 28.5
Ax = 33.250 lb

22.5 22.5
M B( z -axis ) = 0: Ay ( 26 in.) + W (13 in.) T ( 6 in.) T 28.5 ( 6 in.) = 0
35.5

22.5
or Ay ( 26 in.) + ( 285 lb )(13 in.) (100.121 lb ) ( 6 in.)
35.5

22.5
(100.121 lb ) ( 6 in.) = 0
28.5

Ay = 109.615 lb

15 23
M B( y -axis ) = 0: Az ( 26 in.) T ( 6 in.) T 35.5 (15 in.)
35.5

15 9
T ( 6 in.) + T 28.5 (15 in.) = 0
28.5

1
( 90 135) (100.121 lb ) = 0
1
or Az ( 26 in.) + ( 90 + 345)
35.5 28.5
Az = 41.106 lb

or A = ( 33.3 lb ) i + (109.6 lb ) j + ( 41.1 lb ) k

22.5 22.5
Fy = 0: By W + T +T + Ay = 0
35.5 28.5

22.5 22.5
By 285 lb + (100.121 lb ) + + 109.615 lb = 0
35.5 28.5
By = 32.885 lb

15 15
Fz = 0: Bz + Az T T =0
35.5 28.5

15 15
Bz + 41.106 lb (100.121 lb ) + =0
35.5 28.5
Bz = 53.894 lb

or B = ( 32.9 lb ) j + ( 53.9 lb ) k
PROBLEM 4.127
Solve Problem 4.126 assuming that cable DCE is replaced by a cable
attached to point E and hook C.
P4.126 A 285-lb uniform rectangular plate is supported in the position
shown by hinges A and B and by cable DCE, which passes over a
frictionless hook at C. Assuming that the tension is the same in both parts
of the cable, determine (a) the tension in the cable, (b) the reactions at A
and B. Assume that the hinge at B does not exert any axial thrust.

SOLUTION

First note CE =
( 9 in.) i + ( 22.5 in.) j (15 in.) k
28.5 in.
1
= ( 9i + 22.5 j 15k )
28.5
W = ( 285 lb ) j

From f.b.d. of plate


22.5
(a) M x = 0: ( 285 lb )( 7.5 in.) T (15 in.) = 0
28.5
T = 180.500 lb
or T = 180.5 lb

9
(b) Fx = 0: Ax + T =0
28.5

9
Ax + 180.5 lb =0
28.5
Ax = 57.000 lb
PROBLEM 4.127 CONTINUED

22.5
M B( z -axis ) = 0: Ay ( 26 in.) + W (13 in.) T ( 6 in.) = 0
28.5

22.5
Ay ( 26 in.) + ( 285 lb )(13 in.) (180.5 lb ) ( 6 in.) = 0
28.5

Ay = 109.615 lb

15 9
M B( y -axis ) = 0: Az ( 26 in.) T ( 6 in.) + T 28.5 (15 in.) = 0
28.5

45
Az ( 26 in.) + (180.5 lb ) =0
28.5
Az = 10.9615 lb

or A = ( 57.0 lb ) i + (109.6 lb ) j (10.96 lb ) k

22.5
Fy = 0: By W + T + Ay = 0
28.5

22.5
By 285 lb + (180.5 lb ) 109.615 lb = 0
28.5
By = 32.885 lb

15
Fz = 0: Bz + Az T =0
28.5

15
Bz 10.9615 lb 180.5 lb =0
28.5
Bz = 105.962 lb

or B = ( 32.9 lb ) j + (106.0 lb ) k
PROBLEM 4.128
The tensioning mechanism of a belt drive consists of frictionless pulley
A, mounting plate B, and spring C. Attached below the mounting plate is
slider block D which is free to move in the frictionless slot of bracket E.
Knowing that the pulley and the belt lie in a horizontal plane, with
portion F of the belt parallel to the x axis and portion G forming an angle
of 30 with the x axis, determine (a) the force in the spring, (b) the
reaction at D.

SOLUTION

From f.b.d. of plate B


(a) Fx = 0: 12 N + (12 N ) cos 30 T = 0

T = 22.392 N or T = 22.4 N
(b) Fy = 0: Dy = 0

Fz = 0: Dz (12 N ) sin 30 = 0

Dz = 6 N or D = ( 6.00 N ) k

M x = 0: M Dx (12 N ) sin 30 ( 22 mm ) = 0

M Dx = 132.0 N mm

M D( y -axis ) = 0: M Dy + ( 22.392 N )( 30 mm ) (12 N )( 75 mm ) (12 N ) cos 30 ( 75 mm ) = 0

M Dy = 1007.66 N mm

M D( z -axis ) = 0: M Dz + ( 22.392 N )(18 mm ) (12 N )( 22 mm ) (12 N ) cos 30 ( 22 mm ) = 0

M Dz = 89.575 N mm

or M D = ( 0.1320 N m ) i + (1.008 N m ) j + ( 0.0896 N m ) k


PROBLEM 4.129
The assembly shown is welded to collar A which fits on the vertical pin
shown. The pin can exert couples about the x and z axes but does not
prevent motion about or along the y axis. For the loading shown,
determine the tension in each cable and the reaction at A.

SOLUTION

( 0.16 m ) i + ( 0.12 m ) j
First note TCF = CFTCF = TCF
( 0.16 ) + ( 0.12 ) m
2 2

= TCF ( 0.8i + 0.6 j)

TDE = DETDE =
( 0.24 m ) j ( 0.18 m ) k T
DE
( 0.24 )2 + ( 0.18)2 m
= TDE ( 0.8j 0.6k )

(a) From f.b.d. of assembly


Fy = 0: 0.6TCF + 0.8TDE 800 N = 0

or 0.6TCF + 0.8TDE = 800 N (1)

M y = 0: ( 0.8TCF ) ( 0.27 m ) + ( 0.6TDE )( 0.16 m ) = 0

or TDE = 2.25TCF (2)


PROBLEM 4.129 CONTINUED

Substituting Equation (2) into Equation (1)

0.6TCF + 0.8 ( 2.25 ) TCF = 800 N

TCF = 333.33 N or TCF = 333 N

and from Equation (2) TDE = 2.25 ( 333.33 N ) = 750.00 N or TDE = 750 N

(b) From f.b.d. of assembly


Fz = 0: Az ( 0.6 )( 750.00 N ) = 0 Az = 450.00 N

Fx = 0: Ax ( 0.8 )( 333.33 N ) = 0 Ax = 266.67 N

or A = ( 267 N ) i + ( 450 N ) k

M x = 0: M Ax + ( 800 N )( 0.27 m ) ( 333.33 N )( 0.6 ) ( 0.27 m ) ( 750 N )( 0.8 ) ( 0.18 m ) = 0

M Ax = 54.001 N m

M z = 0: M Az ( 800 N )( 0.16 m ) + ( 333.33 N )( 0.6 ) ( 0.16 m ) + ( 750 N )( 0.8 ) ( 0.16 m ) = 0

M Az = 0

or M A = ( 54.0 N m ) i
PROBLEM 4.130
The lever AB is welded to the bent rod BCD which is supported by
bearing E and by cable DG. Assuming that the bearing can exert an axial
thrust and couples about axes parallel to the x and z axes, determine
(a) the tension in cable DG, (b) the reaction at E.

SOLUTION

( 0.12 m ) j ( 0.225 m ) k
First note TDG = DGTDG = TDG
( 0.12 ) + ( 0.225) m
2 2

TDG
= ( 0.12 j 0.225k )
0.255
(a) From f.b.d. of weldment
0.225
M y = 0: TDG ( 0.16 m ) ( 220 N )( 0.24 m ) = 0
0.255
TDG = 374.00 N or TDG = 374 N
(b) From f.b.d. of weldment
Fx = 0: Ex 220 N = 0

Ex = 220.00 N

0.12
Fy = 0: E y ( 374.00 N ) =0
0.255
E y = 176.000 N
PROBLEM 4.130 CONTINUED

0.225
Fz = 0: Ez ( 374.00 N ) =0
0.255
Ez = 330.00 N

or E = ( 220 N ) i + (176.0 N ) j + ( 330 N ) k

M x = 0: M Ex + ( 330.00 N )( 0.19 m ) = 0

M Ex = 62.700 N m

0.12
M z = 0: ( 220 N )( 0.06 m ) + M Ez ( 374.00 N ) ( 0.16 m ) = 0
0.255

M Ez = 14.9600 N m

or M E = ( 62.7 N m ) i (14.96 N m ) k
PROBLEM 4.131
Solve Problem 4.124 assuming that the hinge at A is removed and that the
hinge at B can exert couples about the y and z axes.
P4.124 A small door weighing 16 lb is attached by hinges A and B to a
wall and is held in the horizontal position shown by rope EFH. The rope
passes around a small, frictionless pulley at F and is tied to a fixed cleat
at H. Assuming that the hinge at A does not exert any axial thrust,
determine (a) the tension in the rope, (b) the reactions at A and B.

SOLUTION
From f.b.d. of door
(a) M B = 0: rG/B W + rE/B TEF + M B = 0

where
W = (16 lb ) j

M B = M By j + M Bz k

TEF = EFTEF =
(12 in.) i + ( 54 in.) j ( 28 in.) k T
EF
(12 )2 + ( 54 )2 + ( 28)2 in.
TEF
= ( 6i + 27 j 14k )
31
rG/B = (15 in.) i + (14 in.) k

rE/B = ( 4 in.) i + ( 28 in.) k

i j k i j k

T
15 0 14 (16 lb ) + 4 0 28 EF + M By j + M Bz k = 0
31
( )
0 1 0 6 27 14

or ( 224 24.387TEF ) i + ( 3.6129TEF + M By j)


(
+ 240 3.4839TEF + M Bz k = 0 )
From i-coefficient 224 24.387TEF = 0

TEF = 9.1852 lb

or TEF = 9.19 lb W

(b) From j-coefficient 3.6129 ( 9.1852 ) + M By = 0

M By = 33.185 lb in.
PROBLEM 4.131 CONTINUED

From k-coefficient 240 3.4839 ( 9.1852 ) + M Bz = 0

M Bz = 208.00 lb in.

or M B = ( 33.2 lb in.) j ( 208 lb in.) k W

6
Fx = 0: Bx + ( 9.1852 lb ) = 0
31
Bx = 1.77778 lb

27
Fy = 0: By 16 lb + ( 9.1852 lb ) = 0
31
By = 8.0000 lb

14
Fz = 0: Bz ( 9.1852 lb ) = 0
31
Bz = 4.1482 lb

or B = (1.778 lb ) i + ( 8.00 lb ) j + ( 4.15 lb ) k W


PROBLEM 4.132
The frame shown is supported by three cables and a ball-and-socket joint
at A. For P = 0, determine the tension in each cable and the reaction at A.

SOLUTION
First note
( 0.65 m ) i + ( 0.2 m ) j ( 0.44 m ) k
TDI = DI TDI = TDI
( 0.65)2 + ( 0.2 )2 + ( 0.44 )2 m
TDI
= ( 0.65i + 0.2 j 0.44k )
0.81
( 0.45 m ) i + ( 0.24 m ) j
TEH = EH TEH = TEH
( 0.45)2 + ( 0.24 )2 m
TEH
= ( 0.45i ) + ( 0.24 j)
0.51
( 0.45 m ) i + ( 0.2 m ) j + ( 0.36 m ) k
TFG = FGTFG = TFG
( 0.45 ) 2
+ ( 0.2 ) + ( 0.36 ) m
2 2

TFG
= ( 0.45i + 0.2 j + 0.36k )
0.61
From f.b.d. of frame
M A = 0: rD/ A TDI + rC/ A ( 280 N ) j + rH / A TEH + rF / A TFG + rF / A ( 360 N ) j = 0

i j k i j k i j k i j k
TDI TEH TFG
or 0.65 0.2 0 + 0.65 0 0 ( 280 N ) + 0 0.32 0 + 0.45 0 0.06
0.81 0.51 0.61
0.65 0.2 0.44 0 1 0 0.45 0.24 0 0.45 0.2 0.36

i j k
+ 0.45 0 0.06 ( 360 N ) = 0
0 1 0

TDI T
or ( 0.088i + 0.286 j + 0.26k ) + ( 0.65k ) 280 N + ( 0.144k ) EH
0.81 0.51
TFG
+ ( 0.012i 0.189 j + 0.09k ) + ( 0.06i 0.45k )( 360 N ) = 0
0.61
PROBLEM 4.132 CONTINUED

T T
From i-coefficient 0.088 DI 0.012 FG + 0.06 ( 360 N ) = 0
0.81 0.61
0.108642TDI + 0.0196721TFG = 21.6 (1)

T T
From j-coefficient 0.286 DI 0.189 FG = 0
0.81 0.61
TFG = 1.13959TDI (2)
From k-coefficient
T T T
0.26 DI 0.65 ( 280 N ) + 0.144 EH + 0.09 FG
0.81 0.51 0.61
0.45 ( 360 N ) = 0

0.32099TDI + 0.28235TEH + 0.147541TFG = 344 N (3)


Substitution of Equation (2) into Equation (1)
0.108642TDI + 0.0196721(1.13959TDI ) = 21.6

TDI = 164.810 N

or TDI = 164.8 N W
Then from Equation (2)
TFG = 1.13959 (164.810 N ) = 187.816 N

or TFG = 187.8 N W
And from Equation (3)
0.32099 (164.810 N ) + 0.28235TEH + 0.147541(187.816 N ) = 344 N

TEH = 932.84 N

or TEH = 933 N W
The vector forms of the cable forces are:
164.810 N
TDI = ( 0.65i + 0.2 j 0.44k )
0.81
= (132.25 N ) i + ( 40.694 N ) j ( 89.526 N ) k

932.84 N
TEH = ( 0.45i + 0.24 j) = (823.09 N ) i + ( 438.98 N ) j
0.51
187.816 N
TFG = ( 0.45i + 0.2 j + 0.36k )
0.61
= (138.553 N ) i + ( 61.579 N ) j + (110.842 N ) k
PROBLEM 4.132 CONTINUED

Then, from f.b.d. of frame


Fx = 0: Ax 132.25 823.09 138.553 = 0

Ax = 1093.89 N

Fy = 0: Ay + 40.694 + 438.98 + 61.579 360 280 = 0

Ay = 98.747 N

Fz = 0: Az 89.526 + 110.842 = 0

Az = 21.316 N

or A = (1094 N ) i + ( 98.7 N ) j ( 21.3 N ) k W


PROBLEM 4.133
The frame shown is supported by three cables and a ball-and-socket joint
at A. For P = 50 N, determine the tension in each cable and the reaction
at A.

SOLUTION
First note
( 0.65 m ) i + ( 0.2 m ) j ( 0.44 m ) k
TDI = DI TDI = TDI
( 0.65)2 + ( 0.2 )2 + ( 0.44 )2 m
TDI
= ( 65i + 20 j 44k )
81
( 0.45 m ) i + ( 0.24 m ) j
TEH = EH TEH = TEH
( 0.45)2 + ( 0.24 )2 m
TEH
= ( 15i + 8j)
17
( 0.45 m ) i + ( 0.2 m ) j + ( 0.36 m ) k
TFG = FGTFG = TFG
( 0.45 ) + ( 0.2 ) + ( 0.36 ) m
2 2 2

TFG
= ( 45i + 20 j + 36k )
61
From f.b.d. of frame

M A = 0: rD/ A TDI + rC/ A ( 280 N ) j + ( 50 N ) k

+ rH / A TEH + rF / A TFG + rF / A ( 360 N ) j

i j k i j k i j k
TDI T
or 0.65 0.2 0 + 0.65 0 0 + 0 0.32 0 EH
81 17
65 20 44 0 280 50 15 8 0

i j k i j k
TFG
+ 0.45 0 0.06 + 0.45 0 0.06 ( 360 N ) = 0
61
45 20 36 0 1 0

( 8.8i + 28.6 j + 26k )


TDI TEH
and + ( 32.5 j 182k ) + ( 4.8k )
81 17

T
+ ( 1.2i 18.9 j + 9.0k ) FG + ( 0.06i 0.45k ) ( 360 ) = 0
61
PROBLEM 4.133 CONTINUED
T T
From i-coefficient 8.8 DI 1.2 FG + 0.06 ( 360 ) = 0
81 61
0.108642TDI + 0.0196721TFG = 21.6 (1)

T T
From j-coefficient 28.6 DI 32.5 18.9 FG = 0
81 61
0.35309TDI 0.30984TFG = 32.5 (2)
From k-coefficient
T T T
26 DI 182 + 4.8 EH + 9.0 FG 0.45 ( 360 ) = 0
81 17 61
0.32099TDI + 0.28235TEH + 0.147541TFG = 344 (3)

3.25 Equation (1) 0.35309TDI 0.063935TFG = 70.201

Add Equation (2) 0.35309TDI 0.30984TFG = 32.5

0.37378TFG = 37.701

TFG = 100.864 N

or TFG = 100.9 N W
Then from Equation (1)
0.108642TDI + 0.0196721(100.864 ) = 21.6

TDI = 180.554 N

or TDI = 180.6 N W
and from Equation (3)
0.32099 (180.554 ) + 0.28235TEH + 0.147541(100.864 ) = 344

TEH = 960.38 N

or TEH = 960 N W
The vector forms of the cable forces are:
180.554 N
TDI = ( 65i + 20 j 44k )
81
= (144.889 N ) i + ( 44.581 N ) j ( 98.079 N ) k

960.38 N
TEH = ( 15i + 8j) = (847.39 N ) i + ( 451.94 N ) j
17
100.864 N
TFG = ( 45i + 20 j + 36k )
61
= ( 74.409 N ) i + ( 33.070 N ) j + ( 59.527 N ) k
PROBLEM 4.133 CONTINUED

Then from f.b.d. of frame


Fx = 0: Ax 144.889 847.39 74.409 = 0

Ax = 1066.69 N

Fy = 0: Ay + 44.581 + 451.94 + 33.070 360 280 = 0

Ay = 110.409 N

Fz = 0: Az 98.079 + 59.527 + 50 = 0

Az = 11.448 N

Therefore, A = (1067 N ) i + (110.4 N ) j (11.45 N ) k W


PROBLEM 4.134
The rigid L-shaped member ABF is supported by a ball-and-socket joint
at A and by three cables. For the loading shown, determine the tension in
each cable and the reaction at A.

SOLUTION
First note
(18 in.) i + (13.5 in.) k
TBG = BGTBG = TBG
(18) 2
+ (13.5 ) in.
2

= TBG ( 0.8i + 0.6k )

(18 in.) i + ( 24 in.) j


TDH = DH TDH = TDH
(18)2 + ( 24 )2 in.
= TDH ( 0.6i + 0.8 j)

Since FJ = DH ,

TFJ = TFJ ( 0.6i + 0.8 j)

From f.b.d. of member ABF


M A( x-axis ) = 0: ( 0.8TFJ ) ( 48 in.) + ( 0.8TDH )( 24 in.) (120 lb )( 36 in.) (120 lb )(12 in.) = 0
3.2TFJ + 1.6TDH = 480 (1)

M A( z -axis ) = 0: ( 0.8TFJ ) (18 in.) + ( 0.8TDH )(18 in.) (120 lb )(18 in.) (120 lb )(18 in.) = 0
3.2TFJ 3.2TDH = 960 (2)

Equation (1) + Equation (2) TDH = 300 lb W

Substituting in Equation (1) TFJ = 0 W

M A( y -axis ) = 0: ( 0.6TFJ ) ( 48 in.) + 0.6 ( 300 lb ) ( 24 in.) ( 0.6TBG ) (18 in.) = 0


TBG = 400 lb W
PROBLEM 4.134 CONTINUED

Fx = 0: 0.6TFJ 0.6TDH 0.8TBG + Ax = 0

0.6 ( 300 lb ) 0.8 ( 400 lb ) + Ax = 0

Ax = 500 lb

Fy = 0: 0.8TFJ + 0.8TDH 240 lb + Ay = 0

0.8 ( 300 lb ) 240 + Ay = 0

Ay = 0

Fz = 0: 0.6TBG + Az = 0

0.6 ( 400 lb ) + Az = 0

Az = 240 lb

Therefore, A = ( 500 lb ) i ( 240 lb ) k W


PROBLEM 4.135
Solve Problem 4.134 assuming that the load at C has been removed.
P4.134 The rigid L-shaped member ABF is supported by a ball-and-
socket joint at A and by three cables. For the loading shown, determine
the tension in each cable and the reaction at A.

SOLUTION
First
(18 in.) i + (13.5 in.) k
TBG = BGTBG = TBG
(18) 2
+ (13.5 ) in.
2

= TBG ( 0.8i + 0.6k )

(18 in.) i + ( 24 in.) j


TDH = DH TDH = TDH
(18)2 + ( 24 )2 in.
= TDH ( 0.6i + 0.8 j)

Since FJ = DH

TFJ = TFJ ( 0.6i + 0.8 j)

From f.b.d. of member ABF


M A( x-axis ) = 0: ( 0.8TFJ ) ( 48 in.) + ( 0.8TDH )( 24 in.) (120 lb )( 36 in.) = 0
3.2TFJ + 1.6TDH = 360 (1)

M A( z -axis ) = 0: ( 0.8TFJ ) (18 in.) + ( 0.8TDH )(18 in.) (120 lb )(18 in.) = 0
3.2TFJ 3.2TDH = 480 (2)

Equation (1) + Equation (2) TDH = 75.0 lb W

Substituting into Equation (2) TFJ = 75.0 lb W

M A( y -axis ) = 0: ( 0.6TFJ ) ( 48 in.) + ( 0.6TDH )( 24 in.) ( 0.6TBG ) (18 in.) = 0


or ( 75.0 lb )( 48 in.) + ( 75.0 lb )( 24 in.) = TBG (18 in.)
TBG = 300 lb W
PROBLEM 4.135 CONTINUED

Fx = 0: 0.6TFJ 0.6TDH 0.8TBG + Ax = 0

0.6 ( 75.0 + 75.0 ) 0.8 ( 300 ) + Ax = 0

Ax = 330 lb

Fy = 0: 0.8TFJ + 0.8TDH 120 lb + Ay = 0

0.8 (150 lb ) 120 lb + Ay = 0

Ay = 0

Fz = 0: 0.6TBG + Az = 0

0.6 ( 300 lb ) + Az = 0

Az = 180 lb

Therefore A = ( 330 lb ) i (180 lb ) k W


PROBLEM 4.136
In order to clean the clogged drainpipe AE, a plumber has disconnected
both ends of the pipe and inserted a power snake through the opening at
A. The cutting head of the snake is connected by a heavy cable to an
electric motor which rotates at a constant speed as the plumber forces the
cable into the pipe. The forces exerted by the plumber and the motor on
the end of the cable can be represented by the wrench F = ( 60 N ) k ,
M = (108 N m ) k. Determine the additional reactions at B, C, and D
caused by the cleaning operation. Assume that the reaction at each
support consists of two force components perpendicular to the pipe.

SOLUTION
From f.b.d. of pipe assembly ABCD
Fx = 0: Bx = 0

M D( x-axis ) = 0: ( 60 N )( 2.5 m ) Bz ( 2 m ) = 0
Bz = 75.0 N

and B = ( 75.0 N ) k

M D( z -axis ) = 0: C y ( 3 m ) 108 N m = 0

C y = 36.0 N

M D( y -axis ) = 0: Cz ( 3 m ) ( 75 N )( 4 m ) + ( 60 N )( 4 m ) = 0

C z = 20.0 N

and C = ( 36.0 N ) j ( 20.0 N ) k

Fy = 0: Dy + 36.0 = 0

D y = 36.0 N

Fz = 0: Dz 20.0 N + 75.0 N 60 N = 0

Dz = 5.00 N

and D = ( 36.0 N ) j + ( 5.00 N ) k


PROBLEM 4.137
Solve Problem 4.136 assuming that the plumber exerts a force
F = ( 60 N ) k and that the motor is turned off ( M = 0 ) .
P4.136 In order to clean the clogged drainpipe AE, a plumber has
disconnected both ends of the pipe and inserted a power snake through
the opening at A. The cutting head of the snake is connected by a heavy
cable to an electric motor which rotates at a constant speed as the
plumber forces the cable into the pipe. The forces exerted by the plumber
and the motor on the end of the cable can be represented by the wrench
F = ( 60 N ) k , M = (108 N m ) k. Determine the additional
reactions at B, C, and D caused by the cleaning operation. Assume that
the reaction at each support consists of two force components
perpendicular to the pipe.

SOLUTION
From f.b.d. of pipe assembly ABCD
Fx = 0: Bx = 0

M D( x-axis ) = 0: ( 60 N )( 2.5 m ) Bz ( 2 m ) = 0
Bz = 75.0 N

and B = ( 75.0 N ) k

M D( z -axis ) = 0: C y ( 3 m ) Bx ( 2 m ) = 0

Cy = 0

M D( y -axis ) = 0: Cz ( 3 m ) ( 75.0 N )( 4 m ) + ( 60 N )( 4 m ) = 0

C z = 20 N

and C = ( 20.0 N ) k

Fy = 0: Dy + C y = 0

Dy = 0

Fz = 0: Dz + Bz + C z F = 0

Dz + 75 N 20 N 60 N = 0

Dz = 5.00 N

and D = ( 5.00 N ) k
PROBLEM 4.138
Three rods are welded together to form a corner which is supported by
three eyebolts. Neglecting friction, determine the reactions at A, B, and C
when P = 240 N, a = 120 mm, b = 80 mm, and c = 100 mm.

SOLUTION
From f.b.d. of weldment
M O = 0: rA/O A + rB/O B + rC/O C = 0

i j k i j k i j k
120 0 0 + 0 80 0 + 0 0 100 = 0
0 Ay Az Bx 0 Bz Cx Cy 0

( 120 Az j + 120 Ayk ) + (80Bzi 80Bxk ) + ( 100C yi + 100Cx j) = 0


From i-coefficient 80Bz 100C y = 0

or Bz = 1.25C y (1)

j-coefficient 120 Az + 100Cx = 0

or Cx = 1.2 Az (2)

k-coefficient 120 Ay 80Bx = 0

or Bx = 1.5 Ay (3)

F = 0: A + B + C P = 0

or ( Bx + Cx ) i + ( Ay + C y 240 N ) j + ( Az + Bz ) k = 0
From i-coefficient Bx + Cx = 0

or Cx = Bx (4)

j-coefficient Ay + C y 240 N = 0

or Ay + C y = 240 N (5)

k-coefficient Az + Bz = 0

or Az = Bz (6)
PROBLEM 4.138 CONTINUED

Substituting Cx from Equation (4) into Equation (2)

Bz = 1.2 Az (7)
Using Equations (1), (6), and (7)
Bz Az 1 Bx Bx
Cy = = = = (8)
1.25 1.25 1.25 1.2 1.5
From Equations (3) and (8)
1.5 Ay
Cy = or C y = Ay
1.5
and substituting into Equation (5)
2 Ay = 240 N

Ay = C y = 120 N (9)

Using Equation (1) and Equation (9)


Bz = 1.25 (120 N ) = 150.0 N

Using Equation (3) and Equation (9)


Bx = 1.5 (120 N ) = 180.0 N

From Equation (4) Cx = 180.0 N

From Equation (6) Az = 150.0 N

Therefore A = (120.0 N ) j (150.0 N ) k

B = (180.0 N ) i + (150.0 N ) k

C = (180.0 N ) i + (120.0 N ) j
PROBLEM 4.139
Solve Problem 4.138 assuming that the force P is removed and is
replaced by a couple M = + ( 6 N m ) j acting at B.
P4.138 Three rods are welded together to form a corner which is
supported by three eyebolts. Neglecting friction, determine the reactions
at A, B, and C when P = 240 N, a = 120 mm, b = 80 mm, and
c = 100 mm.

SOLUTION
From f.b.d. of weldment
M O = 0: rA/O A + rB/O B + rC/O C + M = 0

i j k i j k i j k
0.12 0 0 + 0 0.08 0 + 0 0 0.1 + ( 6 N m ) j = 0
0 Ay Az Bx 0 Bz Cx Cy 0

( 0.12 Az j + 0.12 Ayk ) + ( 0.08Bz j 0.08Bxk )


( )
+ 0.1C y i + 0.1Cx j + ( 6 N m ) j = 0

From i-coefficient 0.08Bz 0.1C y = 0

or C y = 0.8Bz (1)

j-coefficient 0.12 Az + 0.1Cx + 6 = 0

or Cx = 1.2 Az 60 (2)

k-coefficient 0.12 Ay 0.08Bx = 0

or Bx = 1.5 Ay (3)

F = 0: A + B + C = 0

( Bx + Cx ) i + ( Ay + C y ) j + ( Az + Bz ) k = 0

From i-coefficient Cx = Bx (4)

j-coefficient C y = Ay (5)

k-coefficient Az = Bz (6)

Substituting Cx from Equation (4) into Equation (2)

B
Az = 50 x (7)
1.2
PROBLEM 4.139 CONTINUED

Using Equations (1), (6), and (7)


2
C y = 0.8Bz = 0.8 Az = Bx 40 (8)
3
From Equations (3) and (8)
C y = Ay 40

Substituting into Equation (5) 2 Ay = 40

Ay = 20.0 N

From Equation (5) C y = 20.0 N

Equation (1) Bz = 25.0 N

Equation (3) Bx = 30.0 N

Equation (4) Cx = 30.0 N

Equation (6) Az = 25.0 N

Therefore A = ( 20.0 N ) j + ( 25.0 N ) k

B = ( 30.0 N ) i ( 25.0 N ) k

C = ( 30.0 N ) i ( 20.0 N ) j
PROBLEM 4.140
The uniform 10-lb rod AB is supported by a ball-and-socket joint at A and
leans against both the rod CD and the vertical wall. Neglecting the effects
of friction, determine (a) the force which rod CD exerts on AB, (b) the
reactions at A and B. (Hint: The force exerted by CD on AB must be
perpendicular to both rods.)

SOLUTION
(a) The force acting at E on the f.b.d. of rod AB is perpendicular to AB
and CD. Letting E = direction cosines for force E,

rB/ A k
E =
rB/ A k

( 32 in.) i + ( 24 in.) j ( 40 in.) k k


=
( 32 ) + ( 24 ) in.
2 2

= 0.6i + 0.8 j

Also, W = (10 lb ) j

B = Bk
E = E ( 0.6i + 0.8 j)

From f.b.d. of rod AB


M A = 0: rG/ A W + rE/ A E + rB/ A B = 0

i j k i j k i j k
16 12 20 (10 lb ) + 24 18 30 E + 32 24 40 B = 0
0 1 0 0.6 0.8 0 0 0 1

( 20i + 16k )(10 lb ) + ( 24i 18 j 30k ) E + ( 24i + 32 j) B = 0


From k-coefficient 160 30 E = 0
E = 5.3333 lb
and E = 5.3333 lb ( 0.6i + 0.8 j)

or E = ( 3.20 lb ) i + ( 4.27 lb ) j

(b) From j-coefficient 18 ( 5.3333 lb ) + 32 B = 0

B = 3.00 lb
or B = ( 3.00 lb ) k
PROBLEM 4.140 CONTINUED

From f.b.d. of rod AB


F = 0: A + W + E + B = 0

Ax i + Ay j + Az k (10 lb ) j + ( 3.20 lb ) i + ( 4.27 lb ) j + ( 3.00 lb ) k = 0

From i-coefficient Ax + 3.20 lb = 0

Ax = 3.20 lb

j-coefficient Ay 10 lb + 4.27 lb = 0

Ay = 5.73 lb

k-coefficient Az + 3.00 lb = 0

Az = 3.00 lb

Therefore A = ( 3.20 lb ) i + ( 5.73 lb ) j ( 3.00 lb ) k


PROBLEM 4.141
A 21-in.-long uniform rod AB weighs 6.4 lb and is attached to a ball-and-
socket joint at A. The rod rests against an inclined frictionless surface and
is held in the position shown by cord BC. Knowing that the cord is 21 in.
long, determine (a) the tension in the cord, (b) the reactions at A and B.

SOLUTION

First note
W = ( 6.4 lb ) j

N B = N B ( 0.8j + 0.6k )

LAB = 21 in.

= ( xB )2 + (13 + 3)2 + ( 4 )2 = xB2 + (16 ) + ( 4 )


2 2

xB = 13 in.

TBC = BCTBC =
(13 in.) i + (16 in.) j ( 4 in.) k T
BC
21 in.
TBC
= (13i + 16 j 4k )
21
From f.b.d. of rod AB
M A = 0: rG/ A W + rB/ A N B + rC/ A TBC = 0

i j k i j k i j k
26TBC
6.5 8 2 + 13 16 4 N B + 1 0 0 =0
21
0 6.4 0 0 0.8 0.6 13 16 4

26TBC
(12.8i 41.6k ) + ( 12.8i 7.8j + 10.4k ) N B + ( 4 j + 16k ) =0
21
PROBLEM 4.141 CONTINUED

From i-coeff. 12.8 12.8N B = 0 N B = 1.00 lb

or N B = ( 0.800 lb ) j + ( 0.600 lb ) k

26
From j-coeff. 7.8 N B + 4 TBC = 0 TBC = 1.575 lb
21
From f.b.d. of rod AB
F = 0: A + W + N B + TBC = 0

( Axi + Ay j + Azk ) ( 6.4 lb ) j + ( 0.800 lb ) j + ( 0.600 lb ) k + 1.575


21

(13i + 16 j 4k ) = 0

From i-coefficient Ax = 0.975 lb

j-coefficient Ay = 4.40 lb

k-coefficient Az = 0.3 lb

(a) TBC = 1.575 lb

(b) A = ( 0.975 lb ) i + ( 4.40 lb ) j ( 0.300 lb ) k

N B = ( 0.800 lb ) j + ( 0.600 lb ) k
PROBLEM 4.142
While being installed, the 56-lb chute ABCD is attached to a wall with
brackets E and F and is braced with props GH and IJ. Assuming that the
weight of the chute is uniformly distributed, determine the magnitude of
the force exerted on the chute by prop GH if prop IJ is removed.

SOLUTION

First note
42 in.
= tan 1 = 16.2602
144 in.

xG = ( 50 in.) cos16.2602 = 48 in.

yG = 78 in. ( 50 in.) sin16.2602 = 64 in.

(144 in.) i + ( 42 in.) j 1


BA = = ( 24i + 7 j)
(144 ) 2
+ ( 42 ) in.
2 25

rK / A = ( 72 in.) i ( 21 in.) j + ( 9 in.) k

rG/ A = ( 48 in.) i ( 78 in. 64 in.) j + (18 in.) k = ( 48 in.) i (14 in.) j + (18 in.) k

W = ( 56 lb ) j

PHG = HG PHG

( 2 in.) i + ( 64 in.) j (16 in.) k


= PHG
( 2 ) + ( 64 ) + (16 ) in.
2 2 2

PHG
= ( i + 32 j 8k )
33
PROBLEM 4.142 CONTINUED

From the f.b.d. of the chute

M BA = 0: BA ( rK / A W ) + BA ( rG/ A PHG ) = 0

24 7 0 24 7 0
56 P
72 21 9 + 48 14 18 HG = 0
25 ( )
33 25
0 1 0 1 32 8

216 ( 56 )
+ [ 24 ( 14 ) ( 8 ) ( 24 ) (18 )( 32 ) + 7 (18 ) ( 1) ( 7 )( 48 )( 8 )] HG = 0
P
25 33 ( 25 )

PHG = 29.141 lb

or PHG = 29.1 lb
PROBLEM 4.143
While being installed, the 56-lb chute ABCD is attached to a wall with
brackets E and F and is braced with props GH and IJ. Assuming that the
weight of the chute is uniformly distributed, determine the magnitude of
the force exerted on the chute by prop IJ if prop GH is removed.

SOLUTION

First note
42 in.
= tan 1 = 16.2602
144 in.

xI = (100 in.) cos16.2602 = 96 in.

yI = 78 in. (100 in.) sin16.2602 = 50 in.

(144 in.) i + ( 42 in.) j 1


BA = = ( 24i + 7 j)
(144 ) 2
+ ( 42 ) in.
2 25

rK / A = ( 72 in.) i ( 21 in.) j + ( 9 in.) k

rI / A = ( 96 in.) i ( 78 in. 50 in.) j + (18 in.) k = ( 96 in.) i ( 28 in.) j + (18 in.) k

W = ( 56 lb ) j

PJI = JI PJI

(1 in.) i + ( 50 in.) j (10 in.) k


= PJI
(1) 2
+ ( 50 ) + (10 ) in.
2 2

PJI
= ( i + 50 j 10k )
51
PROBLEM 4.143 CONTINUED

From the f.b.d. of the chute

M BA = 0: BA ( rK / A W ) + BA ( rI / A PJI ) = 0

24 7 0 24 7 0
56 P
72 21 9 + 96 28 18 JI = 0
25 51( 25 )
0 1 0 1 50 10

216 ( 56 )
+ [ 24 ( 28 )( 10 ) ( 24 ) (18 )( 50 ) + 7 (18 ) ( 1) ( 7 )( 96 )( 10 )] JI = 0
P
25 51( 25 )

PJI = 28.728 lb

or PJI = 28.7 lb
PROBLEM 4.144
To water seedlings, a gardener joins three lengths of pipe, AB, BC,
and CD, fitted with spray nozzles and suspends the assembly using
hinged supports at A and D and cable EF. Knowing that the pipe
weighs 0.85 lb/ft, determine the tension in the cable.

SOLUTION
First note rG/ A = (1.5 ft ) i

WAB = ( 0.85 lb/ft )( 3 ft ) j = ( 2.55 lb ) j

rF / A = ( 2 ft ) i

( 2 ft ) i + ( 3 ft ) j ( 4.5 ft ) k
T = FET = T
( 2 )2 + ( 3)2 + ( 4.5)2 ft

T
= ( 2i + 3j 4.5k )
33.25

rB/ A = ( 3 ft ) i

WBC = ( 0.85 lb/ft )(1 ft ) j = ( 0.85 lb ) j

rH / A = ( 3 ft ) i ( 2.25 ft ) k

WCD = ( 0.85 lb/ft )( 4.5 ft ) j = ( 3.825 lb ) j

( 3 ft ) i (1 ft ) j ( 4.5 ft ) k 1
AD = = ( 3i j 4.5k )
( 3)2 + (1)2 + ( 4.5)2 ft 5.5
PROBLEM 4.144 CONTINUED

From f.b.d. of the pipe assembly

M AD = 0: AD ( rG/ A WAB ) + AD ( rF / A T )

+ AD ( rB/ A WBC ) + AD ( rH / A WCD ) = 0

3 1 4.5 3 1 4.5
1 T
1.5 0 0 + 2 0 0
5.5 5.5 33.25
0 2.55 0 2 3 4.5

3 1 4.5 3 1 4.5
1 1
+ 3 0 0 + 3 0 2.25 =0
5.5 5.5
0 0.85 0 0 3.825 0

T
(17.2125) + ( 36 ) + (11.475 ) + ( 25.819 ) = 0
33.25

T = 8.7306 lb

or T = 8.73 lb
PROBLEM 4.145
Solve Problem 4.144 assuming that cable EF is replaced by a cable
connecting E and C.
P4.144 To water seedlings, a gardener joins three lengths of pipe, AB,
BC, and CD, fitted with spray nozzles and suspends the assembly using
hinged supports at A and D and cable EF. Knowing that the pipe weighs
0.85 lb/ft, determine the tension in the cable.

SOLUTION
First note rG/ A = (1.5 ft ) i

WAB = ( 0.85 lb/ft )( 3 ft ) j = ( 2.55 lb ) j

rC/ A = ( 3 ft ) i (1 ft ) j

( 3 ft ) i + ( 4 ft ) j ( 4.5 ft ) k
T = CET = T
( 3)2 + ( 4 )2 + ( 4.5)2 ft

T
= ( 3i + 4 j 4.5k )
45.25

rB/ A = ( 3 ft ) i

WBC = ( 0.85 lb/ft )(1 ft ) j = ( 0.85 lb ) j

rH / A = ( 3 ft ) i ( 2.25 ft ) k

WCD = ( 0.85 lb/ft )(1 ft ) j = ( 3.825 lb ) j

( 3 ft ) i (1 ft ) j ( 4.5 ft ) k 1
AD = = ( 3i j 4.5k )
( 3)2 + (1)2 + ( 4.5)2 ft 5.5
PROBLEM 4.145 CONTINUED

From f.b.d. of the pipe assembly

M AD = 0: AD ( rG/ A WAB ) + AD ( rC/ A T )

+ AD ( rB/ A WBC ) + AD ( rH / A WCD ) = 0

3 1 4.5 3 1 4.5
1 T
1.5 0 0 + 3 1 0
5.5 5.5 45.25
0 2.55 0 3 4 4.5

3 1 4.5 3 1 4.5
1 1
+3 0 0 + 3 0 2.25 =0
5.5 5.5
0 0.85 0 0 3.825 0

T
(17.2125) + ( 40.5) + (11.475 ) + ( 25.819 ) = 0
45.25

T = 9.0536 lb

or T = 9.05 lb
PROBLEM 4.146
The bent rod ABDE is supported by ball-and-socket joints at A and E and
by the cable DF. If a 600-N load is applied at C as shown, determine the
tension in the cable.

SOLUTION
First note
( 70 mm ) i + ( 240 mm ) k 1
AE = = ( 7i + 24k )
( 70 ) 2
+ ( 240 ) mm
2 25

rC/ A = ( 90 mm ) i + (100 mm ) k

FC = ( 600 N ) j

rD/ A = ( 90 mm ) i + ( 240 mm ) k

(160 mm ) i + (110 mm ) j ( 80 mm ) k
T = DFT = T
(160 )2 + (110 )2 + (80 )2 mm

T
= ( 16 i + 11j 8k )
21
From the f.b.d. of the bend rod

( ) (
M AE = 0: AE rC/ A FC + AE rD/ A T = 0 )
7 0 24 7 0 24
600 T
90 0 100 + 90 0 240 =0
25 25 ( 21)
0 1 0 16 11 8

( 700 2160 )
600 T
+ (18 480 + 23 760 ) =0
25 25 ( 21)
T = 853.13 N
or T = 853 N
PROBLEM 4.147
Solve Problem 4.146 assuming that cable DF is replaced by a cable
connecting B and F.
P4.146 The bent rod ABDE is supported by ball-and-socket joints at A
and E and by the cable DF. If a 600-N load is applied at C as shown,
determine the tension in the cable.

SOLUTION
First note

( 70 mm ) i + ( 240 mm ) k 1
AE = = ( 7i + 24k )
( 70 ) 2
+ ( 240 ) mm
2 25

rC/ A = ( 90 mm ) i + (100 mm ) k

FC = ( 600 N ) j

rB/ A = ( 90 mm ) i

(160 mm ) i + (110 mm ) j + (160 mm ) k


T = BFT = T
(160 )2 + (110 )2 + (160 )2 mm

1
= ( 160 i + 110 j + 160k )
251.59

From the f.b.d. of the bend rod

( ) (
M AE = 0: AE rC/ A FC + AE rB/ A T = 0 )
7 0 24 7 0 24
600 T
90 0 100 + 90 0 0 =0
25 25 ( 251.59 )
0 1 0 160 110 160

( 700 2160 )
600 T
+ ( 237 600 ) =0
25 (
25 251.59 )
T = 1817.04 N

or T = 1817 N
PROBLEM 4.148
Two rectangular plates are welded together to form the assembly shown.
The assembly is supported by ball-and-socket joints at B and D and by a
ball on a horizontal surface at C. For the loading shown, determine the
reaction at C.

SOLUTION

( 80 mm ) i ( 90 mm ) j + (120 mm ) k
First note BD =
(80 )2 + ( 90 )2 + (120 )2 mm

1
= ( 8i 9 j + 12k )
17
rA/B = ( 60 mm ) i

P = ( 200 N ) k

rC/D = ( 80 mm ) i

C = (C ) j
From the f.b.d. of the plates

( ) (
M BD = 0: BD rA/B P + BD rC/D C = 0 )
8 9 12 8 9 12
60 ( 200 ) C ( 80 )
1 0 0 + 1 0 0 =0
17 17
0 0 1 0 1 0

( 9 )( 60 )( 200 ) + (12 )(80 ) C = 0


C = 112.5 N or C = (112.5 N ) j
PROBLEM 4.149
Two 1 2-m plywood panels, each of mass 15 kg, are nailed together as
shown. The panels are supported by ball-and-socket joints at A and F and
by the wire BH. Determine (a) the location of H in the xy plane if the
tension in the wire is to be minimum, (b) the corresponding minimum
tension.

SOLUTION

Let (
W1 = W2 = ( mg ) j = (15 kg ) 9.81 m/s 2 j )
= (147.15 N ) j

From the f.b.d. of the panels

( ) ( )
M AF = 0: AF rG/ A W1 + AF rB/ A T + AF rT / A W2 = 0( )
( 2 m ) i (1 m ) j ( 2 m ) k 1
where AF = = ( 2i j 2k )
( 2 )2 + (1)2 + ( 2 )2 m 3

rG/ A = (1 m ) i

rB/ A = ( 2 m ) i

rI / A = ( 2 m ) i (1 m ) k
PROBLEM 4.149 CONTINUED

( x 2) i + ( y ) j ( 2) k
BH =
( x 2 ) 2 + y 2 + ( 2 )2

( x 2) i + ( y ) j ( 2) k
T = BH T =
( x 2 ) 2 + y 2 + ( 2 )2

2 1 2 2 1 2 2 1 2
147.15 T 147.15
1 0 0 + 2 0 0 + 2 0 1 3 = 0
3
x 2 y 2 3 ( x 2 ) + y + ( 2 )
2 2 2
0 1 0 0 1 0

2 (147.15 ) T 147.15
+ ( 4 4 y ) + ( 2 + 4 ) =0
3 3 ( x 2) + y 2 + ( 2) 3
2 2

147.15
or T = ( x 2 ) 2 + y 2 + ( 2 )2
1+ y

( y + 4) 2
1
147.15 2
For x 2 m, T = Tmin Tmin =
(1 + y )

(1 + y ) ( y 2 + 4 ) 2 ( 2 y ) ( y 2 + 4 ) 2 (1)
1 1 1

dT 2
The y-value for Tmin is found from dy = 0: =0
(1 + y )2
Setting the numerator equal to zero, (1 + y ) y = y2 + 4

y = 4m

147.15 (
2 2 ) + ( 4 ) + ( 2 ) = 131.615 N
2 2 2
Then T min =
(1 + 4 )
(a) x = 2.00 m, y = 4.00 m

(b) Tmin = 131.6 N


PROBLEM 4.150
Solve Problem 4.149 subject to the restriction that H must lie on the
y axis.
P4.149 Two 1 2-m plywood panels, each of mass 15 kg, are nailed
together as shown. The panels are supported by ball-and-socket joints at
A and F and by the wire BH. Determine (a) the location of H in the xy
plane if the tension in the wire is to be minimum, (b) the corresponding
minimum tension.

SOLUTION

Let ( )
W1 = W2 = ( mg ) j = (15 kg ) 9.81 m/s 2 j = (147.15 N ) j

From the f.b.d. of the panels

( ) ( )
M AF = 0: AF rG/ A W1 + AF rB/ A T + AF rI / A W2 = 0 ( )
( 2 m ) i (1 m ) j ( 2 m ) k 1
where AF = = ( 2i j 2k )
( 2 )2 + (1)2 + ( 2 )2 m 3

rG/ A = (1 m ) i

rB/ A = ( 2 m ) i

rI / A = ( 2 m ) i (1 m ) k

(2 m) i + ( y) j (2 m) k
T = BH T = T
( 2 )2 + ( y )2 + ( 2 )2 m

T
= ( 2i + yj 2k )
8 + y2
PROBLEM 4.150 CONTINUED

2 1 2 2 1 2 2 1 2
147.15 T 147.15
1 0 0 + 2 0 0 + 2 0 1 3 = 0
3
2 y 2 3 8 + y
2
0 1 0 0 1 0

( )
2 (147.15 ) + ( 4 4 y ) T 8 + y 2 + ( 2 )147.15 = 0

T =
(147.15) 8 + y2
(1 + y )

(1 + y ) 12 (8 + ) ( 2 y ) (8 + )
12 1

dT y2 y2 2
(1)
For Tmin , =0 =0
dy (1 + y )2
Setting the numerator equal to zero,

(1 + y ) y = 8 + y2

y = 8.00 m

(147.15) 8 + (8)2
and Tmin = = 138.734 N
(1 + 8)
(a) x = 0, y = 8.00 m

(b) Tmin = 138.7 N


PROBLEM 4.151
A uniform 20 30-in. steel plate ABCD weighs 85 lb and is attached to
ball-and-socket joints at A and B. Knowing that the plate leans against a
frictionless vertical wall at D, determine (a) the location of D, (b) the
reaction at D.

SOLUTION
(a) Since rD/ A is perpendicular to rB/ A ,

rD/ A rB/ A = 0

where coordinates of D are ( 0, y, z ) , and

rD/ A = ( 4 in.) i + ( y ) j + ( z 28 in.) k

rB/ A = (12 in.) i (16 in.) k

rD/ A rB/ A = 48 16 z + 448 = 0

or z = 25 in.

Since LAD = 30 in.

30 = ( 4 )2 + ( y )2 + ( 25 28)2
900 = 16 + y 2 + 9

or y = 875 in. = 29.580 in.


Coordinates of D : x = 0, y = 29.6 in., z = 25.0 in.
(b) From f.b.d. of steel plate ABCD
M AB = 0: AB ( rD/ A N D ) + AB ( rG/B W ) = 0

(12 in.) i (16 in.) k 1


where AB = = ( 3i 4k )
(12 )2 + (16 )2 in. 5

rD/ A = ( 4 in.) i + ( 29.580 in.) j ( 3 in.) k

N D = N Di
PROBLEM 4.151 CONTINUED

1 1
rG/B = rD/B = (16 in.) i + ( 29.580 in.) j + ( 25 in. 12 in.) k
2 2
W = ( 85 lb ) j

3 0 4 3 0 4
ND 85
4 29.580 3 + 16 29.580 13 =0
5 2 ( 5 )
1 0 0 0 1 0

118.32 N D + ( 39 64 ) 42.5 = 0

N D = 8.9799 lb

or N D = ( 8.98 lb ) i
PROBLEM 4.152
Beam AD carries the two 40-lb loads shown. The beam is held by a fixed
support at D and by the cable BE which is attached to the counter-weight
W. Determine the reaction at D when (a) W = 100 lb, (b) W = 90 lb.

SOLUTION
(a) W = 100 lb
From f.b.d. of beam AD

Fx = 0: Dx = 0

Fy = 0: Dy 40 lb 40 lb + 100 lb = 0

Dy = 20.0 lb

or D = 20.0 lb

M D = 0: M D (100 lb )( 5 ft ) + ( 40 lb )( 8 ft )

+ ( 40 lb )( 4 ft ) = 0

M D = 20.0 lb ft

or M D = 20.0 lb ft

(b) W = 90 lb
From f.b.d. of beam AD

Fx = 0: Dx = 0

Fy = 0: Dy + 90 lb 40 lb 40 lb = 0

Dy = 10.00 lb

or D = 10.00 lb

M D = 0: M D ( 90 lb )( 5 ft ) + ( 40 lb )( 8 ft )

+ ( 40 lb )( 4 ft ) = 0

M D = 30.0 lb ft

or M D = 30.0 lb ft
PROBLEM 4.153
For the beam and loading shown, determine the range of values of W for
which the magnitude of the couple at D does not exceed 40 lb ft.

SOLUTION
For Wmin , M D = 40 lb ft
From f.b.d. of beam AD
M D = 0: ( 40 lb )(8 ft ) Wmin ( 5 ft ) + ( 40 lb )( 4 ft ) 40 lb ft =0

Wmin = 88.0 lb

For Wmax , M D = 40 lb ft
From f.b.d. of beam AD
M D = 0: ( 40 lb )(8 ft ) Wmax ( 5 ft ) + ( 40 lb )( 4 ft ) + 40 lb ft =0

Wmax = 104.0 lb

or 88.0 lb W 104.0 lb
PROBLEM 4.154
Determine the reactions at A and D when = 30.

SOLUTION
From f.b.d. of frame ABCD

M D = 0: A ( 0.18 m ) + (150 N ) sin 30 ( 0.10 m )

+ (150 N ) cos 30 ( 0.28 m ) = 0

A = 243.74 N
or A = 244 N

Fx = 0: ( 243.74 N ) + (150 N ) sin 30 + Dx =0

Dx = 318.74 N

Fy = 0: Dy (150 N ) cos 30 = 0

Dy = 129.904 N

Then D= ( Dx )2 + Dx2 = ( 318.74 )2 + (129.904 )2 = 344.19 N

Dy 1 129.904
and = tan 1 = tan = 22.174
x
D 318.74

or D = 344 N 22.2
PROBLEM 4.155
Determine the reactions at A and D when = 60.

SOLUTION
From f.b.d. of frame ABCD

M D = 0: A ( 0.18 m ) + (150 N ) sin 60 ( 0.10 m )

+ (150 N ) cos 60 ( 0.28 m ) = 0

A = 188.835 N
or A = 188.8 N

Fx = 0: (188.835 N ) + (150 N ) sin 60 + Dx =0

Dx = 318.74 N

Fy = 0: Dy (150 N ) cos 60 = 0

Dy = 75.0 N

( Dx )2 + ( Dy ) ( 318.74 )2 + ( 75.0 )2
2
Then D= = = 327.44 N

Dy 1 75.0
and = tan 1 = tan = 13.2409
Dx 318.74

or D = 327 N 13.24
PROBLEM 4.156
A 2100-lb tractor is used to lift 900 lb of gravel. Determine the reaction at
each of the two (a) rear wheels A, (b) front wheels B.

SOLUTION

(a) From f.b.d. of tractor

M B = 0: ( 2100 lb )( 40 in.) ( 2 A)( 60 in.) ( 900 lb )( 50 in.) = 0


A = 325 lb or A = 325 lb

(b) From f.b.d. of tractor

M A = 0: ( 2B )( 60 in.) ( 2100 lb )( 20 in.) ( 900 lb )(110 in.) = 0


B = 1175 lb or B = 1175 lb
PROBLEM 4.157
A tension of 5 lb is maintained in a tape as it passes the support system
shown. Knowing that the radius of each pulley is 0.4 in., determine the
reaction at C.

SOLUTION

From f.b.d. of system

Fx = 0: C x + ( 5 lb ) = 0

Cx = 5 lb

Fy = 0: C y ( 5 lb ) = 0

C y = 5 lb

( Cx )2 + ( C y ) ( 5 )2 + ( 5 )2
2
Then C = = = 7.0711 lb

+5
and = tan 1 = 45
5

or C = 7.07 lb 45.0

M C = 0: M C + ( 5 lb )( 6.4 in.) + ( 5 lb )( 2.2 in.) = 0

M C = 43.0 lb in or M C = 43.0 lb in.


PROBLEM 4.158
Solve Problem 4.157 assuming that 0.6-in.-radius pulleys are used.
P4.157 A tension of 5 lb is maintained in a tape as it passes the support
system shown. Knowing that the radius of each pulley is 0.4 in., determine
the reaction at C.

SOLUTION
From f.b.d of system

Fx = 0: C x + ( 5 lb ) = 0

Cx = 5 lb

Fy = 0: C y ( 5 lb ) = 0

C y = 5 lb

( C x )2 + ( C y ) ( 5 )2 + ( 5 )2
2
Then C = = = 7.0711 lb

5
and = tan 1 = 45.0
5

or C = 7.07 lb 45.0

M C = 0: M C + ( 5 lb )( 6.6 in.) + ( 5 lb )( 2.4 in.) = 0

M C = 45.0 lb in.

or M C = 45.0 lb in.
PROBLEM 4.159
The bent rod ABEF is supported by bearings at C and D and by wire AH.
Knowing that portion AB of the rod is 250 mm long, determine (a) the
tension in wire AH, (b) the reactions at C and D. Assume that the bearing
at D does not exert any axial thrust.

SOLUTION
(a) From f.b.d. of bent rod

( )
M CD = 0: CD rH /B T + CD rF /E F = 0 ( )
where CD = i

rH /B = ( 0.25 m ) j

T = AH T

=
( y AH ) j ( z AH ) k T
( y AH )2 + ( z AH )2
y AH = ( 0.25 m ) ( 0.25 m ) sin 30

= 0.125 m

z AH = ( 0.25 m ) cos 30

= 0.21651 m

T
T= ( 0.125j 0.21651k )
0.25
rF /E = ( 0.25 m ) k

F = 400 N j

1 0 0 1 0 0
T
0 1 0 ( )
0.25 0 0 1 ( 0.25 )( 400 N ) = 0
+
0.25
0 0.125 0.21651 0 1 0

0.21651T + 0.25 ( 400 N ) = 0

T = 461.88 N
or T = 462 N
PROBLEM 4.159 CONTINUED

(b) From f.b.d. of bent rod


Fx = 0: C x = 0

M D( z -axis ) = 0: ( 461.88 N ) sin 30 ( 0.35 m ) C y ( 0.3 m )

( 400 N )( 0.05 m ) = 0

C y = 336.10 N

M D( y -axis ) = 0: Cz ( 0.3 m ) ( 461.88 N ) cos 30 ( 0.35 m ) = 0

Cz = 466.67 N

or C = ( 336 N ) j + ( 467 N ) k

Fy = 0: Dy 336.10 N + ( 461.88 N ) sin 30 400 N = 0

Dy = 505.16 N

Fz = 0: Dz + 466.67 N ( 461.88 N ) cos30 = 0

Dz = 66.670 N

or D = ( 505 N ) j ( 66.7 N ) k
PROBLEM 4.160
For the beam and loading shown, determine (a) the reaction at A, (b) the
tension in cable BC.

SOLUTION
(a) From f.b.d of beam

Fx = 0: Ax = 0

M B = 0: (15 lb )( 28 in.) + ( 20 lb )( 22 in.) + ( 35 lb )(14 in.)


+ ( 20 lb )( 6 in.) Ay ( 6 in.) = 0

Ay = 245 lb

or A = 245 lb
(b) From f.b.d of beam
M A = 0: (15 lb )( 22 in.) + ( 20 lb )(16 in.) + ( 35 lb )(8 in.)
(15 lb )( 6 in.) TB ( 6 in.) = 0

TB = 140.0 lb

or TB = 140.0 lb
Check:
Fy = 0: 15 lb 20 lb 35 lb 20 lb

15 lb 140 lb + 245 lb = 0?
245 lb 245 lb = 0 ok
PROBLEM 4.161
Frame ABCD is supported by a ball-and-socket joint at A and by three
cables. For a = 150 mm, determine the tension in each cable and the
reaction at A.

SOLUTION
( 0.48 m ) i + ( 0.14 m ) j
First note TDG = DGTDG = TDG
( 0.48)2 + ( 0.14 )2 m

0.48i + 0.14 j
= TDG
0.50
TDG
= ( 24i + 7 j)
25
( 0.48 m ) i + ( 0.2 m ) k
TBE = BETBE = TBE
( 0.48)2 + ( 0.2 )2 m
0.48i + 0.2k
= TBE
0.52
TBE
= ( 12 j + 5k )
13
From f.b.d. of frame ABCD
7
M x = 0: TDG ( 0.3 m ) ( 350 N )( 0.15 m ) = 0
25
or TDG = 625 N

24 5
M y = 0: 625 N ( 0.3 m ) TBE ( 0.48 m ) = 0
25 13
or TBE = 975 N

7
M z = 0: TCF ( 0.14 m ) + 625 N ( 0.48 m )
25
( 350 N )( 0.48 m ) = 0

or TCF = 600 N
PROBLEM 4.161 CONTINUED

Fx = 0: Ax + TCF + (TBE ) x + (TDG ) x = 0

12 24
Ax 600 N 975 N 625 N = 0
13 25
Ax = 2100 N

Fy = 0: Ay + (TDG ) y 350 N = 0

7
Ay + 625 N 350 N = 0
25
Ay = 175.0 N

Fz = 0: Az + (TBE ) z = 0

5
Az + 975 N = 0
13
Az = 375 N

Therefore A = ( 2100 N ) i + (175.0 N ) j ( 375 N ) k


PROBLEM 4.162
Frame ABCD is supported by a ball-and-socket joint at A and by three
cables. Knowing that the 350-N load is applied at D (a = 300 mm),
determine the tension in each cable and the reaction at A.

SOLUTION
( 0.48 m ) i + ( 0.14 m ) j
First note TDG = DGTDG = TDG
( 0.48)2 + ( 0.14 )2 m
0.48i + 0.14 j
= TDG
0.50
TDG
= ( 24i + 7 j)
25
( 0.48 m ) i + ( 0.2 m ) k
TBE = BETBE = TBE
( 0.48)2 + ( 0.2 )2 m
0.48i + 0.2k
= TBE
0.52
TBE
= ( 12i + 5k )
13
From f.b.d of frame ABCD
7
M x = 0: TDG ( 0.3 m ) ( 350 N )( 0.3 m ) = 0
25
or TDG = 1250 N

24 5
M y = 0: 1250 N ( 0.3 m ) TBE ( 0.48 m ) = 0
25 13
or TBE = 1950 N

7
M z = 0: TCF ( 0.14 m ) + 1250 N ( 0.48 m )
25
( 350 N )( 0.48 m ) = 0

or TCF = 0
PROBLEM 4.162 CONTINUED

Fx = 0: Ax + TCF + (TBE ) x + (TDG ) x = 0

12 24
Ax + 0 1950 N 1250 N = 0
13 25
Ax = 3000 N

Fy = 0: Ay + (TDG ) y 350 N = 0

7
Ay + 1250 N 350 N = 0
25
Ay = 0

Fz = 0: Az + (TBE ) z = 0

5
Az + 1950 N = 0
13
Az = 750 N

Therefore A = ( 3000 N ) i ( 750 N ) k


PROBLEM 4.163
In the problems listed below, the rigid bodies considered were completely
constrained and the reactions were statically determinate. For each of
these rigid bodies it is possible to create an improper set of constraints by
changing a dimension of the body. In each of the following problems
determine the value of a which results in improper constraints.
(a) Problem 4.81, (b) Problem 4.82.

SOLUTION

(a)
(a) M B = 0: ( 300 lb )(16 in.) T (16 in.) + T ( a ) = 0

or T =
( 300 lb )(16 in.)
(16 a ) in.
T becomes infinite when
16 a = 0
or a = 16.00 in.

8
(b) M C = 0: (T 80 N )( 0.2 m ) T ( 0.175 m )
(b) 17

15
T ( 0.4 m a ) = 0
17
0.2T 16.0 0.82353T 0.35294T + 0.88235Ta = 0
16.0
or T =
0.88235a 0.23529
T becomes infinite when
0.88235a 0.23529 = 0
a = 0.26666 m
or a = 267 mm

You might also like